Consti Cases Territory National Economy And Patrimony.docx

  • Uploaded by: Jean Ben Go Singson
  • 0
  • 0
  • December 2019
  • PDF

This document was uploaded by user and they confirmed that they have the permission to share it. If you are author or own the copyright of this book, please report to us by using this DMCA report form. Report DMCA


Overview

Download & View Consti Cases Territory National Economy And Patrimony.docx as PDF for free.

More details

  • Words: 57,418
  • Pages: 129
anada vs Angara Justiciable Question; Theory of Auto-Limitation; Declaration of Principles and State Policies

TANADA VS ANGARA G.R. No. 118295

May 2, 1997

Wigberto E. Tanada et al, in representation of various taxpayers and as non-governmental organizations, petitioners, vs. EDGARDO ANGARA, et al, respondents.

Facts: This is a case petition by Sen. Wigberto Tanada, together with other lawmakers, taxpayers, and various NGO’s to nullify the Philippine ratification of the World Trade Organization (WTO) Agreement.

Petitioners believe that this will be detrimental to the growth of our National Economy and against to the “Filipino First” policy. The WTO opens access to foreign markets, especially its major trading partners, through the reduction of tariffs on its exports, particularly agricultural and industrial products. Thus, provides new opportunities for the service sector cost and uncertainty associated with exporting and more investment in the country. These are the predicted benefits as reflected in the agreement and as viewed by the signatory Senators, a “free market” espoused by WTO.

Petitioners also contends that it is in conflict with the provisions of our constitution, since the said Agreement is an assault on the sovereign powers of the Philippines because it meant that Congress could not pass legislation that would be good for national interest and general welfare if such legislation would not conform to the WTO Agreement.

Issues: 1. Whether or not the petition present a justiciable controversy.

2. Whether or not the provisions of the ‘Agreement Establishing the World Trade Organization and the Agreements and Associated Legal Instruments included in Annexes one (1), two (2) and three (3) of that agreement’ cited by petitioners directly contravene or undermine the letter, spirit and intent of Section 19, Article II and Sections 10 and 12, Article XII of the 1987 Constitution. 3. Whether or not certain provisions of the Agreement unduly limit, restrict or impair the exercise of legislative power by Congress. 4. Whether or not certain provisions of the Agreement impair the exercise of judicial power by this Honorable Court in promulgating the rules of evidence. 5. Whether or not the concurrence of the Senate ‘in the ratification by the President of the Philippines of the Agreement establishing the World Trade Organization’ implied rejection of the treaty embodied in the Final Act.

Discussions: 

1987 Constitution states that Judicial power includes the duty of the courts of justice to settle actual controversies involving rights which are legally demandable and enforceable, and to determine whether or not there has been a grave abuse of discretion amounting to lack or excess of jurisdiction on the part of any branch or instrumentality of the government.



Although the Constitution mandates to develop a self-reliant and independent national economy controlled by Filipinos, does not necessarily rule out the entry of foreign investments, goods and services. It contemplates neither “economic seclusion” nor “mendicancy in the international community.” The WTO itself has some built-in advantages to protect weak and developing economies, which comprise the vast majority of its members. Unlike in the UN where major states have permanent seats and veto powers in the Security Council, in the WTO, decisions are made on the basis of sovereign equality, with each member’s vote equal in weight to that of any other. Hence, poor countries can protect their common interests more effectively through the WTO than through one-on-one negotiations with developed countries. Within the WTO, developing countries can form powerful blocs to push their economic agenda more decisively than outside the Organization. Which is not merely a matter of practical alliances but a negotiating strategy rooted in law. Thus, the basic principles underlying the WTO Agreement recognize the need of developing countries like the Philippines to “share in the growth in international trade commensurate with the needs of their economic development.”



In its Declaration of Principles and State Policies, the Constitution “adopts the generally accepted principles of international law as part of the law of the land, and adheres to the policy of peace, equality, justice, freedom, cooperation and amity, with all nations. By the doctrine of incorporation, the country is bound by generally accepted principles of international law, which are considered to be automatically part of our own laws. A state which has contracted valid international obligations is bound to make in its legislations such modifications as may be necessary to ensure the fulfillment of the obligations undertaken. Paragraph 1, Article 34 of the General Provisions and Basic Principles of the Agreement on

Trade-Related Aspects of Intellectual Property Rights (TRIPS) may intrudes on the power of the Supreme Court to promulgate rules concerning pleading, practice and procedures. With regard to Infringement of a design patent, WTO members shall be free to determine the appropriate method of implementing the provisions of TRIPS within their own internal systems and processes. 

The alleged impairment of sovereignty in the exercise of legislative and judicial powers is balanced by the adoption of the generally accepted principles of international law as part of the law of the land and the adherence of the Constitution to the policy of cooperation and amity with all nations. The Senate, after deliberation and voting, voluntarily and overwhelmingly gave its consent to the WTO Agreement thereby making it “a part of the law of the land” is a legitimate exercise of its sovereign duty and power.

Rulings: 1. In seeking to nullify an act of the Philippine Senate on the ground that it contravenes the Constitution, the petition no doubt raises a justiciable controversy. Where an action of the legislative branch is seriously alleged to have infringed the Constitution, it becomes not only the right but in fact the duty of the judiciary to settle the dispute. As explained by former Chief Justice Roberto Concepcion, “the judiciary is the final arbiter on the question of whether or not a branch of government or any of its officials has acted without jurisdiction or in excess of jurisdiction or so capriciously as to constitute an abuse of discretion amounting to excess of jurisdiction. This is not only a judicial power but a duty to pass judgment on matters of this nature.” 2. While the Constitution indeed mandates a bias in favor of Filipino goods, services, labor and enterprises, at the same time, it recognizes the need for business exchange with the rest of the world on the bases of equality and reciprocity and limits protection of Filipino enterprises only against foreign competition and trade practices that are unfair. In other words, the Constitution did not intend to pursue an isolationist policy. It did not shut out foreign investments, goods and services in the development of the Philippine economy. While the Constitution does not encourage the unlimited entry of foreign goods, services and investments into the country, it does not prohibit them either. In fact, it allows an exchange on the basis of equality and reciprocity, frowning only on foreign competition that is unfair. 3. By their inherent nature, treaties really limit or restrict the absoluteness of sovereignty. By their voluntary act, nations may surrender some aspects of their state power in exchange for greater benefits granted by or derived from a convention or pact. After all, states, like individuals, live with coequals, and in pursuit of mutually covenanted objectives and benefits, they also commonly agree to limit the exercise of their otherwise absolute rights. As shown by the foregoing treaties Philippines has entered, a portion of sovereignty may be waived without violating the Constitution, based on the rationale that the Philippines “adopts the generally accepted principles of international law as part of the law of the land and adheres to the policy of cooperation and amity with all nations.”

4. The provision in Article 34 of WTO agreement does not contain an unreasonable burden, consistent as it is with due process and the concept of adversarial dispute settlement inherent in our judicial system. 5. The assailed Senate Resolution No. 97 expressed concurrence in exactly what the Final Act required from its signatories, namely, concurrence of the Senate in the WTO Agreement. Moreover, the Senate was wellaware of what it was concurring in as shown by the members’ deliberation on August 25, 1994. After reading the letter of President Ramos dated August 11, 1994, the senators of the Republic minutely dissected what the Senate was concurring in.

Case Brief: Tanada v Angara NOVEMBER 24, 2014JEFF REY

G.R. No. 118295 May 2, 1997 WIGBERTO E. TAÑADA et al, petitioners, vs. EDGARDO ANGARA, et al, respondents. Facts: Petitioners prayed for the nullification, on constitutional grounds, of the concurrence of the Philippine Senate in the ratification by the President of the Philippines of the Agreement Establishing the World Trade Organization (WTO Agreement, for brevity) and for the prohibition of its implementation and enforcement through the release and utilization of public funds, the assignment of public officials and employees, as well as the use of government properties and resources by respondent-heads of various executive offices concerned therewith. They contended that WTO agreement violates the mandate of the 1987 Constitution to “develop a self-reliant and independent national economy effectively controlled by Filipinos x x x (to) give preference to qualified Filipinos (and to) promote the preferential use of Filipino labor, domestic materials and locally produced goods” as (1) the WTO requires the Philippines “to place nationals and products of membercountries on the same footing as Filipinos and local products” and (2) that the WTO “intrudes, limits and/or impairs” the constitutional powers of both Congress and the Supreme Court. Issue: Whether provisions of the Agreement Establishing the World Trade Organization unduly limit, restrict and impair Philippine sovereignty specifically the legislative power which, under Sec. 2, Article VI, 1987 Philippine Constitution is ‘vested in the Congress of the Philippines. Held: No, the WTO agreement does not unduly limit, restrict, and impair the Philippine sovereignty, particularly the legislative power granted by the Philippine Constitution. The Senate was acting in the proper manner when it concurred with the President’s ratification of the agreement. While sovereignty has traditionally been deemed absolute and all-encompassing on the domestic level, it is however subject to restrictions and limitations voluntarily agreed to by the Philippines, expressly or impliedly, as a member of the family of nations. Unquestionably, the Constitution did not envision a hermit-type isolation of

the country from the rest of the world. In its Declaration of Principles and State Policies, the Constitution “adopts the generally accepted principles of international law as part of the law of the land, and adheres to the policy of peace, equality, justice, freedom, cooperation and amity, with all nations.” By the doctrine of incorporation, the country is bound by generally accepted principles of international law, which are considered to be automatically part of our own laws. One of the oldest and most fundamental rules in international law is pacta sunt servanda — international agreements must be performed in good faith. “A treaty engagement is not a mere moral obligation but creates a legally binding obligation on the parties x x x. A state which has contracted valid international obligations is bound to make in its legislations such modifications as may be necessary to ensure the fulfillment of the obligations undertaken.” By their inherent nature, treaties really limit or restrict the absoluteness of sovereignty. By their voluntary act, nations may surrender some aspects of their state power in exchange for greater benefits granted by or derived from a convention or pact. After all, states, like individuals, live with coequals, and in pursuit of mutually covenanted objectives and benefits, they also commonly agree to limit the exercise of their otherwise absolute rights. Thus, treaties have been used to record agreements between States concerning such widely diverse matters as, for example, the lease of naval bases, the sale or cession of territory, the termination of war, the regulation of conduct of hostilities, the formation of alliances, the regulation of commercial relations, the settling of claims, the laying down of rules governing conduct in peace and the establishment of international organizations. The sovereignty of a state therefore cannot in fact and in reality be considered absolute. Certain restrictions enter into the picture: (1) limitations imposed by the very nature of membership in the family of nations and (2) limitations imposed by treaty stipulations. As aptly put by John F. Kennedy, “Today, no nation can build its destiny alone. The age of selfsufficient nationalism is over. The age of interdependence is here.” The WTO reliance on “most favored nation,” “national treatment,” and “trade without discrimination” cannot be struck down as unconstitutional as in fact they are rules of equality and reciprocity that apply to all WTO members. Aside from envisioning a trade policy based on “equality and reciprocity,” the fundamental law encourages industries that are “competitive in both domestic and foreign markets,” thereby demonstrating a clear policy against a sheltered domestic trade environment, but one in favor of the gradual development of robust industries that can compete with the best in the foreign markets. Indeed, Filipino managers and Filipino enterprises have shown capability and tenacity to compete internationally. And given a free trade environment, Filipino entrepreneurs and managers in Hongkong have demonstrated the Filipino capacity to grow and to prosper against the best offered under a policy of laissez faire.

WHEREFORE, the petition is DISMISSED for lack of merit.

[G.R. No. 132451. December 17, 1999]

CONGRESSMAN ENRIQUE T. GARCIA, petitioner, vs. HON. RENATO C. CORONA, in his capacity as the Executive Secretary, HON. FRANCISCO VIRAY, in his capacity as the Secretary of Energy, CALTEX PHILIPPINES INC., PILIPINAS SHELL PETROLEUM CORP. and PETRON CORP., respondents. DECISION YNARES-SANTIAGO, J.:

On November 5, 1997, this Court in Tatad v. Secretary of the Department of Energy and Lagman, et al., v. Hon. Ruben Torres, et al.,[1] declared Republic Act No. 8180, entitled An Act Deregulating the Downstream Oil Industry and For Other Purposes, unconstitutional, and its implementing Executive Order No. 392 void. R.A. 8180 was struck down as invalid because three key provisions intended to promote free competition were shown to achieve the opposite result. More specifically, this Court ruled that its provisions on tariff differential, stocking of inventories, and predatory pricing inhibit fair competition, encourage monopolistic power, and interfere with the free interaction of the market forces. While R.A. 8180 contained a separability clause, it was declared unconstitutional in its entirety since the three (3) offending provisions so permeated the law that they were so intimately the esse of the law. Thus, the whole statute had to be invalidated. As a result of the Tatad decision, Congress enacted Republic Act No. 8479, a new deregulation law without the offending provisions of the earlier law. Petitioner Enrique T. Garcia, a member of Congress, has now brought this petition seeking to declare Section 19 thereof, which sets the time of full deregulation, unconstitutional. After failing in his attempts to have Congress incorporate in the law the economic theory he espouses, petitioner now asks us, in the name of upholding the Constitution, to undo a violation which he claims Congress has committed. The assailed Section 19 of R.A. 8479 states in full:

SEC. 19. Start of Full Deregulation. --- Full deregulation of the Industry shall start five (5) months following the effectivity of this Act: Provided, however, That when the public interest so requires, the President may accelerate the start of full deregulation upon the recommendation of the DOE and the Department of Finance (DOF) when the prices of crude oil and petroleum products in the world market are declining and the value of the peso in relation to the US dollar is stable, taking into account relevant trends and prospects; Provided, further, That the foregoing provision notwithstanding, the five (5)-month Transition Phase shall continue to apply to LPG,

regular gasoline and kerosene as socially-sensitive petroleum products and said petroleum products shall be covered by the automatic pricing mechanism during the said period. Upon the implementation of full deregulation as provided herein, the Transition Phase is deemed terminated and the following laws are repealed: a) Republic Act No. 6173, as amended; b) Section 5 of Executive Order No. 172, as amended; c) Letter of Instruction No. 1431, dated October 15, 1984; d) Letter of Instruction No. 1441, dated November 20, 1984, as amended; e) Letter of Instruction No. 1460, dated May 9, 1985; f) Presidential Decree No. 1889; and g) Presidential Decree No. 1956, as amended by Executive Order No. 137: Provided, however, That in case full deregulation is started by the President in the exercise of the authority provided in this Section, the foregoing laws shall continue to be in force and effect with respect to LPG, regular gasoline and kerosene for the rest of the five (5)-month period. Petitioner contends that Section 19 of R.A. 8479, which prescribes the period for the removal of price control on gasoline and other finished products and for the full deregulation of the local downstream oil industry, is patently contrary to public interest and therefore unconstitutional because within the short span of five months, the market is still dominated and controlled by an oligopoly of the three (3) private respondents, namely, Shell, Caltex and Petron. The objective of the petition is deceptively simple. It states that if the constitutional mandate against monopolies and combinations in restraint of trade[2] is to be obeyed, there should be indefinite and open-ended price controls on gasoline and other oil products for as long as necessary. This will allegedly prevent the Big 3 --- Shell, Caltex and Petron --- from price-fixing and overpricing. Petitioner calls the indefinite retention of price controls as partial deregulation. The grounds relied upon in the petition are: A.

SECTION 19 OF R.A. NO. 8479 WHICH PROVIDES FOR FULL DEREGULATION FIVE (5) MONTHS OR EARLIER FOLLOWING THE EFFECTIVITY OF THE LAW, IS GLARINGLY PRO-OLIGOPOLY, ANTI-

COMPETITION AND ANTI-PEOPLE, AND IS THEREFORE PATENTLY UNCONSTITUTIONAL FOR BEING IN GROSS AND CYNICAL CONTRAVENTION OF THE CONSTITUTIONAL POLICY AND COMMAND EMBODIED IN ARTCLE XII, SECTION 19 OF THE 1987 CONSTITUTION AGAINST MONOPOLIES AND COMBINATIONS IN RESTRAINT OF TRADE. B.

SAID SECTION 19 OF R.A. No. 8479 IS GLARINGLY PRO-OLIGOPOLY, ANTICOMPETITION AND ANTI-PEOPLE, FOR THE FURTHER REASON THAT IT PALPABLY AND CYNICALLY VIOLATES THE VERY OBJECTIVE AND PURPOSE OF R.A. NO. 8479, WHICH IS TO ENSURE A TRULY COMPETITIVE MARKET UNDER A REGIME OF FAIR PRICES. C.

SAID SECTION 19 OF R.A. No. 8479, BEING GLARINGLY PRO-OLIGOPOLY, ANTI-COMPETITION AND ANTI-PEOPLE, BEING PATENTLY UNCONSTITUTIONAL AND BEING PALPABLY VIOLATIVE OF THE LAWS POLICY AND PURPOSE OF ENSURING A TRULY COMPETITIVE MARKET UNDER A REGIME OF FAIR PRICES, IS A VERY GRAVE AND GRIEVOUS ABUSE OF DISCRETION ON THE PART OF THE LEGISLATIVE AND EXECUTIVE BRANCHES OF GOVERNMENT. D.

PREMATURE FULL DEREGULATION UNDER SECTION 19 OF R.A. NO. 8479 MAY AND SHOULD THEREFORE BE DECLARED NULL AND VOID EVEN AS THE REST OF ITS PROVISIONS REMAIN IN FORCE, SUCH AS THE TRANSITION PHASE OR PARTIAL DEREGULATION WITH PRICE CONTROLS THAT ENSURES THE PROTECTION OF THE PUBLIC INTEREST BY PREVENTING THE BIG 3 OLIGOPOLYS PRICE-FIXING AND OVERPRICING.[3] The issues involved in the deregulation of the downstream oil industry are of paramount significance. The ramifications, international and local in scope, are complex. The impact on the nations economy is pervasive and far-reaching. The amounts involved in the oil business are immense. Fluctuations in the supply and price of oil products have a dramatic effect on economic development and public welfare.As pointed out in the Tatad decision, few cases carry a surpassing importance on the daily life of every Filipino. The issues affect everybody from the poorest wage-earners and their families to the richest entrepreneurs, from industrial giants to humble consumers.

Our decision in this case is complicated by the unstable oil prices in the world market. Even as this case is pending, the price of OPEC oil is escalating to record levels. We have to emphasize that our decision has nothing to do with worldwide fluctuations in oil prices and the counter-measures of Government each time a new development takes place. The most important part of deregulation is freedom from price control. Indeed, the free play of market forces through deregulation and when to implement it represent one option to solve the problems of the oil-consuming public. There are other considerations which may be taken into account such as the reduction of taxes on oil products, the reinstitution of an Oil Price Stabilization Fund, the choice between government subsidies taken from the regular taxpaying public on one hand and the increased costs being shouldered only by users of oil products on the other, and most important, the immediate repeal of the oil deregulation law as wrong policy. Petitioner wants the setting of prices to be done by Government instead of being determined by free market forces. His preference is continued price control with no fixed end in sight. A simple glance at the factors surrounding the present problems besetting the oil industry shows that they are economic in nature. R.A. 8479, the present deregulation law, was enacted to implement Article XII, Section 19 of the Constitution which provides:

The State shall regulate or prohibit monopolies when the public interest so requires. No combinations in restraint of trade or unfair competition shall be allowed. This is so because the Government believes that deregulation will eventually prevent monopoly. The simplest form of monopoly exists when there is only one seller or producer of a product or service for which there are no substitutes. In its more complex form, monopoly is defined as the joint acquisition or maintenance by members of a conspiracy, formed for that purpose, of the power to control and dominate trade and commerce in a commodity to such an extent that they are able, as a group, to exclude actual or potential competitors from the field, accompanied with the intention and purpose to exercise such power.[4] Where two or three or a few companies act in concert to control market prices and resultant profits, the monopoly is called an oligopoly or cartel. It is a combination in restraint of trade. The perennial shortage of oil supply in the Philippines is exacerbated by the further fact that the importation, refining, and marketing of this precious commodity are in the hands of a cartel, local but made up of foreign-owned corporations. Before the start of deregulation, the three private respondents controlled the entire oil industry in the Philippines. It bears reiterating at the outset that the deregulation of the oil industry is a policy determination of the highest order. It is unquestionably a priority program of Government. The Department of Energy Act of 1992[5] expressly mandates that the development and updating of the existing Philippine energy program shall include a policy direction towards deregulation of the power and energy industry. Be that as it may, we are not concerned with whether or not there should be deregulation. This is outside our jurisdiction. The judgment on the issue is a settled matter and only Congress can reverse it.Rather, the question that we should address here is --- are the method and the manner chosen by Government to accomplish its cherished goal offensive to the

Constitution? Is indefinite price control in the manner proposed by petitioner the only feasible and legal way to achieve it? Petitioner has taken upon himself a most challenging task. Unquestionably, the direction towards which the nations efforts at economic and social upliftment should be addressed is a function of Congress and the President. In the exercise of this function, Congress and the President have obviously determined that speedy deregulation is the answer to the acknowledged dominion by oligopolistic forces of the oil industry. Thus, immediately after R.A. 8180 was declared unconstitutional in the Tatad case, Congress took resolute steps to fashion new legislation towards the objective of the earlier law. Invoking the Constitution, petitioner now wants to slow down the process. While the Court respects the firm resolve displayed by Congress and the President, all departments of Government are equally bound by the sovereign will expressed in the commands of the Constitution. There is a need for utmost care if this Court is to faithfully discharge its duties as arbitral guardian of the Constitution. We cannot encroach on the policy functions of the two other great departments of Government. But neither can we ignore any overstepping of constitutional limitations. Locating the correct balance between legality and policy, constitutional boundaries and freedom of action, and validity and expedition is this Courts dilemma as it resolves the legitimacy of a Government program aimed at giving every Filipino a more secure, fulfilling and abundant life. Our ruling in Tatad is categorical that the Constitutions Article XII, Section 19, is anti-trust in history and spirit. It espouses competition. We have stated that only competition which is fair can release the creative forces of the market. We ruled that the principle which underlies the constitutional provision is competition. Thus:

Section 19, Article XII of our Constitution is anti-trust in history and in spirit. It espouses competition. The desirability of competition is the reason for the prohibition against restraint of trade, the reason for the interdiction of unfair competition, and the reason for regulation of unmitigated monopolies. Competition is thus the underlying principle of section 19, Article XII of our Constitution which cannot be violated by R.A. No. 8180. We subscribe to the observation of Prof. Gellhorn that the objective of anti-trust law is to assure a competitive economy, based upon the belief that through competition producers will strive to satisfy consumer wants at the lowest price with the sacrifice of the fewest resources. Competition among producers allows consumers to bid for goods and services, and thus matches their desires with societys opportunity costs. He adds with appropriateness that there is a reliance upon the operation of the market system (free enterprise) to decide what shall be produced, how resources shall be allocated in the production process, and to whom the various products will be distributed. The market system relies on the consumer to decide what and how much shall be produced, and on competition, among producers to determine who will manufacture it.[6] In his recital of the antecedent circumstances, petitioner repeats in abbreviated form the factual findings and conclusions which led the Court to declare R.A. 8180 unconstitutional. The

foreign oligopoly or cartel formed by respondents Shell, Caltex and Petron, their indulging in price-fixing and overpricing, their blockade tactics which effectively obstructed the entry of genuine competitors, the dangers posed by the oil cartel to national security and economic development, and other prevailing sentiments are stated as axiomatic truths. They are repeated in capsulized context as the current background facts of the present petition. The empirical existence of this deplorable situation was precisely the reason why Congress enacted the oil deregulation law. The evils arising from conspiratorial acts of monopoly are recognized as clear and present. But the enumeration of the evils by our Tatad decision was not for the purpose of justifying continued government control, especially price control. The objective was, rather, the opposite. The evils were emphasized to show the need for free competition in a deregulated industry. And to be sure, the measures to address these evils are for Congress to determine, but they have to meet the test of constitutional validity. The Court respects the legislative finding that deregulation is the policy answer to the problems. It bears stressing that R.A. 8180 was declared invalid not because deregulation is unconstitutional. The law was struck down because, as crafted, three key provisions plainly encouraged the continued existence if not the proliferation of the constitutionally proscribed evils of monopoly and restraint of trade. In sharp contrast, the present petition lacks a factual foundation specifically highlighting the need to declare the challenged provision unconstitutional. There is a dearth of relevant, reliable, and substantial evidence to support petitioners theory that price control must continue even as Government is trying its best to get out of regulating the oil industry. The facts of the petition are, in the main, a general dissertation on the evils of monopoly. Petitioner overlooks the fact that Congress enacted the deregulation law exactly because of the monopoly evils he mentions in his petition. Congress instituted the lifting of price controls in the belief that free and fair competition was the best remedy against monopoly power. In other words, petitioners facts are also the reasons why Congress lifted price controls and why the President accelerated the process.The facts adduced in favor of continued and indefinite price control are the same facts which supported what Congress believes is an exercise of wisdom and discretion when it chose the path of speedy deregulation and rejected Congressman Garcias economic theory. The petition states that it is using the very thoughts and words of the Court in its Tatad decision. Those thoughts and words, however, were directed against the tariff differential, the inventory requirement, and predatory pricing, not against deregulation as a policy and not against the lifting of price controls. A dramatic, at times expansive and grandiloquent, reiteration of the same background circumstances narrated in Tatad does not squarely sustain petitioners novel thesis that there can be deregulation without lifting price controls. Petitioner may call the industry subject to price controls as deregulated. In enacting the challenged provision, Congress, on the other hand, has declared that any industry whose prices and profits are fixed by government authority remains a highly regulated one. Petitioner, therefore, engages in a legal paradox. He fails to show how there can be deregulation while retaining government price control. Deregulation means the lifting of control,

governance and direction through rule or regulation. It means that the regulated industry is freed from the controls, guidance, and restrictions to which it used to be subjected. The use of the word partial to qualify deregulation is sugar-coating. Petitioner is really against deregulation at this time. Petitioner states that price control is good. He claims that it was the regulation of the importation of finished oil products which led to the exit of competitors and the consolidation and dominion of the market by an oligopoly, not price control. Congress and the President think otherwise. The argument that price control is not the villain in the intrusion and growth of monopoly appears to be pure theory not validated by experience. There can be no denying the fact that the evils mentioned in the petition arose while there was price control. The dominance of the socalled Big 3 became entrenched during the regime of price control. More importantly, the ascertainment of the cause and the method of dismantling the oligopoly thus created are a matter of legislative and executive choice. The judicial process is equipped to handle legality but not wisdom of choice and the efficacy of solutions. Petitioner engages in another contradiction when he puts forward what he calls a selfevident truth. He states that a truly competitive market and fair prices cannot be legislated into existence. However, the truly competitive market is not being created or fashioned by the challenged legislation. The market is simply freed from legislative controls and allowed to grow and develop free from government interference. R.A. 8479 actually allows the free play of supply and demand to dictate prices. Petitioner wants a government official or board to continue performing this task. Indefinite and open-ended price control as advocated by petitioner would be to continue a regime of legislated regulation where free competition cannot possibly flourish. Control is the antithesis of competition. To grant the petition would mean that the Government is not keen on allowing a free market to develop. Petitioners self-evident truth thus supports the validity of the provision of law he opposes. New players in the oil industry intervened in this case. According to them, it is the free market policy and atmosphere of deregulation which attracted and brought the new participants, themselves included, into the market. The intervenors express their fear that this Court would overrule legislative policy and replace it with petitioners own legislative program. The factual allegations of the intervenors have not been refuted and we see no reason to doubt them. Their argument that the co-existence of many viable rivals create free market conditions induces competition in product quality and performance and makes available to consumers an expanded range of choices cannot be seriously disputed. On the other hand, the pleadings of public and private respondents both put forth the argument that the challenged provision is a policy decision of Congress and that the wisdom of the provision is outside the authority of this Court to consider. We agree. As we have ruled in Morfe v. Mutuc[7]:

(I)t is well to remember that this Court, in the language of Justice Laurel, does not pass upon question or wisdom, justice or expediency of legislation. As expressed by Justice Tuason: It is not the province of the courts to supervise legislation and keep it within the bounds of propriety and common sense. That is primarily and exclusively a

legislative concern. There can be no possible objection then to the observation of Justice Montemayor: As long as laws do not violate any Constitutional provision, the Courts merely interpret and apply them regardless of whether or not they are wise or salutary. For they, according to Justice Labrador, are not supposed to override legitimate policy and x x x never inquire into the wisdom of the law. It is thus settled, to paraphrase Chief Justice Concepcion in Gonzales v. Commission on Elections, that only congressional power or competence, not the wisdom of the action taken, may be the basis for declaring a statute invalid. This is as it ought to be. The principle of separation of powers has in the main wisely allocated the respective authority of each department and confined its jurisdiction to such a sphere. There would then be intrusion not allowable under the Constitution if on a matter left to the discretion of a coordinate branch, the judiciary would substitute its own. If there be adherence to the rule of law, as there ought to be, the last offender should be the courts of justice, to which rightly litigants submit their controversy precisely to maintain unimpaired the supremacy of legal norms and prescriptions.The attack on the validity of the challenged provision likewise insofar as there may be objections, even if valid and cogent, on its wisdom cannot be sustained. In this petition, Congressman Garcia seeks to revive the long settled issue of the timeliness of full deregulation, which issue he had earlier submitted to this Court by way of a Partial Motion for Reconsideration in the Tatad case. In our Resolution dated December 3, 1997, which has long become final and executory, we stated:

We shall first resolve petitioner Garcias linchpin contention that the full deregulation decreed by R.A. No. 8180 to start at the end of March 1997 is unconstitutional. For prescinding from this premise, petitioner suggests that we simply go back to the transition period, price control will be revived through the automatic pricing mechanism based on Singapore Posted Prices. The Energy Regulatory Board x xx would play a limited and ministerial role of computing the monthly price ceiling of each and every petroleum fuel product, using the automatic pricing formula. While the OPSF would return, this coverage would be limited to monthly price increases in excess of P0.50 per liter. We are not impressed by petitioner Garcias submission. Petitioner has no basis in condemning as unconstitutional per se the date fixed by Congress for the beginning of the full deregulation of the downstream oil industry. Our Decision merely faulted the Executive for factoring the depletion of OPSF in advancing the date of full deregulation to February 1997. Nonetheless, the error of the Executive is now a nonissue for the full deregulation set by Congress itself at the end of March 1997 has already come to pass. March 1997 is not an arbitrary date. By that date, the transition period has ended and it was expected that the people would have adjusted to the role

of market forces in shaping the prices of petroleum and its products. The choice of March 1997 as the date of full deregulation is a judgment of Congress and its judgment call cannot be impugned by this Court.[8] Reduced to its basic arguments, it can be seen that the challenge in this petition is not against the legality of deregulation. Petitioner does not expressly challenge deregulation. The issue, quite simply, is the timeliness or the wisdom of the date when full deregulation should be effective. In this regard, what constitutes reasonable time is not for judicial determination. Reasonable time involves the appraisal of a great variety of relevant conditions, political, social and economic. They are not within the appropriate range of evidence in a court of justice. It would be an extravagant extension of judicial authority to assert judicial notice as the basis for the determination.[9] We repeat that what petitioner decries as unsuccessful is not a final result. It is only a beginning. The Court is not inclined to stifle deregulation as enacted by Congress from its very start. We leave alone the program of deregulation at this stage. Reasonable time will prove the wisdom or folly of the deregulation program for which Congress and not the Court is accountable. Petitioner argues further that the public interest requires price controls while the oligopoly exists, for that is the only way the public can be protected from monopoly or oligopoly pricing. But is indefinite price control the only feasible and legal way to enforce the constitutional mandate against oligopolies? Article 186 of the Revised Penal Code, as amended, punishes as a felony the creation of monopolies and combinations in restraint of trade. The Solicitor General, on the other hand, cites provisions of R.A. 8479 intended to prevent competition from being corrupted or manipulated. Section 11, entitled Anti-Trust Safeguards, defines and prohibits cartelization and predatory pricing. It penalizes the persons and officers involved with imprisonment of three (3) to seven (7) years and fines ranging from One million to Two million pesos. For this purpose, a Joint Task Force from the Department of Energy and Department of Justice is created under Section 14 to investigate and order the prosecution of violations. Sections 8 and 9 of the Act, meanwhile, direct the Departments of Foreign Affairs, Trade and Industry, and Energy to undertake strategies, incentives and benefits, including international information campaigns, tax holidays and various other agreements and utilizations, to invite and encourage the entry of new participants. Section 6 provides for uniform tariffs at three percent (3%). Section 13 of the Act provides for Remedies, under which the filing of actions by government prosecutors and the investigation of private complaints by the Task Force is provided. Sections 14 and 15 provide how the Department of Energy shall monitor and prevent the occurrence of collusive pricing in the industry. It can be seen, therefore, that instead of the price controls advocated by the petitioner, Congress has enacted anti-trust measures which it believes will promote free and fair competition. Upon the other hand, the disciplined, determined, consistent and faithful execution

of the law is the function of the President. As stated by public respondents, the remedy against unreasonable price increases is not the nullification of Section 19 of R.A. 8479 but the setting into motion of its various other provisions. For this Court to declare unconstitutional the key provision around which the laws anti-trust measures are clustered would mean a constitutionally interdicted distrust of the wisdom of Congress and of the determined exercise of executive power. Having decided that deregulation is the policy to follow, Congress and the President have the duty to set up the proper and effective machinery to ensure that it works. This is something which cannot be adjudicated into existence. This Court is only an umpire of last resort whenever the Constitution or a law appears to have been violated. There is no showing of a constitutional violation in this case. WHEREFORE, the petition is DISMISSED.

PROVINCE OF NORTH COTABATO VS GOVERNMENT OF THE REPUBLIC OF THE PHILIPPINES Posted by kaye lee on 9:43 PM

G.R. No. 183591 October 14 2008 Province of North Cotabato vs Government of the Republic of the Philippines

FACTS: On August 5, 2008, the Government of the Republic of the Philippines and the Moro Islamic Liberation Front (MILF) were scheduled to sign a Memorandum of Agreement of the Ancestral Domain Aspect of the GRP - MILF Tripoli Agreement on Peace of 2001 in Kuala Lumpur, Malaysia. Invoking the right to information on matters of public concern, the petitioners seek to compel respondents to disclose and furnish them the complete and official copies of the MA-AD and to prohibit the slated signing of the MOA-AD and the holding of public consultation thereon. They also pray that the MOA-AD be declared unconstitutional. The Court issued a TRO enjoining the GRP from signing the same.

ISSUES: 1. Whether or not the constitutionality and the legality of the MOA is ripe for adjudication; 2. Whether or not there is a violation of the people's right to information on matters of public concern (Art 3 Sec. 7) under a state policy of full disclosure of all its transactions involving public interest (Art 2, Sec 28) including public consultation under RA 7160 (Local Government Code of 1991) 3. Whether or not the signing of the MOA, the Government of the Republic of the Philippines would be binding itself a) to create and recognize the Bangsamoro Juridical Entity (BJE) as a separate state, or a juridical, territorial or political subdivision not recognized by law; b) to revise or amend the Constitution and existing laws to conform to the MOA; c) to concede to or recognize the claim of the Moro Islamic Liberation Front for ancestral domain in violation of Republic Act No. 8371 (THE INDIGENOUS PEOPLES RIGHTS ACT OF 1997), particularly Section 3(g) & Chapter VII (DELINEATION, RECOGNITION OF ANCESTRAL DOMAINS)

RULINGS:

1. Yes, the petitions are ripe for adjudication. The failure of the respondents to consult the local government units or communities affected constitutes a departure by respondents from their mandate under EO No. 3. Moreover, the respondents exceeded their authority by the mere act of guaranteeing amendments to the Constitution. Any alleged violation of the Constitution by any branch of government is a proper matter for judicial review. As the petitions involve constitutional issues which are of paramount public interest or of transcendental importance, the Court grants the petitioners, petitioners-in-intervention and intervening respondents the requisite locus standi in keeping with the liberal stance adopted in David v. Macapagal- Arroyo. In Pimentel, Jr. v. Aguirre, this Court held: x x x [B]y the mere enactment of the questioned law or the approval of the challenged action, the dispute is said to have ripened into a judicial controversy even without any other overt act . Indeed, even a singular violation of the Constitution and/or the law is enough to awaken judicial duty.x x x x By the same token, when an act of the President, who in our constitutional scheme is a coequal of Congress, is seriously alleged to have infringed the Constitution and the laws x x x settling the dispute becomes the duty and the responsibility of the courts. That the law or act in question is not yet effective does not negate ripeness.

2. Yes. The Court finds that there is a grave violation of the Constitution involved in the matters of public concern (Sec 7 Art III) under a state policy of full disclosure of all its transactions involving public interest (Art 2, Sec 28) including public consultation under RA 7160 (Local Government Code of 1991). (Sec 7 ArtIII) The right to information guarantees the right of the people to demand information, while Sec 28 recognizes the duty of officialdom to give information even if nobody demands. The complete and effective exercise of the right to information necessitates that its complementary provision on public disclosure derive the same selfexecutory nature, subject only to reasonable safeguards or limitations as may be provided by law. The contents of the MOA-AD is a matter of paramount public concern involving public interest in the highest order. In declaring that the right to information contemplates steps and negotiations leading to the consummation of the contract, jurisprudence finds no distinction as to the executory nature or commercial character of the agreement. E.O. No. 3 itself is replete with mechanics for continuing consultations on both national and local levels and for a principal forum for consensus-building. In fact, it is the duty of the Presidential Adviser on the Peace Process to conduct regular dialogues to seek relevant information, comments, advice, and recommendations from peace partners and concerned sectors of society.

3. a) to create and recognize the Bangsamoro Juridical Entity (BJE) as a separate state, or a juridical, territorial or political subdivision not recognized by law;

Yes. The provisions of the MOA indicate, among other things, that the Parties aimed to vest in the BJE the status of an associated state or, at any rate, a status closely approximating it.

The concept of association is not recognized under the present Constitution.

No province, city, or municipality, not even the ARMM, is recognized under our laws as having an “associative” relationship with the national government. Indeed, the concept implies powers that go beyond anything ever granted by the Constitution to any local or regional government. It also implies the recognition of the associated entity as a state. The Constitution, however, does not contemplate any state in this jurisdiction other than the Philippine State, much less does it provide for a transitory status that aims to prepare any part of Philippine territory for independence.

The BJE is a far more powerful entity than the autonomous region recognized in the Constitution. It is not merely an expanded version of the ARMM, the status of its relationship with the national government being fundamentally different from that of the ARMM. Indeed, BJE is a state in all but name as it meets the criteria of a state laid down in the Montevideo Convention, namely, a permanent population, a defined territory, a government, and a capacity to enter into relations with other states.

Even assuming arguendo that the MOA-AD would not necessarily sever any portion of Philippine territory, the spirit animating it – which has betrayed itself by its use of the concept of association – runs counter to the national sovereignty and territorial integrity of the Republic.

The defining concept underlying the relationship between the national government and the BJE being itself contrary to the present Constitution, it is not surprising that many of the specific provisions of the MOA-AD on the formation and powers of the BJE are in conflict with the Constitution and the laws. The BJE is more of a state than an autonomous region. But even assuming that it is covered by the term “autonomous region” in the constitutional provision just quoted, the MOA-AD would still be in conflict with it.

b) to revise or amend the Constitution and existing laws to conform to the MOA:

The MOA-AD provides that “any provisions of the MOA-AD requiring amendments to the existing legal framework shall come into force upon the signing of a Comprehensive Compact and upon effecting the necessary changes to the legal framework,” implying an amendment of the Constitution to accommodate the MOAAD. This stipulation, in effect, guaranteed to the MILF the amendment of the Constitution .

It will be observed that the President has authority, as stated in her oath of office, only to preserve and defend the Constitution. Such presidential power does not, however, extend to allowing her to change the Constitution, but simply to recommend proposed amendments or revision. As long as she limits herself to recommending these

changes and submits to the proper procedure for constitutional amendments and revision, her mere recommendation need not be construed as an unconstitutional act.

The “suspensive clause” in the MOA-AD viewed in light of the above-discussed standards.

Given the limited nature of the President’s authority to propose constitutional amendments, she cannot guarantee to any third party that the required amendments will eventually be put in place, nor even be submitted to a plebiscite. The most she could do is submit these proposals as recommendations either to Congress or the people, in whom constituent powers are vested.

c) to concede to or recognize the claim of the Moro Islamic Liberation Front for ancestral domain in violation of Republic Act No. 8371 (THE INDIGENOUS PEOPLES RIGHTS ACT OF 1997), particularly Section 3(g) & Chapter VII (DELINEATION, RECOGNITION OF ANCESTRAL DOMAINS) This strand begins with the statement that it is “the birthright of all Moros and all Indigenous peoples of Mindanao to identify themselves and be accepted as ‘Bangsamoros.’” It defines “Bangsamoro people” as the natives or original inhabitants of Mindanao and its adjacent islands including Palawan and the Sulu archipelago at the time of conquest or colonization, and their descendants whether mixed or of full blood, including their spouses.

Thus, the concept of “Bangsamoro,” as defined in this strand of the MOA-AD, includes not only “Moros” as traditionally understood even by Muslims, but all indigenous peoples of Mindanao and its adjacent islands. The MOA-AD adds that the freedom of choice of indigenous peoples shall be respected. What this freedom of choice consists in has not been specifically defined. The MOA-AD proceeds to refer to the “Bangsamoro homeland,” the ownership of which is vested exclusively in the Bangsamoro people by virtue of their prior rights of occupation. Both parties to the MOA-AD acknowledge that ancestral domain does not form part of the public domain.

Republic Act No. 8371 or the Indigenous Peoples Rights Act of 1997 provides for clear-cut procedure for the recognition and delineation of ancestral domain, which entails, among other things, the observance of the free and prior informed consent of the Indigenous Cultural Communities/Indigenous Peoples. Notably, the statute does not grant the Executive Department or any government agency the power to delineate and recognize an ancestral domain claim by mere agreement or compromise.

Two, Republic Act No. 7160 or the Local Government Code of 1991 requires all national offices to conduct consultations beforeany project or program critical to the environment and human ecology including those that may call for the eviction of a particular group of people residing in such locality, is implemented therein. The MOA-AD is one peculiar program that unequivocally and unilaterally vests ownership of a vast territory to the Bangsamoro

people, which could pervasively and drastically result to the diaspora or displacement of a great number of inhabitants from their total environment.

CONCLUSION: In sum, the Presidential Adviser on the Peace Process committed grave abuse of discretion when he failed to carry out the pertinent consultation process, as mandated by E.O. No. 3, Republic Act No. 7160, and Republic Act No. 8371. The furtive process by which the MOA-AD was designed and crafted runs contrary to and in excess of the legal authority, and amounts to a whimsical, capricious, oppressive, arbitrary and despotic exercise thereof. It illustrates a gross evasion of positive duty and a virtual refusal to perform the duty enjoined.

The MOA-AD cannot be reconciled with the present Constitution and laws. Not only its specific provisions but the very concept underlying them, namely, the associative relationship envisioned between the GRP and the BJE, are unconstitutional, for the concept presupposes that the associated entity is a state and implies that the same is on its way to independence.

Case Digest: GR No. 183591 2/4/2015 4 Comments

Province of North Cotabato, Province of Zamboanga Del Norte, City of Iligan, City of Zamboanga, petitioners in intervention Province of Sultan Kudarat, City of Isabela and Municipality of Linnamon, Intervenors Franklin Drilon and Adel Tamano and Sec. Mar Roxas -vsErmita Exec.Sec., Romulo Sec DFA, Andaya Sec DBM, Ventura Administrator National Mapping & Resource Information Authority and Davide Jr. and respondents in intervention Muslim Multi-Sectoral Movement for Peace and Development and Muslim Legal Assistance Foundation Inc., Facts: Subject of this case is the Memorandum of Agreement on the Ancestral Domain (MOA-AD) which is scheduled to be signed by the Government of the Republic of the Philippines and the MILF in August 05, 2008. Five cases bearing the same subject matter were consolidated by this court namely:-



  

GR 183591 by the Province of Cotabato and Vice Governor Pinol on its prayer to declare unconstitutional and to have the MOA-AD disclosed to the public and be open for public consultation. GR 183752 by the City of Zamboanga et al on its prayer to declare null and void said MOA-AD and to exclude the city to the BJE. GR 183893 by the City of Iligan enjoining the respondents from signing the MOA-AD and additionally impleading Exec. Sec. Ermita. GR 183951 by the Province of Zamboanga del Norte et al, praying to declare null and void the MOA-AD and without operative effect and those respondents enjoined from executing the MOA-AD.



GR 183692 by Maceda, Binay and Pimentel III, praying for a judgment prohibiting and permanently enjoining respondents from formally signing and executing the MOA-AD and or any other agreement derived therefrom or similar thereto, and nullifying the MOA-AD for being unconstitutional and illegal and impleading Iqbal.

The MOA-AD is a result of various agreements entered into by and between the government and the MILF starting in 1996; then in 1997, they signed the Agreement on General Cessation of Hostilities; and the following year, they signed the General Framework of Agreement of Intent on August 27, 1998. However, in 1999 and in the early of 2000, the MILF attacked a number of municipalities in Central Mindanao. In March 2000, they took the hall of Kauswagan, Lanao del Norte; hence, then Pres. Estrada declared an all-out warwhich tolled the peace negotiation. It was when then Pres. Arroyo assumed office, when the negotiation regarding peace in Mindanao continued. MILF was hesitant; however, this negotiation proceeded when the government of Malaysia interceded. Formal peace talks resumed and MILF suspended all its military actions. The Tripoli Agreement in 2001 lead to the ceasefire between the parties. After the death of MILF Chairman Hashim and Iqbal took over his position, the crafting of MOA-AD in its final form was born.



MOA-AD Overview

This is an agreement to be signed by the GRP and the MILF. Used as reference in the birth of this MOA-AD are the Tripoli Agreement, organic act of ARMM, IPRA Law, international laws such as ILO Convention 169, the UN Charter etc., and the principle of Islam i.e compact right entrenchment (law of compact, treaty and order). The body is divided into concepts and principles, territory, resources, and governance. Embodied in concepts and principles, is the definition of Bangsamoro as all indigenous peoples of Mindanao and its adjacent islands. These people have the right to self- governance of their Bangsamoro homeland to which they have exclusive ownership by virtue of their prior rights of occupation in the land. The MOA-AD goes on to describe the Bangsamoro people as "the ‘First Nation' with

defined territory and with a system of government having entered into treaties of amity and commerce with foreign nations." It then mentions for the first time the "Bangsamoro Juridical Entity" (BJE) to which it grants the authority and jurisdiction over the Ancestral Domain and Ancestral Lands of the Bangsamoro. As defined in the territory of the MOA-AD, the BJE shall embrace the MindanaoSulu-Palawan geographic region, involving the present ARMM, parts of which are those which voted in the inclusion to ARMM in a plebiscite. The territory is divided into two categories, “A” which will be subject to plebiscite not later than 12 mos. after the signing and “B” which will be subject to plebiscite 25 years from the signing of another separate agreement. Embodied in the MOA-AD that the BJE shall have jurisdiction over the internal waters-15kms from the coastline of the BJE territory; they shall also have "territorial waters," which shall stretch beyond the BJE internal waters up to the baselines of the Republic of the Philippines (RP) south east and south west of mainland Mindanao; and that within these territorial waters, the BJE and the government shall exercise joint jurisdiction, authority and management over all natural resources. There will also be sharing of minerals in the territorial waters; but no provision on the internal waters. Included in the resources is the stipulation that the BJE is free to enter into any economic cooperation and trade relations with foreign countries and shall have the option to establish trade missions in those countries, as well as environmental cooperation agreements, but not to include aggression in the GRP. The external defense of the BJE is to remain the duty and obligation of the government. The BJE shall have participation in international meetings and events" like those of the ASEAN and the specialized agencies of the UN. They are to be entitled to participate in Philippine official missions and delegations for the negotiation of border agreements or protocols for environmental protection and equitable sharing of incomes and revenues involving the bodies of water adjacent to or between the islands forming part of the ancestral domain. The BJE shall also have the right to explore its resources and that the sharing between the Central Government and the BJE of total production pertaining to natural resources is to be 75:25 in favor of the BJE. And they shall have the right to cancel or modify concessions and TLAs. And lastly in the governance, the MOA-AD claims that the relationship between the GRP and MILF is associative i.e. characterized by shared authority and responsibility. This structure of governance shall be further discussed in the

Comprehensive Compact, a stipulation which was highly contested before the court. The BJE shall also be given the right to build, develop and maintain its own institutions, the details of which shall be discussed in the comprehensive compact as well. Issues: 1. WON the petitions have complied with the procedural requirements for the exercise of judicial review 2. WON respondents violate constitutional and statutory provisions on public consultation and the right to information when they negotiated and later initialed the MOA-AD; and 3. WON the contents of the MOA-AD violated the Constitution and the laws Ruling: The SC declared the MOA-AD contrary to law and the Constitution.



On the Procedural Issue

1st issue: As regards the procedural issue, SC upheld that there is indeed a need for the exercise of judicial review. The power of judicial review is limited to actual cases or controversy, that is the court will decline on issues that are hypothetical, feigned problems or mere academic questions. Related to the requirement of an actual case or controversy is the requirement of ripeness. The contention of the SolGen is that there is no issue ripe for adjudication since the MOA-AD is only a proposal and does not automatically create legally demandable rights and obligations. Such was denied. The SC emphasized that the petitions are alleging acts made in violation of their duty or in grave abuse of discretion. Well-settled jurisprudence states that acts made by authority which exceed their authority, by violating their duties under E.O. No. 3 and the provisions of the Constitution and statutes, the petitions make a

prima facie case for Certiorari, Prohibition, and Mandamus, and an actual case or controversy ripe for adjudication exists. When an act of a branch of government is seriously alleged to have infringed the Constitution, it becomes not only the right but in fact the duty of the judiciary to settle the dispute. This is aside from the fact that concrete acts made under the MOA-AD are not necessary to render the present controversy ripe and that the law or act in question as not yet effective does not negate ripeness. With regards to the locus standi, the court upheld the personalities of the Province of Cotabato, Province of Zamboanga del norte, City of Iligan, City of Zamboanga, petitioners in intervention Province of Sultan Kudarat, City of Isabela and Municipality of Linnamon to have locus standi since it is their LGUs which will be affected in whole or in part if include within the BJE. Intervenors Franklin Drilon and Adel Tamano, in alleging their standing as taxpayers, assert that government funds would be expended for the conduct of an illegal and unconstitutional plebiscite to delineate the BJE territory. On that score alone, they can be given legal standing. Senator Mar Roxas is also given a standing as an intervenor. And lastly, the Intervening respondents Muslim Multi-Sectoral Movement for Peace and Development, an advocacy group for justice and the attainment of peace and prosperity in Muslim Mindanao; and Muslim Legal Assistance Foundation Inc., a non-government organization of Muslim lawyers since they stand to be benefited or prejudiced in the resolution of the petitions regarding the MOA-AD. On the contention of mootness of the issue considering the signing of the MOA-AD has already been suspended and that the President has already disbanded the GRP, the SC disagrees. The court reiterates that the moot and academic principle is a general rule only, the exceptions, provided in David v. Macapagal-Arroyo, that it will decide cases, otherwise moot and academic, if it finds that (a) there is a grave violation of the Constitution; (b) the situation is of exceptional character and paramount public interest is involved; (c) the constitutional issue raised requires formulation of controlling principles to guide the bench, the bar, and the public; and (d) the case is capable of repetition yet evading review; and that where there is a voluntary cessation of the activity complained of by the defendant or doer, it does not divest the court the power to hear and try the case especially when the plaintiff is seeking for damages or injunctive relief. Clearly, the suspension of the signing of the MOA-AD and the disbandment of the

GRP did not render the petitions moot and academic. The MOA-AD is subject to further legal enactments including possible Constitutional amendments more than ever provides impetus for the Court to formulate controlling principles to guide the bench, the bar, the public and, in this case, the government and its negotiating entity. At all events, the Court has jurisdiction over most if not the rest of the petitions. There is a reasonable expectation that petitioners will again be subjected to the same problem in the future as respondents' actions are capable of repetition, in another or any form. But with respect to the prayer of Mandamus to the signing of the MOA-AD, such has become moot and academic considering that parties have already complied thereat.



On the Substantive Issue

2nd Issue: The SC ruled that the MOA-AD is a matter of public concern, involving as it does the sovereignty and territorial integrity of the State, which directly affects the lives of the public at large. As enshrined in the Constitution, the right to information guarantees the right of the people to demand information, and integrated therein is the recognition of the duty of the officialdom to give information even if nobody demands. The policy of public disclosure establishes a concrete ethical principle for the conduct of public affairs in a genuinely open democracy, with the people's right to know as the centerpiece. It is a mandate of the State to be accountable by following such policy. These provisions are vital to the exercise of the freedom of expression and essential to hold public officials at all times accountable to the people. Also, it was held that such stipulation in the Constitution is self-executory with reasonable safeguards —the effectivity of which need not await the passing of a statute. Hence, it is essential to keep open a continuing dialogue or process of communication between the government and the people. It is in the interest of the State that the channels for free political discussion be maintained to the end that the government may perceive and be responsive to the people's will.

The idea of a feedback mechanism was also sought for since it is corollary to the twin rights to information and disclosure. And feedback means not only the conduct of the plebiscite as per the contention of the respondents. Clearly, what the law states is the right of the petitioners to be consulted in the peace agenda as corollary to the constitutional right to information and disclosure. As such, respondent Esperon committed grave abuse of discretion for failing to carry out the furtive process by which the MOA-AD was designed and crafted runs contrary to and in excess of the legal authority, and amounts to a whimsical, capricious, oppressive, arbitrary and despotic exercise thereto. Moreover, he cannot invoke of executive privilege because he already waived it when he complied with the Court’s order to the unqualified disclosure of the official copies of the final draft of the MOA-AD. In addition, the LGU petitioners has the right to be involved in matters related to such peace talks as enshrined in the State policy. The MOA-AD is one peculiar program that unequivocally and unilaterally vests ownership of a vast territory to the Bangsamoro people, which could pervasively and drastically result to the diaspora or displacement of a great number of inhabitants from their total environment. With respect to the ICC/IPPs they also have the right to participate fully at all levels on decisions that would clearly affect their lives, rights and destinies. The MOA-AD is an instrument recognizing ancestral domain, hence it should have observed the free and prior informed consent to the ICC/IPPs; but it failed to do so. More specially noted by the court is the excess in authority exercised by the respondent—since they allowed delineation and recognition of ancestral domain claim by mere agreement and compromise; such power cannot be found in IPRA or in any law to the effect. 3rd issue: With regard to the provisions of the MOA-AD, there can be no question that they cannot be all accommodated under the present Constitution and laws. Not only its specific provisions but the very concept underlying them:



On matters of the Constitution.

Association as the type of relationship governing between the parties. The parties manifested that in crafting the MOA-AD, the term association was adapted from the international law. In international law, association happens when two states of equal power voluntarily establish durable links i.e. the one state, the associate, delegates certain responsibilities to the other, principal, while maintaining its international status as state; free association is a middle ground between integration and independence. The MOA-AD contains many provisions that are consistent with the international definition of association which fairly would deduced that the agreement vest into the BJE a status of an associated state, or at any rate, a status closely approximating it. The court vehemently objects because the principle of association is not recognized under the present Constitution.



On the recognition of the BJE entity as a state. The concept implies power beyond what the Constitution can grant to a local government; even the ARMM do not have such recognition; and the fact is such concept implies recognition of the associated entity as a state. There is nothing in the law that contemplate any state within the jurisdiction other than the Philippine State, much less does it provide for a transitory status that aims to prepare any part of Philippine territory for independence. The court disagrees with the respondent that the MOA-AD merely expands the ARMM. BJE is a state in all but name as it meets the criteria of a state laid down in the Montevideo Convention, namely, a permanent population, a defined territory, a government, and a capacity to enter into relations with other states. As such the MOA-AD clearly runs counter to the national sovereignty and territorial integrity of the Republic.



On the expansion of the territory of the BJE. The territory included in the BJE includes those areas who voted in the plebiscite for them to become part of the ARMM. The stipulation of the respondents in the MOA-AD that these areas need not participate in the plebiscite is in contrary to the

express provision of the Constitution. The law states that that "[t]he creation of the autonomous region shall be effective when approved by a majority of the votes cast by the constituent units in a plebiscite called for the purpose, provided that only provinces, cities, and geographic areas voting favorably in such plebiscite shall be included in the autonomous region." Clearly, assuming that the BJE is just an expansion of the ARMM, it would still run afoul the wordings of the law since those included in its territory are areas which voted in its inclusion to the ARMM and not to the BJE.



On the powers vested in the BJE as an entity. The respondents contend that the powers vested to the BJE in the MOA-AD shall be within sub-paragraph 9 of sec 20, art. 10 of the constitution and that a mere passage of a law is necessary in order to vest in the BJE powers included in the agreement. The Court was not persuaded. SC ruled that such conferment calls for amendment of the Constitution; otherwise new legislation will not concur with the Constitution. Take for instance the treaty making power vested to the BJE in the MOA-AD. The Constitution is clear that only the President has the sole organ and is the country’s sole representative with foreign nation. Should the BJE be granted with the authority to negotiate with other states, the former provision must be amended consequently. Section 22 must also be amended—the provision of the law that promotes national unity and development. Because clearly, associative arrangement of the MOA-AD does not epitomize national unity but rather, of semblance of unity. The associative ties between the BJE and the national government, the act of placing a portion of Philippine territory in a status which, in international practice, has generally been a preparation for independence, is certainly not conducive to national unity.



On matters of domestic statutes.

o Provisions contrary to the organic act of ARMM. RA 9054 is a bar to the adoption of the definition of Bangsamoro people used in the MOA-AD. Said law

specifically distinguishes between the Bangsamoro people and the Tribal peoples that is contrary with the definition of the MOA-AD which includes all indigenous people of Mindanao. o Provisions contrary to the IPRA law. Also, the delineation and recognition of the ancestral domain is a clear departure from the procedure embodied in the IPRA law which ironically is the term of reference of the MOA-AD.



On matters of international law.

The Philippines adopts the generally accepted principle of international law as part of the law of the land. In international law, the right to self-determination has long been recognized which states that people can freely determine their political status and freely pursue their economic, social, and cultural development. There are the internal and external self-determination—internal, meaning the selfpursuit of man and the external which takes the form of the assertion of the right to unilateral secession. This principle of self-determination is viewed with respect accorded to the territorial integrity of existing states. External selfdetermination is only afforded in exceptional cases when there is an actual block in the meaningful exercise of the right to internal self-determination. International law, as a general rule, subject only to limited and exceptional cases, recognizes that the right of disposing national territory is essentially an attribute of the sovereignty of every state. On matters relative to indigenous people, international law states that indigenous peoples situated within states do not have a general right to independence or secession from those states under international law, but they do have rights amounting to what was discussed above as the right to internal self-determination; have the right to autonomy or self-government in matters relating to their internal and local affairs, as well as ways and means for financing their autonomous functions; have the right to the lands, territories and resources which they have traditionally owned, occupied or otherwise used or acquired. Clearly, there is nothing in the law that required the State to guarantee the indigenous people their own police and security force; but rather, it shall be the State, through police officers, that will provide for the protection of the people.

With regards to the autonomy of the indigenous people, the law does not obligate States to grant indigenous peoples the near-independent status of a state; since it would impair the territorial integrity or political unity of sovereign and independent states.



On the basis of the suspensive clause.

o It was contented by the respondents that grave abuse of discretion cannot be had, since the provisions assailed as unconstitutional shall not take effect until the necessary changes to the legal framework are effected. The Court is not persuaded. This suspensive clause runs contrary to Memorandum of Instructions from the President stating that negotiations shall be conducted in accordance to the territorial integrity of the country—such was negated by the provision on association incorporated in the MOA-AD. Apart from this, the suspensive clause was also held invalid because of the delegated power to the GRP Peace panel to advance peace talks even if it will require new legislation or even constitutional amendments. The legality of the suspensive clause hence hinges on the query whether the President can exercise such power as delegated by EO No.3 to the GRP Peace Panel. Well settled is the rule that the President cannot delegate a power that she herself does not possess. The power of the President to conduct peace negotiations is not explicitly mentioned in the Constitution but is rather implied from her powers as Chief Executive and Commander-in-chief. As Chief Executive, the President has the general responsibility to promote public peace, and as Commander-in-Chief, she has the more specific duty to prevent and suppress rebellion and lawless violence. As such, the President is given the leeway to explore, in the course of peace negotiations, solutions that may require changes to the Constitution for their implementation. At all event, the president may not, of course, unilaterally implement the solutions that she considers viable; but she may not be prevented from submitting them as recommendations to Congress, which could then, if it is minded, act upon them pursuant to the legal procedures for constitutional amendment and revision. While the President does not possess constituent powers - as those powers may be

exercised only by Congress, a Constitutional Convention, or the people through initiative and referendum - she may submit proposals for constitutional change to Congress in a manner that does not involve the arrogation of constituent powers. Clearly, the principle may be inferred that the President - in the course of conducting peace negotiations - may validly consider implementing even those policies that require changes to the Constitution, but she may not unilaterally implement them without the intervention of Congress, or act in any way as if the assent of that body were assumed as a certainty. The President’s power is limited only to the preservation and defense of the Constitution but not changing the same but simply recommending proposed amendments or revisions. o The Court ruled that the suspensive clause is not a suspensive condition but is a term because it is not a question of whether the necessary changes to the legal framework will take effect; but, when. Hence, the stipulation is mandatory for the GRP to effect the changes to the legal framework –which changes would include constitutional amendments. Simply put, the suspensive clause is inconsistent with the limits of the President's authority to propose constitutional amendments, it being a virtual guarantee that the Constitution and the laws of the Republic of the Philippines will certainly be adjusted to conform to all the "consensus points" found in the MOA-AD. Hence, it must be struck down as unconstitutional.



On the concept underlying the MOA-AD.

While the MOA-AD would not amount to an international agreement or unilateral declaration binding on the Philippines under international law, respondents' act of guaranteeing amendments is, by itself, already a constitutional violation that renders the MOA-AD fatally defective. The MOA-AD not being a document that can bind the Philippines under international law notwithstanding, respondents' almost consummated act of guaranteeing amendments to the legal framework is, by itself, sufficient to constitute grave abuse of discretion. The grave abuse lies not in the fact that they considered, as a solution to the Moro Problem, the creation of a state within a state, but in their brazen willingness to guarantee that Congress and the sovereign Filipino people would give their imprimatur to their solution.Upholding such an act would amount to authorizing a usurpation of the

constituent powers vested only in Congress, a Constitutional Convention, or the people themselves through the process of initiative, for the only way that the Executive can ensure the outcome of the amendment process is through an undue influence or interference with that process.

CASE DIGEST: THE PROVINCE OF NORTH COTABATO, ET AL . V . THE GOVERNMENT OF THE REPUBLIC OF THE PHILIPPINES, ET AL . Published by arce on September 7, 2013 | Leave a response

THE PROVINCE OF NORTH COTABATO, et al . v . THE GOVERNMENT OF THE REPUBLIC OF THE PHILIPPINES, et al . President Gloria Macapagal-Arroyo, in line with the government‘s policy of pursuing peace negotiations with the Moro Islamic Liberation Front (MILF), asked Prime Minister Mahathir Mohammad to convince the MILF to continue negotiating with the government. MILF, thereafter, convened its Central Committee and decided to meet with the Government of the Republic of the Philippines (GRP). Formal peace talks were held in Libya which resulted to the crafting of the GRP-MILF Tripoli Agreement on Peace (Tripoli Agreement 2001) which consists of three (3) aspects: a.) security aspect; b.) rehabilitation aspect; and c.) ancestral domain aspect. Various negotiations were held which led to the finalization of the Memorandum of Agreement on the Ancestral Domain (MOA-AD). The said memorandum was set to be signed last August 5, 2008. In its body, it grants ―the authority and jurisdiction over the Ancestral Domain and Ancestral Lands of the Bangsamoro to the Bangsamoro Juridical Entity (BJE). The latter, in addition, has the freedom to enter into any economic cooperation and trade relation with foreign countries. ―The sharing between the Central Government and the BJE of total production pertaining to natural resources is to be 75:25 in favor of the BJE. The MOA-AD further provides for the extent of the territory of the Bangsamoro. It describes it as ―the land mass as well as the maritime, terrestrial, fluvial and alluvial domains, including the aerial domain and the atmospheric space above it, embracing the Mindanao-Sulu-Palawan geographic region. With regard to governance, on the other hand, a shared responsibility and authority between the Central Government and BJE was provided. The relationship was described as ―associative. With the formulation of the MOA-AD, petitioners aver that the negotiation and finalization of the MOA-AD violates constitutional and statutory provisions on public consultation, as mandated by Executive Order No. 3, and right to information. They further contend that it violates the Constitution and laws. Hence, the filing of the petition. ISSUES: 1) Whether or not the MOA-AD violates constitutional and statutory provisions on public consultation and right to information 2) Whether or not the MOA-AD violates the Constitution and the laws. HELD: The MOA-AD subject of the present cases is of public concern, involving as it does the sovereignty and territorial integrity of the State, which directly affects the lives of the public at large. Intended as a ―splendid symmetry to the right to information under the Bill of Rights is the policy of public disclosure under Section 28, Article II of the

Constitution which provides that subject to reasonable conditions prescribed by law, the State adopts and implements a policy of full public disclosure of all its transactions involving public interest. Moreover, the policy of full public disclosure enunciated in above-quoted Section 28 complements the right of access to information on matters of public concern found in the Bill of Rights. The right to information guarantees the right of the people to demand information, while Section 28 recognizes the duty of officialdom to give information even if nobody demands. The policy of public disclosure establishes a concrete ethical principle for the conduct of public affairs in a genuinely open democracy, with the people‘s right to know as the centerpiece. It is a mandate of the State to be accountable by following such policy. These provisions are vital to the exercise of the freedom of expression and essential to hold public officials at all times accountable to the people. Indubitably, the effectivity of the policy of public disclosure need not await the passing of a statute. As Congress cannot revoke this principle, it is merely directed to provide for ―reasonable safeguards.‖ The complete and effective exercise of the right to information necessitates that its complementary provision on public disclosure derive the same self-executory nature. Since both provisions go handin-hand, it is absurd to say that the broader right to information on matters of public concern is already enforceable while the correlative duty of the State to disclose its transactions involving public interest is not enforceable until there is an enabling law. Respondents cannot thus point to the absence of an implementing legislation as an excuse in not effecting such policy. An essential element of these freedoms is to keep open a continuing dialogue or process of communication between the government and the people. It is in the interest of the State that the channels for free political discussion be maintained to the end that the government may perceive and be responsive to the people‘s will. Envisioned to be corollary to the twin rights to information and disclosure is the design for feedback mechanisms. The imperative of a public consultation, as a species of the right to information, is evident in the ―marching orders‖ to respondents. The mechanics for the duty to disclose information and to conduct public consultation regarding the peace agenda and process is manifestly provided by E.O. No. 3. The preambulatory clause of E.O. No. 3 declares that there is a need to further enhance the contribution of civil society to the comprehensive peace process by institutionalizing the people‘s participation. One of the three underlying principles of the comprehensive peace process is that it ―should be community-based, reflecting the sentiments, values and principles important to all Filipinos and ―shall be defined not by the government alone, nor by the different contending groups only, but by all Filipinos as one community. Included as a component of the comprehensive peace process is consensusbuilding and empowerment for peace, which includes ―continuing consultations on both national and local levels to build consensus for a peace agenda and process, and the mobilization and facilitation of people‘s participation in the peace process.Clearly, E.O. No. 3 contemplates not just the conduct of a plebiscite to effectuate “continuing” consultations, contrary to respondents’ position that plebiscite is “more than sufficient consultation.Further, E.O. No. 3 enumerates the functions and responsibilities of the PAPP, one of which is to ―conduct regular dialogues with the National Peace Forum (NPF) and other peace partners to seek relevant information, comments, recommendations as well as to render appropriate and timely reports on the progress of the comprehensive peace process. E.O. No. 3 mandates the establishment of the NPF to be ―the principal forum for the Presidential Adviser on Peace Progress (PAPP) to

consult with and seek advi[c]e from the peace advocates, peace partners and concerned sectors of society on both national and local levels, on the implementation of the comprehensive peace process, as well as for government[-]civil society dialogue and consensus-building on peace agenda and initiatives. In fine, E.O. No. 3 establishes petitioners’ right to be consulted on the peace agenda, as a corollary to the constitutional right to information and disclosure. In general, the objections against the MOA-AD center on the extent of the powers conceded therein to the BJE. Petitioners assert that the powers granted to the BJE exceed those granted to any local government under present laws, and even go beyond those of the present ARMM. Before assessing some of the specific powers that would have been vested in the BJE, however, it would be useful to turn first to a general idea that serves as a unifying link to the different provisions of the MOA-AD, namely, the international law concept of association. Significantly, the MOA-AD explicitly alludes to this concept, indicating that the Parties actually framed its provisions with it in mind. Association is referred to in paragraph 3 on TERRITORY, paragraph 11 on RESOURCES, and paragraph 4 on GOVERNANCE. It is in the last mentioned provision, however, that the MOA-AD most clearly uses it to describe the envisioned relationship between the BJE and the Central Government. 4. The relationship between the Central Government and the Bangsamoro juridical entity shall be associative characterized by shared authority and responsibility with a structure of governance based on executive, legislative, judicial and administrative institutions with defined powers and functions in the comprehensive compact. A period of transition shall be established in a comprehensive peace compact specifying the relationship between the Central Government and the BJE. The nature of the ―associative relationship may have been intended to be defined more precisely in the still to be forged Comprehensive Compact. Nonetheless, given that there is a concept of ―association in international law, and the MOA-AD – by its inclusion of international law instruments in its TOR– placed itself in an international legal context, that concept of association may be brought to bear in understanding the use of the term ―associative in the MOA-AD. The MOA-AD contains many provisions which are consistent with the international legal concept of association, specifically the following: the BJE‘s capacity to enter into economic and trade relations with foreign countries, the commitment of the Central Government to ensure the BJE‘s participation in meetings and events in the ASEAN and the specialized UN agencies, and the continuing responsibility of the Central Government over external defense. Moreover, the BJE‘s right to participate in Philippine official missions bearing on negotiation of border agreements, environmental protection, and sharing of revenues pertaining to the bodies of water adjacent to or between the islands forming part of the ancestral domain, resembles the right of the governments of FSM and the Marshall Islands to be consulted by the U.S. government on any foreign affairs matter affecting them. These provisions of the MOA indicate, among other things, that the Parties aimed to vest in the BJE the status of an associated state or, at any rate, a status closely approximating it. The concept of association is not recognized under the present Constitution. No province, city, or municipality, not even the ARMM, is recognized under our laws as having an ―associative‖ relationship with the national government. Indeed, the concept implies powers that go beyond anything ever granted by the Constitution to any local or regional government. It also implies the recognition of the associated entity as a state. The Constitution, however, does not

contemplate any state in this jurisdiction other than the Philippine State, much less does it provide for a transitory status that aims to prepare any part of Philippine territory for independence. Even the mere concept animating many of the MOA-AD‘s provisions, therefore, already requires for its validity the amendment of constitutional provisions, specifically the following provisions of Article X: SECTION 1. The territorial and political subdivisions of the Republic of the Philippines are the provinces, cities, municipalities, and barangays. There shall be autonomous regions in Muslim Mindanao and the Cordilleras as hereinafter provided. SECTION 15. There shall be created autonomous regions in Muslim Mindanao and in the Cordilleras consisting of provinces, cities, municipalities, and geographical areas sharing common and distinctive historical and cultural heritage, economic and social structures, and other relevant characteristics within the framework of this Constitution and the national sovereignty as well as territorial integrity of the Republic of the Philippines. It is not merely an expanded version of the ARMM, the status of its relationship with the national government being fundamentally different from that of the ARMM. Indeed, BJE is a state in all but name as it meets the criteria of a state laid down in the Montevideo Convention, namely, a permanent population, a defined territory, a government, and a capacity to enter into relations with other states. The defining concept underlying the relationship between the national government and the BJE being itself contrary to the present Constitution, it is not surprising that many of the specific provisions of the M OA-AD on the formation and powers of the BJE are in conflict with the Constitution and the laws. Article X, Section 18 of the Constitution provides that ―[t]he creation of the autonomous region shall be effective when approved by a majority of the votes cast by the constituent units in a plebiscite called for the purpose, provided that only provinces, cities, and geographic areas voting favorably in such plebiscite shall be included in the autonomous region. The BJE is more of a state than an autonomous region. But even assuming that it is covered by the term ―autonomous region in the constitutional provision just quoted, the MOA-AD would still be in conflict with it. Under paragraph 2(c) on TERRITORY in relation to 2(d) and 2(e), the present geographic area of the ARMM and, in addition, the municipalities of Lanao del Norte which voted for inclusion in the ARMM during the 2001 plebiscite – Baloi, Munai, Nunungan, Pantar, Tagoloan and Tangkal – are automatically part of the BJE without need of another plebiscite, in contrast to the areas under Categories A and B mentioned earlier in the overview. That the present components of the ARMM and the above-mentioned municipalities voted for inclusion therein in 2001, however, does not render another plebiscite unnecessary under the Constitution, precisely because what these areas voted for then was their inclusion in the ARMM, not the BJE. Article II, Section 22 of the Constitution must also be amended if the scheme envisioned in the MOA-AD is to be effected. That constitutional provision states: ―The State recognizes and promotes the rights of indigenous cultural communities within the framework of national unity and development. An associative arrangement does not uphold national unity. While there may be a semblance of unity because of the associative ties between the BJE and the national government, the act of placing a portion of Philippine territory in a status which, in international practice, has generally been a preparation for independence, is certainly not conducive to national unity.

The MOA-AD cannot be reconciled with the present Constitution and laws. Not only its specific provisions but the very concept underlying them, namely, the associative relationship envisioned between the GRP and the BJE, are unconstitutional, for the concept presupposes that the associated entity is a state and implies that the same is on its way to independence. While there is a clause in the MOA-AD stating that the provisions thereof inconsistent with the present legal framework will not be effective until that framework is amended, the same does not cure its defect. The inclusion of provisions in the MOA-AD establishing an associative relationship between the BJE and the Central Government is, itself, a violation of the Memorandum of Instructions from the President dated March 1, 2001, addressed to the government peace panel. Moreover, as the clause is worded, it virtually guarantees that the necessary amendments to the Constitution and the laws will eventually be put in place. Neither the GRP Peace Panel nor the President herself is authorized to make such a guarantee. Upholding such an act would amount to authorizing a usurpation of the constituent powers vested only in Congress, a Constitutional Convention, or the people themselves through the process of initiative, for the only way that the Executive can ensure the outcome of the amendment process is through an undue influence or interference with that process.

PRESS RELEASE THE SOUTH CHINA SEA ARBITRATION (THE REPUBLIC OF THE PHILIPPINES V . THE PEOPLE’ S REPUBLIC OF CHINA ) The Hague, 12 July 2016 The Tribunal Renders Its Award A unanimous Award has been issued today by the Tribunal constituted under Annex VII to the UnitedNations Convention on the Law of the Sea (the “Convention”) in the arbitration instituted by the Republic of the Philippines against the People’s Republic of China. This arbitration concerned the role of historic rights and the source of maritime entitlements in the South China Sea, the status of certain maritime features and the maritime entitlements they are capable of generating, and the lawfulness of certain actions by China that were alleged by the Philippines to violate the Convention. In light of limitations on compulsory dispute settlement under the Convention, the Tribunal has emphasized that it does not rule on any question of sovereignty over land territory and does not delimit any boundary between the Parties. China has repeatedly stated that “it will neither accept nor participate in the arbitration unilaterally initiated by the Philippines.” Annex VII, however, provides that the “[a]bsence of a party or failure of a party to defend its case shall not constitute a bar to the proceedings.” Annex VII also provides that, in the event that a party does not participate in the proceedings, a tribunal “must satisfy itself not only that it has jurisdiction over the dispute but also that the claim is well founded in fact and law.” Accordingly, throughout these proceedings, the Tribunal has taken steps to test the accuracy of the Philippines’ claims, including by requesting further written submissions from the Philippines, by questioning the Philippines both prior to and during two hearings, by appointing independent experts to report to the Tribunal on technical matters, and by obtaining historical evidence concerning features in the South China Sea and providing it to the Parties for comment. China has also made clear—through the publication of a Position Paper in December 2014 and in other official statements—that, in its view, the Tribunal lacks jurisdiction in this matter. Article 288 of the Convention provides that: “In the event of a dispute as to whether a court or tribunal has jurisdiction, the matter shall be settled by decision of that court or tribunal.” Accordingly, the Tribunal convened a hearing on jurisdiction and admissibility in July 2015 and rendered an Award on Jurisdiction and Admissibility on 29 October 2015, deciding some issues of jurisdiction and deferring others for further consideration. The Tribunal then convened a hearing on the merits from 24 to 30 November 2015. The Award of today’s date addresses the issues of jurisdiction not decided in the Award on Jurisdiction and Admissibility and the merits of the Philippines’ claims over which the Tribunal has jurisdiction. The Award is final and binding, as set out in Article 296 of the Convention and Article 11 of Annex VII. Historic Rights and the ‘Nine-Dash Line’: The Tribunal found that it has jurisdiction to consider the Parties’ dispute concerning historic rights and the source of maritime entitlements in the South China Sea. On the merits, the Tribunal concluded that the Convention comprehensively allocates rights to maritime areas and that protections for pre-existing rights to resources were considered, but not adopted in the Convention. Accordingly, the Tribunal concluded that, to the extent China had historic rights to resources in the waters of the South China Sea, such rights were extinguished to the

extent they were incompatible with the exclusive economic zones provided for in the Convention. The Tribunal also noted that, although Chinese navigators and fishermen, as well as those of other States, had historically made use of the islands in the South China Sea, there was no evidence that China had historically exercised exclusive control over the waters or their resources. The Tribunal concluded that there was no legal basis for China to claim historic rights to resources within the sea areas falling within the ‘nine-dash line’. Status of Features: The Tribunal next considered entitlements to maritime areas and the status of features. The Tribunal first undertook an evaluation of whether certain reefs claimed by China are above water at high tide. Features that are above water at high tide generate an entitlement to at least a 12 nautical mile territorial sea, whereas features that are submerged at high tide do not. The Tribunal noted that the reefs have been heavily modified by land reclamation and construction, recalled that the Convention classifies features on their natural condition, and relied on historical materials in evaluating the features. The Tribunal then considered whether any of the features claimed by China could generate maritime zones beyond 12 nautical miles. Under the Convention, islands generate an exclusive economic zone of 200 nautical miles and a continental shelf, but “[r]ocks which cannot sustain human habitation or economic life of their own shall have no exclusive economic zone or continental shelf.” The Tribunal concluded that this provision depends upon the objective capacity of a feature, in its natural condition, to sustain either a stable community of people or economic activity that is not dependent on outside resources or purely extractive in nature. The Tribunal noted that the current presence of official personnel on many of the features is dependent on outside support and not reflective of the capacity of the features. The Tribunal found historical evidence to be more relevant and noted that the Spratly Islands were historically used by small groups of fishermen and that several Japanese fishing and guano mining enterprises were attempted. The Tribunal concluded that such transient use does not constitute inhabitation by a stable community and that all of the historical economic activity had been extractive. Accordingly, the Tribunal concluded that none of the Spratly Islands is capable of generating extended maritime zones. The Tribunal also held that the Spratly Islands cannot generate maritime zones collectively as a unit. Having found that none of the features claimed by China was capable of generating an exclusive economic zone, the Tribunal found that it could—without delimiting a boundary—declare that certain sea areas are within the exclusive economic zone of the Philippines, because those areas are not overlapped by any possible entitlement of China. Lawfulness of Chinese Actions: The Tribunal next considered the lawfulness of Chinese actions in the South China Sea. Having found that certain areas are within the exclusive economic zone of the Philippines, the Tribunal found that China had violated the Philippines’ sovereign rights in its exclusive economic zone by (a) interfering with Philippine fishing and petroleum exploration, (b) constructing artificial islands and (c) failing to prevent Chinese fishermen from fishing in the zone. The Tribunal also held that fishermen from the Philippines (like those from China) had traditional fishing rights at Scarborough Shoal and that China had interfered with these rights in restricting access. The Tribunal further held that Chinese law enforcement vessels had unlawfully created a serious risk of collision when they physically obstructed Philippine vessels. Harm to Marine Environment: The Tribunal considered the effect on the marine environment of China’s recent large-scale land reclamation and construction of artificial islands at seven features in the Spratly Islands and found that China had caused severe harm to the coral reef environment and violated its obligation to preserve and protect fragile ecosystems and the habitat of depleted, threatened, or endangered species. The Tribunal also found that Chinese authorities were aware that

Chinese fishermen have harvested endangered sea turtles, coral, and giant clams on a substantial scale in the South China Sea (using methods that inflict severe damage on the coral reef environment) and had not fulfilled their obligations to stop such activities. Aggravation of Dispute: Finally, the Tribunal considered whether China’s actions since the commencement of the arbitration had aggravated the dispute between the Parties. The Tribunal found that it lacked jurisdiction to consider the implications of a stand-off between Philippine marines and Chinese naval and law enforcement vessels at Second Thomas Shoal, holding that this dispute involved military activities and was therefore excluded from compulsory settlement. The Tribunal found, however, that China’s recent large-scale land reclamation and construction of artificial islands was incompatible with the obligations on a State during dispute resolution proceedings, insofar as China has inflicted irreparable harm to the marine environment, built a large artificial island in the Philippines’ exclusive economic zone, and destroyed evidence of the natural condition of features in the South China Sea that formed part of the Parties’ dispute. An expanded summary of the Tribunal’s decisions is set out below. The Tribunal was constituted on 21 June 2013 pursuant to the procedure set out in Annex VII of the Convention to decide the dispute presented by the Philippines. The Tribunal is composed of Judge Thomas A. Mensah of Ghana, Judge Jean-Pierre Cot of France, Judge Stanislaw Pawlak of Poland, Professor Alfred H.A. Soons of the Netherlands, and Judge Rüdiger Wolfrum of Germany. Judge Thomas A. Mensah serves as President of the Tribunal. The Permanent Court of Arbitration acts as the Registry in the proceedings. Further information about the case may be found at www.pcacases.com/web/view/7, including the Award on Jurisdiction and Admissibility, the Rules of Procedure, earlier Press Releases, hearing transcripts, and photographs. Procedural Orders, submissions by the Philippines, and reports by the Tribunal’s experts will be made available in due course, as will unofficial Chinese translations of the Tribunal’s Awards. Background to the Permanent Court of Arbitration The Permanent Court of Arbitration (PCA) is an intergovernmental organization established by the 1899 Hague Convention on the Pacific Settlement of International Disputes. The PCA has 121 Member States. Headquartered at the Peace Palace in The Hague, the Netherlands, the PCA facilitates arbitration, conciliation, fact-finding, and other dispute resolution proceedings among various combinations of States, State entities, intergovernmental organizations, and private parties. The PCA’s International Bureau is currently administering 8 interstate disputes, 73 investor-State arbitrations, and 34 cases arising under contracts involving a State or other public entity. The PCA has administered 12 cases initiated by States under Annex VII to the United Nations Convention on the Law of the Sea. In July 2013, the Tribunal in the South China Sea Arbitration appointed the PCA to serve as Registry for the proceedings. The Tribunal’s Rules of Procedure provide that the PCA shall “maintain an archive of the arbitral proceedings and provide appropriate registry services as directed by the Arbitral Tribunal.” Such services include assisting with the identification and appointment of experts; publishing information about the arbitration and issuing press releases; organizing the hearings at the Peace Palace in The Hague; and the financial management of the case, which involves holding a deposit for expenses in the arbitration, such as to pay arbitrator fees, experts, technical support, court

reporters etc. The Registry also serves as the channel of communications amongst the Parties and the Tribunal and observer States.

Photograph: Hearing in session, July 2015, Peace Palace, The Hague. Clockwise from top left: Registrar and PCA Senior Legal Counsel Judith Levine; Judge Stanislaw Pawlak; Professor Alfred H. A. Soons; Judge Thomas A. Mensah (Presiding Arbitrator); Judge Jean-Pierre Cot; Judge Rüdiger Wolfrum; PCA Senior Legal Counsel Garth Schofield; former Secretary for Foreign Affairs of the Philippines, Mr. Albert F. Del Rosario; former Solicitor General Mr. Florin T. Hilbay, Counsel for the Philippines; Mr. Paul S. Reichler; Professor Philippe Sands; Professor Bernard H. Oxman; Professor Alan E. Boyle; Mr. Lawrence H. Martin.

SUMMARY OF THE TRIBUNAL’S DECISIONS ON ITS JURISDICTION AND ON THE MERITS OF THE PHILIPPINES ’ CLAIMS 1. Background to the Arbitration The South China Sea Arbitration between the Philippines and China concerned an application by the Philippines for rulings in respect of four matters concerning the relationship between the Philippines and China in the South China Sea. First, the Philippines sought a ruling on the source of the Parties’

rights and obligations in the South China Sea and the effect of the United Nations Convention on the Law of the Sea (“Convention”) on China’s claims to historic rights within its so-called ‘nine-dash line’. Second, the Philippines sought a ruling on whether certain maritime features claimed by both China and the Philippines are properly characterized as islands, rocks, low-tide elevations or submerged banks under the Convention. The status of these features under the Convention determines the maritime zones they are capable of generating. Third, the Philippines sought rulings on whether certain Chinese actions in the South China Sea have violated the Convention, by interfering with the exercise of the Philippines’ sovereign rights and freedoms under the Convention or through construction and fishing activities that have harmed the marine environment. Finally, the Philippines sought a ruling that certain actions taken by China, in particular its large-scale land reclamation and construction of artificial islands in the Spratly Islands since this arbitration was commenced, have unlawfully aggravated and extended the Parties’ dispute. The Chinese Government has adhered to the position of neither accepting nor participating in these arbitral proceedings. It has reiterated this position in diplomatic notes, in the “Position Paper of the Government of the People’s Republic of China on the Matter of Jurisdiction in the South China Sea Arbitration Initiated by the Republic of the Philippines” dated 7 December 2014 (“China’s Position Paper”), in letters to members of the Tribunal from the Chinese Ambassador to the Kingdom of the Netherlands, and in many public statements. The Chinese Government has also made clear that these statements and documents “shall by no means be interpreted as China’s participation in the arbitral proceeding in any form.” Two provisions of the Convention address the situation of a party that objects to the jurisdiction of a tribunal and declines to participate in the proceedings: (a) Article 288 of the Convention provides that: “In the event of a dispute as to whether a court or tribunal has jurisdiction, the matter shall be settled by decision of that court or tribunal.” (b) Article 9 of Annex VII to the Convention provides that: If one of the parties to the dispute does not appear before the arbitral tribunal or fails to defend its case, the other party may request the tribunal to continue the proceedings and to make its award. Absence of a party or failure of a party to defend its case shall not constitute a bar to the proceedings. Before making its award, the arbitral tribunal must satisfy itself not only that it has jurisdiction over the dispute but also that the claim is well founded in fact and law. Throughout these proceedings, the Tribunal has taken a number of steps to fulfil its duty to satisfy itself as to whether it has jurisdiction and whether the Philippines’ claims are “well founded in fact and law”. With respect to jurisdiction, the Tribunal decided to treat China’s informal communications as equivalent to an objection to jurisdiction, convened a Hearing on Jurisdiction and Admissibility on 7 to 13 July 2015, questioned the Philippines both before and during the hearing on matters of jurisdiction, including potential issues not raised in China’s informal communications, and issued an Award on Jurisdiction and Admissibility on 29 October 2015 (the “Award on Jurisdiction”), deciding some issues of jurisdiction and deferring others for further consideration in conjunction with the merits of the Philippines’ claims. With respect to the merits, the Tribunal sought to test the accuracy of the Philippines’ claims by requesting further written submissions from the Philippines, by convening a hearing on the merits from 24 to 30 November 2015, by questioning the Philippines both before and during the hearing with respect to its claims, by appointing independent experts to report to the Tribunal on technical matters, and by obtaining historical records and

hydrographic survey data for the South China Sea from the archives of the United Kingdom Hydrographic Office, the National Library of France, and the French National Overseas Archives and providing it to the Parties for comment, along with other relevant materials in the public domain. 2. The Parties’ Positions The Philippines made 15 Submissions in these proceedings, requesting the Tribunal to find that: (1) China’s maritime entitlements in the South China Sea, like those of the Philippines, may not extend beyond those expressly permitted by the United Nations Convention on the Law of the Sea; (2) China’s claims to sovereign rights jurisdiction, and to “historic rights”, with respect to the maritime areas of the South China Sea encompassed by the so-called “nine-dash line” are contrary to the Convention and without lawful effect to the extent that they exceed the geographic and substantive limits of China’s maritime entitlements expressly permitted by UNCLOS; (3) Scarborough Shoal generates no entitlement to an exclusive economic zone or continental shelf; (4) Mischief Reef, Second Thomas Shoal, and Subi Reef are low-tide elevations that do not generate entitlement to a territorial sea, exclusive economic zone or continental shelf, and are not features that are capable of appropriation by occupation or otherwise; (5) Mischief Reef and Second Thomas Shoal are part of the exclusive economic zone and continental shelf of the Philippines; (6) Gaven Reef and McKennan Reef (including Hughes Reef) are low-tide elevations that do not generate entitlement to a territorial sea, exclusive economic zone or continental shelf, but their lowwater line may be used to determine the baseline from which the breadth of the territorial sea of Namyit and Sin Cowe, respectively, is measured; (7) Johnson Reef, Cuarteron Reef and Fiery Cross Reef generate no entitlement to an exclusive economic zone or continental shelf; (8) China has unlawfully interfered with the enjoyment and exercise of the sovereign rights of the Philippines with respect to the living and non-living resources of its exclusive economic zone and continental shelf; (9) China has unlawfully failed to prevent its nationals and vessels from exploiting the living resources in the exclusive economic zone of the Philippines; (10) China has unlawfully prevented Philippine fishermen from pursuing their livelihoods by interfering with traditional fishing activities at Scarborough Shoal; (11) China has violated its obligations under the Convention to protect and preserve the marine environment at Scarborough Shoal, Second Thomas Shoal, Cuarteron Reef, Fiery Cross Reef, Gaven Reef, Johnson Reef, Hughes Reef and Subi Reef; (12) China’s occupation of and construction activities on Mischief Reef (a) violate the provisions of the Convention concerning artificial islands, installations and structures;

(b) violate China’s duties to protect and preserve the marine environment under the Convention; and (c) constitute unlawful acts of attempted appropriation in violation of the Convention; (13) China has breached its obligations under the Convention by operating its law enforcement vessels in a dangerous manner, causing serious risk of collision to Philippine vessels navigating in the vicinity of Scarborough Shoal; (14) Since the commencement of this arbitration in January 2013, China has unlawfully aggravated and extended the dispute by, among other things: (a) interfering with the Philippines’ rights of navigation in the waters at, and adjacent to, Second Thomas Shoal; (b) preventing the rotation and resupply of Philippine personnel stationed at Second Thomas Shoal; (c) endangering the health and well-being of Philippine personnel stationed at Second Thomas Shoal; and (d) conducting dredging, artificial island-building and construction activities at Mischief Reef, Cuarteron Reef, Fiery Cross Reef, Gaven Reef, Johnson Reef, Hughes Reef and Subi Reef; and (15) China shall respect the rights and freedoms of the Philippines under the Convention, shall comply with its duties under the Convention, including those relevant to the protection and preservation of the marine environment in the South China Sea, and shall exercise its rights and freedoms in the South China Sea with due regard to those of the Philippines under the Convention. With respect to jurisdiction, the Philippines has asked the Tribunal to declare that the Philippines’ claims “are entirely within its jurisdiction and are fully admissible.” China does not accept and is not participating in this arbitration but stated its position that the Tribunal “does not have jurisdiction over this case.” In its Position Paper, China advanced the following arguments: 





The essence of the subject-matter of the arbitration is the territorial sovereignty over several maritime features in the South China Sea, which is beyond the scope of the Convention and does not concern the interpretation or application of the Convention; China and the Philippines have agreed, through bilateral instruments and the Declaration on the Conduct of Parties in the South China Sea, to settle their relevant disputes through negotiations. By unilaterally initiating the present arbitration, the Philippines has breached its obligation under international law; Even assuming, arguendo, that the subject-matter of the arbitration were concerned with the interpretation or application of the Convention, that subject-matter would constitute an integral part of maritime delimitation between the two countries, thus falling within the scope of the declaration filed by China in 2006 in accordance with the Convention, which excludes, inter alia, disputes concerning maritime delimitation from compulsory arbitration and other compulsory dispute settlement procedures;

Although China has not made equivalent public statements with respect to the merits of the majority of the Philippines’ claims, the Tribunal has sought throughout the proceedings to ascertain China’s position on the basis of its contemporaneous public statements and diplomatic correspondence. 3. The Tribunal’s Decisions on the Scope of its Jurisdiction The Tribunal has addressed the scope of its jurisdiction to consider the Philippines’ claims both in its Award on Jurisdiction, to the extent that issues of jurisdiction could be decided as a preliminary matter, and in its Award of 12 July 2016, to the extent that issues of jurisdiction were intertwined with the merits of the Philippines’ claims. The Tribunal’s Award of 12 July 2016 also incorporates and reaffirms the decisions on jurisdiction taken in the Award on Jurisdiction. For completeness, the Tribunal’s decisions on jurisdiction in both awards are summarized here together. a. Preliminary Matters In its Award on Jurisdiction, the Tribunal considered a number of preliminary matters with respect to its jurisdiction. The Tribunal noted that both the Philippines and China are parties to the Convention and that the Convention does not permit a State to except itself generally from the mechanism for the resolution of disputes set out in the Convention. The Tribunal held that China’s non-participation does not deprive the Tribunal of jurisdiction and that the Tribunal had been properly constituted pursuant to the provisions of Annex VII to the Convention, which include a procedure to form a tribunal even in the absence of one party. Finally, the Tribunal rejected an argument set out in China’s Position Paper and held that the mere act of unilaterally initiating an arbitration cannot constitute an abuse of the Convention. b. Existence of a Dispute Concerning Interpretation and Application of the Convention In its Award on Jurisdiction, the Tribunal considered whether the Parties’ disputes concerned the interpretation or application of the Convention, which is a requirement for resort to the dispute settlement mechanisms of the Convention. The Tribunal rejected the argument set out in China’s Position Paper that the Parties’ dispute is actually about territorial sovereignty and therefore not a matter concerning the Convention. The Tribunal accepted that there is a dispute between the Parties concerning sovereignty over islands in the South China Sea, but held that the matters submitted to arbitration by the Philippines do not concern sovereignty. The Tribunal considered that it would not need to implicitly decide sovereignty to address the Philippines’ Submissions and that doing so would not advance the sovereignty claims of either Party to islands in the South China Sea. The Tribunal also rejected the argument set out in China’s Position Paper that the Parties’ dispute is actually about maritime boundary delimitation and therefore excluded from dispute settlement by Article 298 of the Convention and a declaration that China made on 25 August 2006 pursuant to that Article. The Tribunal noted that a dispute concerning whether a State has an entitlement to a maritime zone is a distinct matter from the delimitation of maritime zones in an area in which they overlap. The Tribunal noted that entitlements, together with a wide variety of other issues, are commonly considered in a boundary delimitation, but can also arise in other contexts. The Tribunal held that it does not follow that a dispute over each of these issues is necessarily a dispute over boundary delimitation.

Finally, the Tribunal held that each of the Philippines’ Submissions reflected a dispute concerning the Convention. In doing so, the Tribunal emphasized (a) that a dispute concerning the interaction between the Convention and other rights (including any Chinese “historic rights”) is a dispute concerning the Convention and (b) that where China has not clearly stated its position, the existence of a dispute may be inferred from the conduct of a State or from silence and is a matter to be determined objectively. c. Involvement of Indispensable Third-Parties In its Award on Jurisdiction, the Tribunal considered whether the absence from this arbitration of other States that have made claims to the islands of the South China Sea would be a bar to the Tribunal’s jurisdiction. The Tribunal noted that the rights of other States would not form “the very subject-matter of the decision,” the standard for a third-party to be indispensable. The Tribunal further noted that in December 2014, Viet Nam had submitted a statement to the Tribunal, in which Viet Nam asserted that it has “no doubt that the Tribunal has jurisdiction in these proceedings.” The Tribunal also noted that Viet Nam, Malaysia, and Indonesia had attended the hearing on jurisdiction as observers, without any State raising the argument that its participation was indispensable. In its Award of 12 July 2016, the Tribunal noted that it had received a communication from Malaysia on 23 June 2016, recalling Malaysia’s claims in the South China Sea. The Tribunal compared its decisions on the merits of the Philippines’ Submissions with the rights claimed by Malaysia and reaffirmed its decision that Malaysia is not an indispensable party and that Malaysia’s interests in the South China Sea do not prevent the Tribunal from addressing the Philippines’ Submissions. d. Preconditions to Jurisdiction In its Award on Jurisdiction, the Tribunal considered the applicability of Articles 281 and 282 of the Convention, which may prevent a State from making use of the mechanisms under the Convention if they have already agreed to another means of dispute resolution. The Tribunal rejected the argument set out in China’s Position Paper that the 2002 China–ASEAN Declaration on the Conduct of Parties in the South China Sea prevented the Philippines from initiating arbitration. The Tribunal held that the Declaration is a political agreement and not legally binding, does not provide a mechanism for binding settlement, does not exclude other means of dispute settlement, and therefore does not restrict the Tribunal’s jurisdiction under Articles 281 or 282. The Tribunal also considered the Treaty of Amity and Cooperation in Southeast Asia, and the Convention on Biological Diversity, and a series of joint statements issued by the Philippines and China referring to the resolution of disputes through negotiations and concluded that none of these instruments constitute an agreement that would prevent the Philippines from bringing its claims to arbitration. The Tribunal further held that the Parties had exchanged views regarding the settlement of their disputes, as required by Article 283 of the Convention, before the Philippines initiated the arbitration. The Tribunal concluded that this requirement was met in the record of diplomatic communications between the Philippines and China, in which the Philippines expressed a clear preference for multilateral negotiations involving the other States surrounding the South China Sea, while China insisted that only bilateral talks could be considered. e. Exceptions and Limitations to Jurisdiction

In its Award of 12 July 2016, the Tribunal considered whether the Philippines’ Submissions concerning Chinese historic rights and the ‘nine-dash line’ were affected by the exception from jurisdiction for disputes concerning “historic title” in Article 298 of the Convention. The Tribunal reviewed the meaning of “historic title” in the law of the sea and held that this refers to claims of historic sovereignty over bays and other near-shore waters. Reviewing China’s claims and conduct in the South China Sea, the Tribunal concluded that China claims historic rights to resources within the ‘nine-dash line’, but does not claim historic title over the waters of the South China Sea. Accordingly, the Tribunal concluded that it had jurisdiction to consider the Philippines’ claims concerning historic rights and, as between the Philippines and China, the ‘nine-dash line’. In its Award of 12 July 2016, the Tribunal also considered whether the Philippines’ Submissions were affected by the exception from jurisdiction in Article 298 for disputes concerning sea boundary delimitation. The Tribunal had already found in its Award on Jurisdiction that the Philippines’ Submissions do not concern boundary delimitation as such, but noted that several of the Philippines’ Submissions were dependent on certain areas forming part of the Philippines’ exclusive economic zone. The Tribunal held that it could only address such submissions if there was no possibility that China could have an entitlement to an exclusive economic zone overlapping that of the Philippines and deferred a final decision on its jurisdiction. In its Award of 12 July 2016, the Tribunal reviewed evidence about the reefs and islands claimed by China in the South China Sea and concluded that none is capable of generating an entitlement to an exclusive economic zone. Because China has no possible entitlement to an exclusive economic zone overlapping that of the Philippines in the Spratly Islands, the Tribunal held that the Philippines’ submissions were not dependent on a prior delimitation of a boundary. In its Award of 12 July 2016, the Tribunal also considered whether the Philippines’ Submissions were affected by the exception from jurisdiction in Article 298 for disputes concerning law enforcement activities in the exclusive economic zone. The Tribunal recalled that the exception in Article 298 would apply only if the Philippines’ Submissions related to law enforcement activities in China’s exclusive economic zone. Because, however, the Philippines’ Submissions related to events in the Philippines’ own exclusive economic zone or in the territorial sea, the Tribunal concluded that Article 298 did not pose an obstacle to its jurisdiction. Lastly, in its Award of 12 July 2016, the Tribunal considered whether the Philippines’ submissions were affected by the exception from jurisdiction in Article 298 for disputes concerning military activities. The Tribunal considered that the stand-off between Philippine marines on Second Thomas Shoal and Chinese naval and law enforcement vessels constituted military activities and concluded that it lacked jurisdiction over the Philippines’ Submission No. 14(a)-(c). The Tribunal also considered whether China’s land reclamation and construction of artificial islands at seven features in the Spratly Islands constituted military activities, but noted that China had repeatedly emphasized the non-military nature of its actions and had stated at the highest level that it would not militarize its presence in the Spratlys. The Tribunal decided that it would not deem activities to be military in nature when China itself had repeatedly affirmed the opposite. Accordingly, the Tribunal concluded that Article 298 did not pose an obstacle to its jurisdiction. 4. The Tribunal’s Decisions on the Merits of the Philippines’ Claims a. The ‘Nine-Dash Line’ and China’s Claim to Historic Rights in the Maritime Areas of the South China Sea

In its Award of 12 July 2016, the Tribunal considered the implications of China’s ‘nine-dash line’ and whether China has historic rights to resources in the South China Sea beyond the limits of the maritime zones that it is entitled to pursuant to the Convention. The Tribunal examined the history of the Convention and its provisions concerning maritime zones and concluded that the Convention was intended to comprehensively allocate the rights of States to maritime areas. The Tribunal noted that the question of pre-existing rights to resources (in particular fishing resources) was carefully considered during the negotiations on the creation of the exclusive economic zone and that a number of States wished to preserve historic fishing rights in the new zone. This position was rejected, however, and the final text of the Convention gives other States only a limited right of access to fisheries in the exclusive economic zone (in the event the coastal State cannot harvest the full allowable catch) and no rights to petroleum or mineral resources. The Tribunal found that China’s claim to historic rights to resources was incompatible with the detailed allocation of rights and maritime zones in the Convention and concluded that, to the extent China had historic rights to resources in the waters of the South China Sea, such rights were extinguished by the entry into force of the Convention to the extent they were incompatible with the Convention’s system of maritime zones. The Tribunal also examined the historical record to determine whether China actually had historic rights to resources in the South China Sea prior to the entry into force of the Convention. The Tribunal noted that there is evidence that Chinese navigators and fishermen, as well as those of other States, had historically made use of the islands in the South China Sea, although the Tribunal emphasized that it was not empowered to decide the question of sovereignty over the islands. However, the Tribunal considered that prior to the Convention, the waters of the South China Sea beyond the territorial sea were legally part of the high seas, in which vessels from any State could freely navigate and fish. Accordingly, the Tribunal concluded that historical navigation and fishing by China in the waters of the South China Sea represented the exercise of high seas freedoms, rather than a historic right, and that there was no evidence that China had historically exercised exclusive control over the waters of the South China Sea or prevented other States from exploiting their resources. Accordingly, the Tribunal concluded that, as between the Philippines and China, there was no legal basis for China to claim historic rights to resources, in excess of the rights provided for by the Convention, within the sea areas falling within the ‘nine-dash line’. b. The Status of Features in the South China Sea In its Award of 12 July 2016, the Tribunal considered the status of features in the South China Sea and the entitlements to maritime areas that China could potentially claim pursuant to the Convention. The Tribunal first undertook a technical evaluation as to whether certain coral reefs claimed by China are or are not above water at high tide. Under Articles 13 and 121 of the Convention, features that are above water at high tide generate an entitlement to at least a 12 nautical mile territorial sea, whereas features that are submerged at high tide generate no entitlement to maritime zones. The Tribunal noted that many of the reefs in the South China Sea have been heavily modified by recent land reclamation and construction and recalled that the Convention classifies features on the basis of their natural condition. The Tribunal appointed an expert hydrographer to assist it in evaluating the Philippines’ technical evidence and relied heavily on archival materials and historical hydrographic surveys in evaluating the features. The Tribunal agreed with the Philippines that Scarborough Shoal,

Johnson Reef, Cuarteron Reef, and Fiery Cross Reef are high-tide features and that Subi Reef, Hughes Reef, Mischief Reef, and Second Thomas Shoal were submerged at high tide in their natural condition. However, the Tribunal disagreed with the Philippines regarding the status of Gaven Reef (North) and McKennan Reef and concluded that both are high tide features. The Tribunal then considered whether any of the features claimed by China could generate an entitlement to maritime zones beyond 12 nautical miles. Under Article 121 of the Convention, islands generate an entitlement to an exclusive economic zone of 200 nautical miles and to a continental shelf, but “[r]ocks which cannot sustain human habitation or economic life of their own shall have no exclusive economic zone or continental shelf.” The Tribunal noted that this provision was closely linked to the expansion of coastal State jurisdiction with the creation of the exclusive economic zone and was intended to prevent insignificant features from generating large entitlements to maritime zones that would infringe on the entitlements of inhabited territory or on the high seas and the area of the seabed reserved for the common heritage of mankind. The Tribunal interpreted Article 121 and concluded that the entitlements of a feature depend on (a) the objective capacity of a feature, (b) in its natural condition, to sustain either (c) a stable community of people or (d) economic activity that is neither dependent on outside resources nor purely extractive in nature. The Tribunal noted that many of the features in the Spratly Islands are currently controlled by one or another of the littoral States, which have constructed installations and maintain personnel there. The Tribunal considered these modern presences to be dependent on outside resources and support and noted that many of the features have been modified to improve their habitability, including through land reclamation and the construction of infrastructure such as desalination plants. The Tribunal concluded that the current presence of official personnel on many of the features does not establish their capacity, in their natural condition, to sustain a stable community of people and considered that historical evidence of habitation or economic life was more relevant to the objective capacity of the features. Examining the historical record, the Tribunal noted that the Spratly Islands were historically used by small groups of fishermen from China, as well as other States, and that several Japanese fishing and guano mining enterprises were attempted in the 1920s and 1930s. The Tribunal concluded that temporary use of the features by fishermen did not amount to inhabitation by a stable community and that all of the historical economic activity had been extractive in nature. Accordingly, the Tribunal concluded that all of the high-tide features in the Spratly Islands (including, for example, Itu Aba, Thitu, West York Island, Spratly Island, North-East Cay, SouthWest Cay) are legally “rocks” that do not generate an exclusive economic zone or continental shelf. The Tribunal also held that the Convention does not provide for a group of islands such as the Spratly Islands to generate maritime zones collectively as a unit. c. Chinese Activities in the South China Sea In its Award of 12 July 2016, the Tribunal considered the lawfulness under the Convention of various Chinese actions in the South China Sea. Having found that Mischief Reef, Second Thomas Shoal and Reed Bank are submerged at high tide, form part of the exclusive economic zone and continental shelf of the Philippines, and are not overlapped by any possible entitlement of China, the Tribunal concluded that the Convention is clear in allocating sovereign rights to the Philippines with respect to sea areas in its exclusive economic zone. The Tribunal found as a matter of fact that China had (a) interfered with Philippine petroleum exploration at Reed Bank, (b) purported to prohibit fishing by Philippine vessels within the

Philippines’ exclusive economic zone, (c) protected and failed to prevent Chinese fishermen from fishing within the Philippines’ exclusive economic zone at Mischief Reef and Second Thomas Shoal, and (d) constructed installations and artificial islands at Mischief Reef without the authorization of the Philippines. The Tribunal therefore concluded that China had violated the Philippines’ sovereign rights with respect to its exclusive economic zone and continental shelf. The Tribunal next examined traditional fishing at Scarborough Shoal and concluded that fishermen from the Philippines, as well as fishermen from China and other countries, had long fished at the Shoal and had traditional fishing rights in the area. Because Scarborough Shoal is above water at high tide, it generates an entitlement to a territorial sea, its surrounding waters do not form part of the exclusive economic zone, and traditional fishing rights were not extinguished by the Convention. Although the Tribunal emphasized that it was not deciding sovereignty over Scarborough Shoal, it found that China had violated its duty to respect to the traditional fishing rights of Philippine fishermen by halting access to the Shoal after May 2012. The Tribunal noted, however, that it would reach the same conclusion with respect to the traditional fishing rights of Chinese fishermen if the Philippines were to prevent fishing by Chinese nationals at Scarborough Shoal. The Tribunal also considered the effect of China’s actions on the marine environment. In doing so, the Tribunal was assisted by three independent experts on coral reef biology who were appointed to assist it in evaluating the available scientific evidence and the Philippines’ expert reports. The Tribunal found that China’s recent large scale land reclamation and construction of artificial islands at seven features in the Spratly Islands has caused severe harm to the coral reef environment and that China has violated its obligation under Articles 192 and 194 of the Convention to preserve and protect the marine environment with respect to fragile ecosystems and the habitat of depleted, threatened, or endangered species. The Tribunal also found that Chinese fishermen have engaged in the harvesting of endangered sea turtles, coral, and giant clams on a substantial scale in the South China Sea, using methods that inflict severe damage on the coral reef environment. The Tribunal found that Chinese authorities were aware of these activities and failed to fulfill their due diligence obligations under the Convention to stop them. Finally, the Tribunal considered the lawfulness of the conduct of Chinese law enforcement vessels at Scarborough Shoal on two occasions in April and May 2012 when Chinese vessels had sought to physically obstruct Philippine vessels from approaching or gaining entrance to the Shoal. In doing so, the Tribunal was assisted by an independent expert on navigational safety who was appointed to assist it in reviewing the written reports provided by the officers of the Philippine vessels and the expert evidence on navigational safety provided by the Philippines. The Tribunal found that Chinese law enforcement vessels had repeatedly approached the Philippine vessels at high speed and sought to cross ahead of them at close distances, creating serious risk of collision and danger to Philippine ships and personnel. The Tribunal concluded that China had breached its obligations under the Convention on the International Regulations for Preventing Collisions at Sea, 1972, and Article 94 the Convention concerning maritime safety. d. Aggravation of the Dispute between the Parties In its Award of 12 July 2016, the Tribunal considered whether China’s recent large-scale land reclamation and construction of artificial islands at seven features in the Spratly Islands since the commencement of the arbitration had aggravated the dispute between the Parties. The Tribunal recalled that there exists a duty on parties engaged in a dispute settlement procedure to refrain from aggravating or extending the dispute or disputes at issue during the pendency of the settlement

process. The Tribunal noted that China has (a) built a large artificial island on Mischief Reef, a lowtide elevation located in the exclusive economic zone of the Philippines; (b) caused permanent, irreparable harm to the coral reef ecosystem and (c) permanently destroyed evidence of the natural condition of the features in question. The Tribunal concluded that China had violated its obligations to refrain from aggravating or extending the Parties’ disputes during the pendency of the settlement process. e. Future Conduct of the Parties Finally, the Tribunal considered the Philippines’ request for a declaration that, going forward, China shall respect the rights and freedoms of the Philippines and comply with its duties under the Convention. In this respect, the Tribunal noted that both the Philippines and China have repeatedly accepted that the Convention and general obligations of good faith define and regulate their conduct. The Tribunal considered that the root of the disputes at issue in this arbitration lies not in any intention on the part of China or the Philippines to infringe on the legal rights of the other, but rather in fundamentally different understandings of their respective rights under the Convention in the waters of the South China Sea. The Tribunal recalled that it is a fundamental principle of international law that bad faith is not presumed and noted that Article 11 of Annex VII provides that the “award . . . shall be complied with by the parties to the dispute.” The Tribunal therefore considered that no further declaration was necessary.

Francisco Chavez vs Public Estates Authority (July 2002) February 27, 2012 Share this...

4

0

0

0

ADVERTISEMENTS

384 SCRA 152 – Civil Law – Land Titles and Deeds – Lands of the Public Domain The Public Estates Authority (PEA) is the central implementing agency tasked to undertake reclamation projects nationwide. It took over the leasing and selling functions of the DENR (Department of Environmental and Natural Resources) insofar as reclaimed or about to be reclaimed foreshore lands are concerned. PEA sought the transfer to the Amari Coastal Bay and Development Corporation, a private corporation, of the ownership of 77.34 hectares of the Freedom Islands. PEA also sought to have 290.156 hectares of submerged areas of Manila Bay to Amari. ISSUE: Whether or not the transfer is valid. HELD: No. To allow vast areas of reclaimed lands of the public domain to be transferred to Amari as private lands will sanction a gross violation of the constitutional ban on private corporations from acquiring any kind of alienable land of the public domain. The Supreme Court affirmed that the 157.84 hectares of reclaimed lands comprising the Freedom Islands, now covered by certificates of title in the name of PEA, are alienable lands of the public domain. The 592.15 hectares of submerged areas of Manila Bay remain inalienable natural resources of the public domain. The transfer (as embodied in a joint venture agreement) to AMARI, a private corporation, ownership of 77.34 hectares of the Freedom Islands, is void for being contrary to Section 3, Article XII of the 1987 Constitution which prohibits private corporations from acquiring any kind of alienable land of the public domain. Furthermore, since the Amended JVA also seeks to transfer to Amari ownership of 290.156 hectares of still submerged areas of Manila Bay, such transfer is void for being

contrary to Section 2, Article XII of the 1987 Constitution which prohibits the alienation of natural resources other than agricultural lands of the public domain.

Chavez v. Pea and Amari Fact: In 1973, the Comissioner on Public Highways entered into a contract to reclaim areas of Manila Bay with the Construction and Development Corportion of the Philippines (CDCP). PEA (Public Estates Authority) was created by President Marcos under P.D. 1084, tasked with developing and leasing reclaimed lands. These lands were transferred to the care of PEA under P.D. 1085 as part of the Manila Cavite Road and Reclamation Project (MCRRP). CDCP and PEA entered into an agreement that all future projects under the MCRRP would be funded and owned by PEA. By 1988, President Aquino issued Special Patent No. 3517 transferring lands to PEA. It was followed by the transfer of three Titles (7309, 7311 and 7312) by the Register of Deeds of Paranaque to PEA covering the three reclaimed islands known as the FREEDOM ISLANDS. Subsquently, PEA entered into a joint venture agreement (JVA) with AMARI, a Thai-Philippine corporation to develop the Freedom Islands. Along with another 250 hectares, PEA and AMARI entered the JVA which would later transfer said lands to AMARI. This caused a stir especially when Sen. Maceda assailed the agreement, claiming that such lands were part of public domain (famously known as the “mother of all scams”). Peitioner Frank J. Chavez filed case as a taxpayer praying for mandamus, a writ of preliminary injunction and a TRO against the sale of reclaimed lands by PEA to AMARI and from implementing the JVA. Following these events, under President Estrada’s admin, PEA and AMARI entered into an Amended JVA and Mr. Chaves claim that the contract is null and void. Issue: w/n: the transfer to AMARI lands reclaimed or to be reclaimed as part of the stipulations in the (Amended) JVA between AMARI and PEA violate Sec. 3 Art. XII of the 1987 Constitution w/n: the court is the proper forum for raising the issue of whether the amended joint venture agreement is grossly disadvantageous to the government. Held: On the issue of Amended JVA as violating the constitution: 1. The 157.84 hectares of reclaimed lands comprising the Freedom Islands, now covered by certificates of title in the name of PEA, are alienable lands of the public domain. PEA may lease these lands to private corporations but may not sell or transfer ownership of these lands to private corporations. PEA may only sell these lands to Philippine citizens, subject to the ownership limitations in the 1987 Constitution and existing laws. 2. The 592.15 hectares of submerged areas of Manila Bay remain inalienable natural resources of the public domain until classified as alienable or disposable lands open to disposition and declared no longer needed for public service. The government can make such classification and declaration only after PEA has reclaimed these submerged areas. Only then can these lands qualify as agricultural lands of the public domain, which are the only natural resources the government can alienate. In their present state, the 592.15 hectares of submerged areas are inalienable and outside the commerce of man. 3. Since the Amended JVA seeks to transfer to AMARI, a private corporation, ownership of 77.34 hectares110 of the Freedom Islands, such transfer is void for being contrary to Section 3, Article XII of the 1987 Constitution which prohibits private corporations from acquiring any kind of alienable land of the public domain.

4. Since the Amended JVA also seeks to transfer to AMARI ownership of 290.156 hectares111 of still submerged areas of Manila Bay, such transfer is void for being contrary to Section 2, Article XII of the 1987 Constitution which prohibits the alienation of natural resources other than agricultural lands of the public domain. PEA may reclaim these submerged areas. Thereafter, the government can classify the reclaimed lands as alienable or disposable, and further declare them no longer needed for public service. Still, the transfer of such reclaimed alienable lands of the public domain to AMARI will be void in view of Section 3, Article XII of the 1987Constitution which prohibits private corporations from acquiring any kind of alienable land of the public domain

G.R. No. 201031 TOMAS R. LEONIDAS, Petitioner vs. TANCREDO VARGAS and REPUBLIC OF THE PIDLIPPINES, Respondents DECISION DEL CASTILLO, J.: Assailed in this Petition for Review on Certiorari1 are the August 13, 2009 Decision2 and February 22, 2012 Resolution3 of the Court of Appeals (CA) in CA-G.R. CV No. 02296, which affirmed with modification the March 19, 2007 Decision4 of the Regional Trial Court (RTC) of Barotac Viejo, Iloilo, Branch 66, in LRC Case No. 02-195. Factual Antecedents On February 2, 2002, Tomas R. Leonidas (herein petitioner) filed an application for land registration5 (Application) covering Lot 566 and Lot 1677 which are both situated in Concepcion, Iloilo (collectively, subject lots). Petitioner alleged that he inherited the subject lots from his parents, Ponciano Leonidas, Jr. (Ponciano) and Asuncion Roxas de Leonidas (Asuncion); that as evidenced by the 1-fay 17, 193 7 Certificate of Sale issued by the Provincial Treasurer of Iloilo, the subject lots, then covered by Tax Declaration (TD) No. 722, were purchased by Asuncion when auctioned due to delinquency in the payment of real property taxes by the original owners, the heirs of Inis Luching; that Asuncion immediately took possession of the subject lots and exercised dominical rights thereover notoriously, continuously, and exclusively; that upon Asuncion’s death in 1986, Ponciano succeeded to the ownership and possession of the subject lots; that after Ponciano’s death in l 991, the subject lots became his (petitioner’s) own exclusive property; that he permitted and tolerated the occupation of some portions of the subject lots by Juanito Tisolan, Pancing Guevarra, Carmencita Guevarra, Delia .Aspera-Ecleo, Victorino Mosqueda, Nora Binas, Crisanto Amangas (Amangas),6 Rosana Vasquez, Henry Asturias, Ronnie Astorias, Antonio Asturias, and Jacob Narciso; that as far as known to him (petitioner), the following are the owners of all adjoining properties, i.e.the owners of Lot 564, Lot 565 Lot 1578, and Lot 1677, Mansueto Sicad, Francisco Aspero, Brigido Celestial, and Eugenio Bondoc, Jr. who are all from Poblacion, Concepcion, Iloilo, and Carmen Paoli of unknown address; that Lot 566 is bounded on the west by the provincial road and the (petitioner) does not claim any portion thereof; that the latest assessed value of the subject lots is ₱52, 660.00 as certified by the Provincial Treasurer of Iloilo; that to the best of his knowledge and belief, there is no mortgage or encumbrance of any kind whatsoever affecting the subject lots except for taxes due thereon; that a certain Tomas Vargas (Tomas), however, had declared a portion of the subject lots in his name for taxation purposes; but that Tomas died shortly after the end of the Second World War, and the whereabouts of his heirs, if any, are unknown, despite his diligent search to locate them in Concepcion, Iloilo, and elsewhere. Petitioner also alleged that he was 77 years old, Filipino, a resident of No. 55 Chestnut St., West Fairview, Quezon City, and married to Ofelia Gustilo Leonidas (Ofelia); that attached to his Application were original Survey Plans with photographic copies each, the Tracing Cloth Plan land Management Service, Department of Environment and Natural Resources (DENR), Region VI, Iloilo

City, in lieu of the surveyor’s Certificate, Technical Description with three photographic copies, the Certificate in quadruplicate of the Provincial Treasurer showing the latest assessed value of the subject lots, and a copy of the muniment of title to prove ownership of the subject lots, with the original to be presented at the trial. Petitioner thus prayed that the subject lots be brought under the operation of the Property Registration Decree7 (PD 1529) and that the titles thereto be registered and confirmed in his name. The Republic of the Philippines (Republic), represented by the Office of the Solicitor General (OSG), opposed the said Application. The Republic claimed that neither the petitioner nor his predecessorsin-interest had been in continuous, exclusive, and notorious possession and occupation of the subject lots since June 12, 1945, or prior thereto, as required by Section 48 of Commonwealth Act (CA) No. 141, as amended by PD 1073; that the petitioner’s muniment/s of title, tax declarations, and tax payment receipts did not constitute competent and sufficient evidence of either a bona fide acquisition of the subject lots, and neither did the petitioner's bare claim of open, continuous, exclusive, and notorious possession and occupation thereof in the concept of owner since June 12, 1945, or prior thereto, amount to convincing proof of his claim of possession and ownership over the subject lots; that, although the petitioner's muniments of title might appear genuine, the tax declarations and/or tax payments showing the pretended possession were, in fact, of recent vintage; that the claim of ownership in fee simple on the basis of a Spanish title or grant could no longer be availed of by petitioner who had failed to file an appropriate application therefor within the period of six months from Feb1uary 16, 1976, as required by PD 892; and that the subject lots are portions of the public domain belonging to the Republic which are not subject to private appropriation. Thus, the Republic prayed that the petitioner's Application be denied and that the subject lots be declared part of the public domain. On March 11, 2003, Tancredo Vargas (Tancredo) also filed an Opposition8 to the Application. Tancredo averred that he is Tomas' legitimate son and compulsory heir; that during Tomas's lifetime, the latter was the absolute and exclusive owner of a certain parcel of land located at Loong, Concepcion, Iloilo, which parcel of land is bounded on the north by the seashore, on the south by Severino Asturias (Asturias),9 on the east by the seashore, and on the west by Asturias and Braulio Celestial; that this parcel of land had an area of 36,237 square meters and was covered by TD No. 3549 in Tomas's name; that the petitioner does not exclusively own Lot 1677 since it had been split into two, viz. Lot 1677-A and Lot 1677-B; that he (Tancredo) is the owner of Lot 1677-A; that Lot 566 was also not exclusively owned by the petitioner, as this Lot 566 had also been divided into two lots, viz. Lot 566-A and Lot 566-B; that he (Tancredo) is the owner of Lot 566-A as shown in the RPTA Tax Mapping project in the Municipal of Concepcion, Iloilo; that the petitioner's allegation that the owners of the property covered by TD 772 became delinquent in the payment of the tax due thereon, for which reason the Provincial Treasurer of Iloilo allegedly sold the same to Asuncion, was not at all true; that the property covered by TD 772 was not sold at public auction because the forfeiture was lifted prior to the public auction sale; and that the fact that the Office of the Provincial Treasurer of Iloilo did not have a copy of the Certificate of Sale dated May 17, 1937 bolstered the argument that petitioner’s allegation is questionable. Tancredo thus prayed that the petitioner's Application be denied insofar as the portions covered by the TDs in the name of Tomas (disputed portions) are concerned. On March 21, 2003, another Opposition10 to the Application was filed by Moncerat A. Sicad-De Julian, Gil A. Sicad, represented by his wife, Elizabeth Sicad, Teresita A. Sicad-Bayuran, Villaluz Sicad-Zarriz, Eden A. Sicad, and Melchor Sicad, represented by his wife, Elena D. Sicad, (Elena; collectively, the Sicads) all represented by their attorney-in-fact, Elena.11 These oppositors claimed that they are the heirs of the late Mansueto Sicad (Mansueto) who was the owner of a portion of the subject lots (Sicads’s contested portion); that the Sicads's contested portion was bought by Mansueto from Asturias as evidenced by the Deed of Definite Sale of a Parcel of Land described as

Doc. No. 75, Page No. 35, Book No. 1, Series of 1950 of the notarial register of notary public Crespo Celestial; that the Sicads's contested portion had been in the possession of Mansueto during the latter's lifetime; that they had been in possession of the Sicads’s contested portion since Mansueto's death; that part of the Sicads’s contested portion had already been registered under Original Certificate of Title (OCT) No. F-36795; and that the petitioner had never been in possession of the lots subject of his Application. The Sicads thus prayed that the petitioner's Application be dismissed, insofar as it concerned the Sicads’s contested portion as set forth in the aforesaid Deed of Definite Sale; and that the Sicads’s contested portion be registered instead in their names. At the trial, the petitioner presented himself and Geronimo C. Pefiaflorida (Peñaflorida), Land Management Inspector, DENR, Community Environment and Natural Resources Office (CENRO), at Sara, lloilo as witnesses.12On the other hand, Catalino Guinez, Emeliana Isturias Matulac, and Elena testified for the Sicads.13 For his part, Tancredo presented himself and a forn1er overseer or tenant of the Vargas family,14 Jose Etchona (Etchona).15 Then on August 8, 2003, the petitioner filed his Formal Offer of Evidence16 wherein he submitted the Certificate of Sale dated May 17, 1937, TD 014134 for the year 1976 in Asuncion's name and covering Cadastral Lot Nos. 1, 2, and 3 PSU-216090, TD 0037 for the year 1994 in the names of Asuncion and Ponciano and covering Cadastral Lot No. 1677, TD 0036 for the year 1994 in the names of Asuncion and Ponciano and covering Cadastral Lot No. 566, TD 0114_ for the year 2003 in the names of Asuncion and Ponciano and covering CadastraJ Lot No. 1677-A, TD 0118 for the year 2003 in the names of Asuncion and Ponciano and covering Cadastral Lot No. 1677-B, TD 0116 for the year 2003 in the names of Asuncion and Ponciano and covering Cadastral Lot No. 566-A; and TD 0117 for the year 2003 in the names of Asuncion and Ponciano and covering Cadastral Lot No. 566-B,17 tax receipts for the years 1986, 1987, 1988, 1989, 1990, 1991, 1994, 2002 and 2003, statement of the assessed value issued by the Provincial Assessor of Iloilo on March 26, 1996, Lot No. 566's Blue Print Survey Plan with technical description, Lot l 677’s Blue Print Survey Plan with technical description, Certificate of Unavailability of Surveyor's Certificate of Survey for Lots 566 and 1677, and Survey Inspection Report dated August 28, 1997 for Lot Nos. 566 and 1677 issued by Peñaflorida,18 i.e. CENRO Report dated August 28, 1997, to the effect that the subject lots are free from liens and encumbrances, and are moreover within the alienable and disposable area. Pursuant to the RTC's directive, petitioner also offered as additional evidence the originally-approved subdivision plan covering Lot No. 1677, Csd-06-008798 to prove the identity and location of the easement for public use;19 and a certification by Joel B. Diaz, CENRO at Sara, Iloilo, to the effect that Lot No. 1677, Pls 1099, situated in Brgy; Loong, Concepcion, Iloilo, with an area of 8,062 square meters was issued Patent No. 063015-92-846 dated May 28, 1992 in the name ofFlordeluz Sedigo, but that Lot No. 1677 has doubled with the lot situated at Poblacion, Concepcion, Iloilo in the name of the Heirs of Ponciano and that this latter lot is not covered by any public land application filed with the CENRO in Sara, Iloilo, which explained why no patent has been issued therefor, hence indicating that this other Lot No. 1677, Pls 1099, which is situated in Brgy. Aglusong, Concepcion, Iloilo is entirely different from Lot No. 1677, which is situated in Sitio Loong, Poblacion, Concepcion, Iloilo.20 The petitioner likewise submitted in evidence an Ocular Inspection Report covering an ocular inspection earlier ordered by the RTC.21 Ruling of the Regional Trial Court In its Decision dated March 19, 2007, the RTC disposed of this case in this wise: WHEREFORE, general default having been declared and the [A]pplication supported by evidence, the adjudication and registration of portion of Lot No. 566 with an area of 3.1161 hectares and

portion of Lot 1677 with an area of 3. 7255 hectares, all of Concepcion Cadastre, together with all the improvements thereon are hereby ordered in favor of applicant [petitioner], of legal age, married to [Ofelia], Filipino, and resident of Fairview, Quezon City, Philippines. Portions of Lot [No.] 1677 with an area of 2.3642 hectares and portion of Lot [No.] 566 with an area of 1.1782 hectares are hereby adjudicated in favor of [Tancredo ], of legal age, single, Filipino, and resident of Lawa-an Village, Balantang, Jaro, Iloilo City, Philippines which portions shall be segregated in a proper subdivision survey and to follow the description of the plan of Municipal Assessor of Concepcion, Iloilo commensurate to Lot 1677-A under [T.D.] No. 054822 and 566-A under [T.D.] No. 0550. The easement of right of way of the lots, highways, streets, alleys, shorelines and other portion[s] of land not specified as lots located within the borders of the land covered by this case are declared to be the properties of the [Republic]. The Clerk of Court is directed to forward copies of this decision to all government agencies concerned. And finally, the Administrator, Land Registration Authority, is hereby directed, after this decision shall have become final for which he shall be duly advised by specific order of this Court, to issue [a] decree of registration and title in accordance with the amended plan on file in the record. SO ORDERED.23 The RTC held that petitioner had sufficiently established that his predecessors-in-interest had possessed and owned a parcel of land in Barangay Loong, Concepcion, Iloilo to the extent not covered by Tancredo’s Opposition; that while petitioner and his predecessors-in-interest might not have been in actual possession of the subject lots at all time, they nonetheless had been consistently visiting the same; and that petitioner’s claim of possession and ownership is supported by documents consisting of the Certificate of Sale issued by the Provincial Treasurer of Iloilo on May 17, 1937, the tax declarations in Asuncion's name for the years 1976, 1994, and 2003, the official receipts showing payments of real estate taxes thereon, and the statement of the assessed value issued by the Provincial Assessor of Iloilo on May 26, 1996. The RTC stressed that the period of possession by petitioner and his predecessors-in-interest sufficed to confer a registrable title upon petitioner. The RTC likewise ruled that Tancredo was also able to establish a superior claim with respect to his disputed portions; that all of the tax declarations in Asuncion's name continuously bore the annotation acknowledging Tomas's adverse claim relative to Tancredo's disputed portions; that Tomas's open and continuous possession for more than the required number of years was sufficiently shown by a tax declaration issued as early as the year 1945; that the overseers and other persons authorized to manage Tancredo's disputed portions were never driven out by petitioner; and that Tancredo had visited the disputed portions more frequently than petitioner who, as the evidence shows, has his permanent residence in Quezon City, Metro Manila. With regard to the claim of the Sicads, the RTC held that Mansueto and his successors-in-interest had no more interest in the Sicads' contested portion because what was shown to have been sold by Asturias to Mansueto pertained to a lot measuring only two hectares, 52 acres, and 92 ares, a parcel of land at par with the land covered by the aforementioned free patent issued to Mansueto. The RTC emphasized that it is well-entrenched in jurisprudence that alienable public land openly, continuously, and exclusively possessed by a person personally or through his predecessors-ininterest for at least 30 years becomes ipsojure private property by mere lapse of time, or by

completion of said period pursuant to Section 48(b) of CA 141, as amended by RA 1942 and RA 3872. Ruling of the Court of Appeals Only the petitioner and the Republic filed their respective Notices of Appeal24 which were given due course by the RTC in its Order of May 25, 2007.25 These notices of appeal were consolidated and docketed as CA-G.R. CV No. 02296. In a Decision dated August 13, 2009, the CA disposed as follows: WHEREFORE, the Decision dated March 19, 2007 is modified, as follows: 1.) the portion pertaining to the award of [Lot No.] 566 with an area of3.1161 hectares and [Lot No.] 1677 with an area of 3.7255 hectares to [petitioner], is REVERSED and SET ASIDE; and 2.) the portion pertaining to the award of [Lot No.] 1677 with an area of 2.3642 hectares and [Lot No.] 566 with an area of 1.1782 hectares in favor of [Tancredo] is AFFIRMED. SO ORDERED.26 The CA held that, contrary to the Republic's stance, the records showed that there had been compliance with the jurisdictional requirements of publication, posting, and notice; that petitioner had properly identified the subject lots; that the subject lots had already been classified as alienable and disposable at the time that petitioner filed the Application in 2002, pursuant to the CENRO Report dated August 28, 1997 issued by Peñaflorida; that it has been held that "[a] certification by the CENRO of the DENR stating that the subject lots are found to be within the alienable and disposable site per land classification project map is sufficient evidence to show the real character of the land subject of the application;"27 that these notwithstanding, petitioner failed to prove with the requisite evidence the kind of possession and the length of time required by law for the registration of the subject lots in his name, because his lone testimony did not suffice to establish his and his predecessors-in-interest’s alleged open, continuous, exclusive, and notorious possession over the subject lots since June 12, 1945, or earlier; that petitioner's alleged acts of swimming in, and planting trees on the subject lots, his having finished high school at the Victorino Salcedo High School in the neighboring town of Sara, Iloilo, and his having left the subject lots when he attended college - all these neither added up nor supported his assertion of dominion or ownership· over the subject lots; that his allegation that his childhood memories regarding the subject lots all came back to him after the death of his father Ponciano was indicative of the fact that he was really unaware of the existence of the subject lots; that his Application was even opposed by Tancredo and by the Sicads who claimed exclusive possession over certain portions of the subject lots; that petitioner's failure to explain why he or his predecessors-in-interest declared the subject lots for taxation purposes only in 1976, was inconsistent with his claim of possession thereover since 1937; and that it is an axiom of the law that the burden of proof in a land registration case rests upon the applicant who must present clear, positive, and convincing evidence establishing the alleged possession and occupation in good faith, and for the period required by law. On the other hand, the CA ruled that Tancredo had sufficiently proven his open, continuous, exclusive, and notorious possession and occupation for the period required by law, over the portions of the subject lots he was claiming in the concept of an owner; that Tomas's adverse claims were annotated on the TDs issued in Asuncion’s name covering the disputed portions, i.e. TD 014134, 0114, and 0117 ;28 that Tomas declared the disputed portions for taxation purposes in his name as early as 1945; that Tancredo himself testified that Tomas first used the disputed portions as rice land and converted the same into coconut land in the 1960s; that Tancredo’s witness, Etchona, likewise testified that Tomas employed him and Domingo Celestial not only to cultivate, but also to guard the disputed portions, and that Tomas himself appropriated the harvest from the disputed portions and

introduced improvements thereon; and that even petitioner himself admitted in his Application that Tomas had declared the disputed portions in his (Tomas’) name for taxation purposes. Petitioner moved for reconsideration29 but was denied by the CA in its Resolution of February 22, 2012.30 Issue Before this Court, petitioner now raises the following issue: [Whether] the [CA] gravely abused its discretion in denying the registration of [his] already vested title [over] Lot [Nos.] 566 and 1677 of the Concepcion, Iloilo Cadastre as his private property, and in awarding some portions thereof in favor of [Tancredo] in this land registration proceeding.31 Petitioner’s arguments Petitioner insists in his Petition,32 Consolidated Reply,33 and Memorandum34 that the CA erred in finding that he failed to prove that he and his predecessors-in-interest had been in open, continuous, exclusive, and notorious possession and occupation of the subject lots since June 12, 1945, or earlier, and that there is indubitable evidence that the subject lots were in fact sold in a tax sale on May 1 7, 1937 by the gove1nment through the Provincial Treasurer of Iloilo; that he filed the present Application so that an OCT can be issued in his name as evidence of his vested title over the subject lots; that assuming that the subject lots are still part of the public domain, he is nevertheless still entitled to have the subject lots registered in his name by reason of his and his predecessors-ininterest's exclusive possession and occupation thereof for more than 30 years, as compared to Tancredo's possession which supposedly began only in 1945; that under the Land Registration Act, as amended, the possessor is deemed to have acquired by operation of law the right to a government grant upon compliance with the conditions therefor, which was just what he did in this case; that the confirmation proceeding is a mere formality and the registration thereunder does not confer title but merely recognizes a title that is already vested; that rejection of his vested title to the questioned lots will occasion loss of confidence in the government's sales of forfeited property by reason of tax delinquency; that the CA erred in finding that the TDs in Asuncion's name carried Tomas's adverse claim, as the attached copies thereof did not bear any such annotations; that the CA also erred in stating that petitioner did not present any TDs to support his claim of ownership over the subject lots for the reason that the CA Decision itself mentioned that he submitted a TD for the year 1976; that contrary to the CA's findings, he did testify that he had visited the subject lots every so often to plant trees after he and his parents left Concepcion in 1945, and that such improvements were reflected in his exhibits; that the CA likewise erred in holding that he only came to know about the subject lots after the death of his father, Ponciano, for the fact is that he did testify that he and his cousins used to swim in the sea near the subject lots, as early as when he was 12 years old; that the CA moreover erred in concluding that Tancredo had successfully established his claims over the disputed portions of the subject lots because the TDs in Asuncion's name are all annotated with Tomas’s adverse claim, and that Tomas had declared said disputed portions in his name as early as 1945; that the tax declarations supposedly in Tomas's name were neither presented nor offered in evidence; that Tancredo admitted during his cross-examination that Tomas’s 1945 tax declaration was procured notwithstanding the fact that the subject lots had already been declared in Asuncion’s name; that Tancredo did not comply with the pertinent provisions of the Land Registration Act, as amended, because he did not present evidence to prove the specific date in 1945 when Tomas acquired the disputed portions, or how Tomas in fact acquired the same; that besides these, Tancredo could not identify the disputed portions that he was claiming; that if Tancredo wanted to vindicate his claims of ownership over the disputed portions, then Tancredo should institute the proper action before a court of general jurisdiction, and not in the land

registration court, as the subject lots were no longer part of the public domain; that the issue of whether the sale by the government to Asuncion on May 17, 1937 changed the classification of the subject lots from public to private is of first impression and should be resolved by the Supreme Court EnBanc; and that the circumstances obtaining in this case are exceptions to the rule that only questions of law are allowed in a petition filed pursuant to Rule 45 of the Revised Rules of Court; and that to deny his Application, or to render judgment ordering the reversion to public ownership of the subject lots would amount to grave abuse on the part of the judiciary. The Republic’s Arguments In its Comment35 and Memorandum,36 the Republic counters that the instant Petition merely raises questions of fact which are proscribed under Rule 45 of the Revised Rules of Court; that this Court is not a trier of facts; that petitioner's case does not fall under any of the exceptions to the rule that factual findings of the CA are invariably binding upon the Supreme Court; and that the assailed CA Decision should not be disturbed because the CA had amply justified the reversal of the RTC Decision which was erected upon the petitioner's failure to substantiate his claim of ownership over the subject lots. Tancredo’s Arguments In his Comment37 and Memorandum,38 Tancredo maintains that the disputed portions had been in the absolute possession and dominion of Tomas; that the findings of the RTC and the CA regarding petitioner's ineligibility to obtain title to the disputed portions due to non-compliance with the requirements of the law, and for insufficiency of evidence, should not be disturbed; that the CA’s finding that petitioner's TDs bore the annotated claims of Tomas on the subject lots is a factual finding and should not be disturbed; that petitioner's possession is not the possession required by law for purposes of land registration because petitioner failed to present evidence that would prove actual, notorious, continuous, and exclusive possession and occupation of the subject lots; that the evidence adduced by petitioner is self-serving, hence undeserving of any weight; that the origin of the disputed portions as pointed out by the RTC is Assessor's Lot No. 337, which is individually identified after the Cadastral Survey as Lot Nos. 1676- A, 1677-A, and 566-A, all of the Concepcion (Iloilo) Cadastre; that petitioner is barred or estopped from questioning the identity of the disputed portions that had been adjudicated to him (Tancredo ), as the lack of sufficient identification pertained to the subject lots that petitioner himself was trying to register; and that the issues raised by petitioner were factual in nature, and the same is proscribed under Rule 45 of the Revised Rules of Court. The fundamental issues to be resolved in this case are: (1) Whether the petitioner is entitled to obtain a title over the subject lots; and (2) Wether Tancredo has established, by his own evidence, that he was qualified to acquire title over the disputed portions claimed by him. The Court’s Ruling The Petition is denied. Requisites for the confirmation and registration of an imperfect and incomplete title under CA 141 and PD 1529 "The Regalian doctrine, embodied in Section 2, Article XII of the 1987 Constitution, provides that all lands of the public domain belong to the State, which is the source of any asserted right to

ownership of land."39 "[Commonwealth Act No. 141, in turn,] goven1s the classification and disposition of lands of the public domain. Section 11 [thereof] provides, as one of the modes of disposing public lands that are suitable for agriculture, the 'confirmation of imperfect or incomplete titles.’ Section 48 [thereof], on the other hand, enumerates those who are considered to have acquired an imperfect or incomplete title over public lands and, therefore, entitled to confirmation and registration under the Land Registration Act [now PD 1529]."40 The latter law then "specifies who are qualified to apply for registration of land."41 Taken together, all the foregoing provide for the requisites for the confirmation and registration of an imperfect and incomplete title, thus x x x In particular, Section 14 (l) [of PD 1529] in relation to Section 48 (b) of[CA] 141, as amended by Section4 of P.D. No. 1073, states: SEC. 14. Who may apply. -The following persons may file in the proper Court of First Instance [now Regional Trial Court] an application for registration of title to land, whether personally or through their duly authorized representatives: (1) Those who by themselves or through their predecessors-in-interest have been in open, continuous, exclusive and notorious possession and occupation of alienable and disposable lands of the public domain under a bonafide claim of ownership since June 12, 1945, or earlier. xxx

xxx

xxx

Section 48. The following described citizens of the Philippines, occupying lands of the public domain or claiming to own any such lands or an interest therein, but whose titles have not been perfected or completed, may apply to the Court of First Instance [now Regional Trial Court] of the province where the land is located for confirmation of their claims and the issuance of a certificate of title therefor, under [PD 1529], to wit: xxx

xxx

xxx

(b) Those who by themselves or through their predecessors-in-interest have been in open, continuous, exclusive and notorious possession and occupation of [alienable and disposable lands] of the public domain, under a bonafide claim of acquisition of ownership, since June 12, 1945, or earlier, immediately preceding the filing of the application for confirmation of title except when prevented by war or force majeure. These shall be conclusively presumed to have performed all the conditions essential to a Government grant and shall be entitled to a certificate of title under the provisions of this chapter. Based on these legal parameters, applicants for registration of title under Section 14 (1) must sufficiently establish: (1) that the subject land forms part of the disposable and alienable lands of the public domain; (2) that the applicant and his predecessors-in-interest have been in open, continuous, exclusive and notorious possession and occupation of the same; and (3) that his possession has been under a bonafide claim of ownership since June 12, 1945, or earlier. These triple requirements of alienability and possession and occupation since June 12, 1945 or earlier under Section 14 (1) are indispensable prerequisites to a favorable registration of title to the property. Each element must necessarily be proven by no less than clear, positive and convincing evidence; otherwise, the application for registration should be denied.42 Petitioner did not cite the specific provision of CA 141 upon which he based his Application. Neve1theless, the allegations therein seem to establish the fact that his claim is one of imperfect title under the above- quoted Section 48(b) of CA 141 in relation to Section 14(1) of PD 1529

The subject lots are considered alienable and disposable lands of the public domain The first· requirement is complied with in the case at bench. Notwithstanding that only a CENRO certification covering the subject lots was presented in the instant case, the subject lots are considered alienable and disposable lands of the public domain because of this Court’s ruling that an application for land registration may be granted despite the absence of the DENR Secretary's certification, provided that the same was pending at the time Republic v. Vega43 was promulgated on January 17, 2011. In Republic v. Alora,44 this Court expressly clarified this matter in this wise: x x x [I]n Republic v. T.A.N Properties, Inc., which was promulgated on 26 June 2008 x x x we held that applicants for land registration must present a copy of the original classification approved by the DENR Secretary and certified as true copy by the legal custodian of the official records. x x x x x x In Republic v. Serrano [(decided on 24 February 2010)], we allowed the approval of a land registration application even without the submission of the certification from the DENR Secretary. As this ruling presented an apparent contradiction with our earlier pronouncement in Republic v. T.A.N Properties, Inc., we sought to harmonize our previous rulings in Republic v. vega [(decided on 17 January 201 l)]. We then said that the applications for land registration may be granted even without the DENR Secretary's certification provided that the application was currently pending at the time Republic v. vega was promulgated. x x x45 It is worth stressing, however, that the foregoing ruling is the exception, not the rule. As explicitly elucidated in Republic v. Vega:46 It must be emphasized that the present ruling on substantial compliance applies pro hac vice. It does not in any way detract from our rulings in Republic v T.A.N Properties, Inc., and similar cases which impose a strict requirement to prove that the public land is alienable and disposable, especially in this case when the Decisions of the lower court and the [CA] were rendered prior to these rulings. To establish that the land subject of the application is alienable and disposable public land, the general rule remains: all applications for original registration under [PD 1529] must include both (1) a CENRO or PENRO certification and (2) a certified true of the original classification made by the DENR Secretary. As an exception, however, the courts - in their sound discretion and based solely on the evidence presented on record - may approve the application, pro hac vice, on the ground of substantial compliance showing that there has been a positive act of government to show the nature and character of the land and an absence of effective opposition from the government. This exception shall only apply to applications for registration currently pending before the trial court prior to this Decision and shall be inapplicable to all future applications. (Underscoring and emphases in the original)47 That said, we hold that both the petitioner and Tancredo failed to establish clearly and convincingly their respective rights to registration of imperfect titles under CA 141 and PD 1529, as will be discussed below. Petitioner failed to prove possession of the subject lots in the manner and for the period required by law

First off, petitioner failed to establish bonafide possession and ownership over the subject lots since June 12, 1945 or earlier. His contention that his predecessors-in-interest became the owners of the subject lots pursuant to the May 17, 1937 Certificate of Sale48 of the Forfeited Real Property issued by the Provincial Treasurer of Iloilo appears to be consistent with the fact that TD 3549 in Tomas’s name which was found by the CA as issued in 1945 bears an annotation stating that such is "[ c ]ontested by [Asuncion]".49 Even then, the Certificate of Public Sale indicated that the balance of the purchase price in the amount of ₱29.44, was yet to be paid on or before December 31, 1937.50 No incontrovertible proof was, however, presented to establish the fact that this balance of the purchase price in the said amount of ₱29 .44 had indeed been paid on or before December 31, 193 7. In addition, the CA also correctly pointed out that even as petitioner was able to submit TDs and evidence of tax payments only for a few years, he nevertheless failed to explain why he or his predecessors-in-interest declared the subject lots for taxation purposes only in 1976, this despite his claim that his predecessors-in-interest had been in possession and occupation of the subject lots since 1937, as allegedly shown in the Provincial Treasurer’s Certificate of Sale. It is settled that intermittent and irregular tax payments run counter to a claim of ownership or possession.51 Second, even assuming for argument's sake that petitioner’s predecessors-in-interest had paid the balance of the delinquent tax payment, petitioner nonetheless failed to prove his and his predecessors-in-interests actual, notorious, exclusive and continuous possession of the subject lots for the length of time required by law. To be sure, petitioner's failure to explain what happened after his family supposedly left the subject lots in 1941, when the war broke out, vis-a-vis his failure to prove that he had indeed introduced valuable improvements in the subject lots during the time that he and his parents had been allegedly in actual possession and occupation thereof, cast doubts upon his claim of actual possession and occupation thereof. Withal, petitioner's testimony of having swum near the subject lots, of having planted trees thereon, and his having finished high school at the Victorino Salcedo High School in the neighboring town of Sara can hardly be considered as acts of dominion or ownership over the subject lots. Besides, petitioner did not present clear and convincing evidence that the subject lots had indeed been cultivated by him or by his predecessors-in-interest for the period of time required by law. Needless to say, all these failings weaken his claim that he has been a bonafide possessor and occupant of the subject lots in the manner and for the period prescribed by law, to wit: The possession contemplated by Section 48 (b) of [CA] 141 is actual, not fictional or constructive. In Carlos v. Republic of the Philippines, the Court explained the character of the required possession, as follows: The law speaks of possession and occupation. Since these words are separated by the conjunction and, the clear intention of the law is not to make one synonymous with the other. Possession is broader than occupation because it includes constructive possession. When, therefore, the law adds the word occupation, it seeks to delimit the all-encompassing effect of constructive possession, Taken together with the words open, continuous, exclusive and notorious, the word occupation serves to highlight the fact ti1at for an applicant to qualify, his possession must not be a mere fiction. Actual possession of a land consists in the manifestation of acts of dominion over it of such a nature as a party would naturally exercise over his own property.52 (Emphases in the original) Oddly enough, while in its Decision the RTC appeared to have granted petitioner’s Application, said Decision seemed to have indulged in a bit of non-sequitur when it said that "[petitioner] and his predecessors were not in actual possession of the [subject Jots] all the time" x x x.53 Simply said, the

-A effectively ruled that since petitioner failed to prove that he or his predecessors-in-interest had indeed performed the required acts of possession and occupation, or specific acts of dominion over the subject lots, it stands to reason that registration thereof in his name cannot be allowed. Tancredo also failed to establish possession and occupation over the disputed portions in the manner and for the period required by law At this juncture, we shall revisit the uniform finding by both the RTC and the CA, which in effect upheld Tancredo's right to register the disputed portions in his name (as an exception to the settled rule that questions of fact are proscribed in a Rule 45 petition since a correct evaluation of the facts will yield a different conclusion).54 First off, Tancredo failed to show that his or his predecessor-ininterest’s possession and occupation over the disputed portions had been under a bonafide claim of ownership since June 12, 1945, or earlier. We are inclined to agree with petitioner's posture that Tancredo failed to adduce clear and convincing evidence which established the origin or antecedents of Tomas's straightforward possession and occupation, or claim of ownership, over the disputed portions. Consider the following exchange/s between/among Tancredo, the petitioner, and the Court [Petitioner]: (to the witness[, Tancredo]) Q: When did your father acquire this property? A: In 1945. Q: From whom? A: I have no idea. xxxx Q: Did you not ask your father from whom he acquired this property? A: No, I did not. Q: As a matter of fact[,] until the death of your father[,] you have not ask[ed] him from whom did he acquire the property? A: No, Sir. xxxx COURT: (to the witness[, Tancredo]) Q: Your father died in 1995 [,] why did you not [cause] the transfer of tax declaration in your name or to the heirs?

A: Because the plan of the heirs is, if the property [is registered] in my father[']s name [then] the title should be transferred in my name. xxxx Q: Your tax receipts correspond only [to] the year 2003, how about other tax receipts? A: I [will just [try] to find out if the Provincial Treasurer’s Office still has the copy. Q: Even just a certification stating that you [continued] in paying realty tax from 1946 up to 2003? A: Yes, I can ask the provincial treasurer for that matter. Q: When you secure[d] the tax declaration[,] you [knew] that the lot was also declared in the name of [Asuncion], is it not? A: Yes, Your Honor. Q: That was in the office of the Municipal Assessor? A: Yes, Your Honor. Q: Did you verify if they were paying taxes also? A: No, Your Honor. Q: You did not? A: I [did] not[,] Your Honor. Q: If that is the case[,] why did you [say] a while ago that you [knew]. only [about] the case of [petitioner] when this case was filed because the tax declaration itself [stated] that the lot was also declared in the name of [Asuncion]? A: Although I have already seen the notation on the tax declaration that they also [secured a] tax declaration [over] the [disputed Portions]. I did not mind it Your Honor because they did not openly claim ownership over the [disputed portions]. And in the same manner[.] Your Honor[,] in their tax declaration it is also indicated that the [disputed portions] is also declare[d] in the name of [Tomas].55 More than this, Tancredo did not present clear, convincing evidence to support his claim that the disputed portions were in fact transferred to him by his father, Tomas. Tancredo merely testified that the disputed portions were given to him solely by Tomas, an act that was allegedly consented to by his siblings. Thus[Petitioner]: (to the witness, Tancredo) Q: You have siblings, meaning brothers and sisters? A: Yes, Sir.

Q: You said a while ago that you succeeded to the ownership of the [subject lots] when your father died in 1985, how about your siblings[?] [Did they] not succeed to the [ownership of the subject lots?] A: They sign[ed] a deed of adjudication in favor of me[.] I have a copy and it was notarized. xxxx Q: In your [O]pposition you said that you were authorized? A: Yes, Sir. Q: By whom? A: By my brothers and sisters. Q: Where is your authority? A: I can produce it. I can pass [sic] it anytime. Q: You did not [s]tate in your [O]pposition that you have your siblings with you? A: Because the property was given to me by my father. 56 Nonetheless, there is nothing in the records to support or confirm Tancredo’s claim that the property was in fact deeded over to him by his father, Tomas. In Buenaventura v. Pascual,57 this Court affirmed the lower courts' dismissal of the claims for registration of imperfect titles because, among others, both the applicant and oppositors failed to adduce evidence as to how they acquired the subject property from their respective predecessors-ininterest, i.e., whether by succession or by donation or by some other mode. Furthermore, we stressed therein that the applicant failed to prove the manner by which her predecessors-in-interest possessed the subject property. Then, again, Tancredo also failed to establish that he and his predecessors-in-interest had/have been in open, continuous, exclusive and notorious possession and occupation of the disputed portions since June 12, 1945, or prior thereto. If anything, the records showed that Tancredo merely submitted photocopies of four tax declarations which were attached as annexes to his Opposition. These included the 1945 TD 3549 as adverted to by the CA in the records58pertaining to a 3.6237-hectare lot in an unstated cadastral lot, TD 0548 covering an 813-hectare lot in Cadastral Lot No. 1676-A,59 TD 0549 for a 2.3642-hectare lot in Cadastral Lot No. 1677-A,60 and TD 0550 concen1ing a 1.1782-hectare lot in Cadastral Lot No. 566A.61 All four TDs are in Tomas's name, without copies of the dorsal portions thereof, and bearing annotations stating either "[c]ontested by [Asuncion]" or "[a]lso declared in the name of [Asuncion] or [Ponciano]". 1âw phi 1

It would thus appear that Tancredo had erected his opposition/claim to the lots in question upon the said photocopies of four tax declarations whose authenticity or genuineness is open to the most serious doubts. And, even on the assumption that the said tax declarations are in fact authentic and genuine, still it is settled that tax declarations are not conclusive proof of ownership. If anything, tax declarations are merely corroborative of a person's claim of possession. More than that, as

elsewhere indicated, intermittent and irregular tax payments, as in this case, do not really provide strong support for a claim of ownership or possession.62 It is axiomatic of course that "[i]t is the policy of the State to encourage and promote the distribution of alienable public lands as a spur to economic growth and in line with the social justice ideal enshrined in the Constitution. At the same time, the law imposes stringent safeguards upon the grant of such resources lest they fall into the wrong hands to the prejudice of the national patrimony."63 This ruling controls the present case. As a final note: All of the foregoing discussion showed that the issues raised in this case have all been previously resolved and determined by settled jurisprudence; hence, there is no reason to grant petitioner's prayer for this case to be referred to or heard by the Court EnBanc, as this is not a case of first impression at all. WHEREFORE, the Petition is hereby DENIED. We AFFIRMwithMODIFICATION the August 13, 2009 Decision and the February 22, 2012 Resolution of the Court of Appeals in CA-G.R. CV No. 02296 in that the award by the Regional Trial Court of Barotac Viejo, Iloilo, Branch 66 in LRC Case No. 02~ 195 of Lot No. 1677 with an area of 2.3642 hectares and Lot No. 566 with an area of 1.1 782 hectares, both in favor of respondent Tancredo Vargas, is OVERTURNED and NULLIFIED SO ORDERED.

Cruz vs Secretary of DENR Natural Resources and Environmental Law; Constitutional Law; IPRA; Regalian Doctrine

GR. No. 135385, Dec. 6, 2000

FACTS: Petitioners Isagani Cruz and Cesar Europa filed a suit for prohibition and mandamus as citizens and taxpayers, assailing the constitutionality of certain provisions of Republic Act No. 8371, otherwise known as the Indigenous People’s Rights Act of 1997 (IPRA) and its implementing rules and regulations (IRR). The petitioners assail certain provisions of the IPRA and its IRR on the ground that these amount to an unlawful deprivation of the State’s ownership over lands of the public domain as well as minerals and other natural resources therein, in violation of the regalian doctrine embodied in section 2, Article XII of the Constitution.

ISSUE: Do the provisions of IPRA contravene the Constitution?

HELD: No, the provisions of IPRA do not contravene the Constitution. Examining the IPRA, there is nothing in the law that grants to the ICCs/IPs ownership over the natural resources within their ancestral domain. Ownership over the natural resources in the ancestral domains remains with the State and the rights granted by the IPRA to the ICCs/IPs over the natural resources in their ancestral domains merely gives them, as owners and occupants of the land on which the resources are found, the right to the small scale utilization of these resources, and at the same time, a priority in their large scale development and exploitation.

Additionally, ancestral lands and ancestral domains are not part of the lands of the public domain. They are private lands and belong to the ICCs/IPs by native title, which is a concept of private land title that existed irrespective of any royal grant from the State. However, the right of ownership and possession by the ICCs/IPs of their ancestral domains is a limited form of ownership and does not include the right to alienate the same.

Land Titles And Deeds Case Digest: Oh Cho V. Director Of Lands (1946) G.R. No. L-48321. August 31, 1946 Lessons Applicable: (Land Titles and Deeds) Sec. 2 Art. XII, 1987 Constitution FACTS: 

Oh Cho is appealing from the rejection of his application based on disqualification as alien (Chinese) from acquiring lands of the public domain.



He had open, continuous, exclusive and notorious possession of the lot from 1880 to filing of the application for registration on January 17, 1940

ISSUE: W/N Oh Cho entitled to decree or registration of the lot.

HELD: NO. 

GR: All lands that were not acquired from the Government, either by purchase or by grant below to the public domain



Exception: in the possession of an occupant and of his predecessors in interest since time immemorial, for such possession would justify the presumption that the land had never been part of the public domain or that it had been a private property even before the Spanish conquest. (Cariño v. Insular Government) - not applicable since only from 1880



His immediate possesor failed to comply with the condition precedent to apply for the registration of the land of which they had been in possession at least since July 26, 1894 so what was transferred to Oh Cho is merely possesory right which cannot ripen to ownership by prescription (aliens disqualified to own by prescription)

G.R. No. 48321, August 31, 1946 o o o o

GR: All lands are acquired from the Government, either by purchase or by grant. EXCEPTION: Lands under private ownership since time immemorial. Application for decree of registration is a condition precedent to acquisition of title. Noncompliance gives rise to mere possessory right. An alien cannot acquire title to lands of the public domain by prescription. FACTS: Oh Cho, a Chinese citizen, purchased from the Lagdameos a parcel of land in Tayabas, which they openly, continuously and adversely possessed since 1880. On January 17, 1940, Oh Cho applied for registration of this land. The Solicitor General opposed on the ground that Oh Cho lacked title to said land and also because he was an alien. ISSUEs:

o o

Whether or not Oh Cho had title Whether or not Oh Cho is entitled to a decree of registration HELD: Oh Cho failed to show that he has title to the lot, which may be confirmed under the Land Registration Act. All lands that were not acquired from the Government, either by purchase or by grant, belong to the public domain. An exception to the rule would be any land that should have been in the possession of an occupant and of his predecessors in interest since time immemorial, for such possession would justify the presumption that the land had never been part of the public domain or that it had been a private property even before the Spanish conquest. The applicant does not come under the exception, for the earliest possession of the lot by his first predecessor in interest began in 1880. Under the Public Land Act, Oh Cho is not entitled to a decree of registration of the lot, because he is an alien disqualified from acquiring lands of the public domain. Oh Cho's predecessors in interest would have been entitled toa decree of registration had they applied for the same. The application for the registration of the land was a condition precedent, which was not complied with by the Lagmeos. Hence, the most

they had was mere possessory right, not title. This possessory right was what was transferred to Oh Cho, but since the latter is an alien, the possessory right could never ripen to ownership by prescription. As an alien, Oh Cho is disqualified from acquiring title over public land by prescription.

La Bugal-B’Laan v. Ramos Chester Cabalza recommends his visitors to please read the original & full text of the case cited. Xie xie! La Bugal-B’Laan v. Ramos G.R. No. 127882. December 1, 2004 Facts: The Petition for Prohibition and Mandamus before the Court challenges the constitutionality of (1) Republic Act 7942 (The Philippine Mining Act of 1995); (2) its Implementing Rules and Regulations (DENR Administrative Order [DAO] 96-40); and (3) the Financial and Technical Assistance Agreement (FTAA) dated 30 March 1995, executed by the government with Western Mining Corporation (Philippines), Inc. (WMCP). On 27 January 2004, the Court en banc promulgated its Decision, granting the Petition and declaring the unconstitutionality of certain provisions of RA 7942, DAO 96-40, as well as of the entire FTAA executed between the government and WMCP, mainly on the finding that FTAAs are service contracts prohibited by the 1987 Constitution. The Decision struck down the subject FTAA for being similar to service contracts,[9] which, though permitted under the 1973 Constitution, were subsequently denounced for being antithetical to the principle of sovereignty over our natural resources, because they allowed foreign control over the exploitation of our natural resources, to the prejudice of the Filipino nation. The Decision quoted several legal scholars and authors who had criticized service contracts for, inter alia, vesting in the foreign contractor exclusive management and control of the enterprise, including operation of the field in the event petroleum was discovered; control of production, expansion and development; nearly unfettered control over the disposition and sale of the products discovered/extracted; effective ownership of the natural resource at the point of extraction; and beneficial ownership of our economic resources. According to the Decision, the 1987 Constitution (Section 2 of Article XII) effectively banned such service contracts. Subsequently, Victor O. Ramos (Secretary, Department of Environment and Natural Resources [DENR]), Horacio Ramos (Director, Mines and Geosciences Bureau [MGB-DENR]), Ruben Torres (Executive Secretary), and the WMC (Philippines) Inc. filed separate Motions for Reconsideration. Issue: Whether or not the Court has a role in the exercise of the power of control over the EDU of our natural resources? Held: The Chief Executive is the official constitutionally mandated to “enter into agreements with foreign owned corporations.” On the other hand, Congress may review the action of the President once it is notified of “every contract entered into in accordance with this [constitutional] provision within thirty days from its execution.” In contrast to this express mandate of the President and Congress in the exploration, development and utilization (EDU) of natural resources, Article XII of the Constitution is silent on the role of the judiciary. However, should the President and/or Congress gravely abuse their discretion in this regard, the courts may -- in a proper case -- exercise their residual duty under Article VIII. Clearly then, the judiciary should not inordinately interfere in the exercise of this presidential power of control over the EDU of our natural resources. Under the doctrine of separation of powers and due respect for co-equal and coordinate branches of government, the Court must restrain itself from intruding into policy matters and must allow the President and Congress maximum discretion in using the resources of our country and in securing the assistance of foreign groups to eradicate the grinding poverty of our people and answer their cry for viable employment opportunities in the country. “The judiciary is loath to interfere with the due exercise by coequal branches of government of their official functions.” As aptly spelled out seven decades ago by Justice George Malcolm, “Just as the Supreme Court, as the guardian of constitutional rights, should not sanction usurpations by any other department of government, so should it as strictly

confine its own sphere of influence to the powers expressly or by implication conferred on it by the Organic Act.” Let the development of the mining industry be the responsibility of the political branches of government. And let not the Court interfere inordinately and unnecessarily. The Constitution of the Philippines is the supreme law of the land. It is the repository of all the aspirations and hopes of all the people. The Constitution should be read in broad, life-giving strokes. It should not be used to strangulate economic growth or to serve narrow, parochial interests. Rather, it should be construed to grant the President and Congress sufficient discretion and reasonable leeway to enable them to attract foreign investments and expertise, as well as to secure for our people and our posterity the blessings of prosperity and peace. The Court fully sympathize with the plight of La Bugal B’laan and other tribal groups, and commend their efforts to uplift their communities. However, the Court cannot justify the invalidation of an otherwise constitutional statute along with its implementing rules, or the nullification of an otherwise legal and binding FTAA contract. The Court believes that it is not unconstitutional to allow a wide degree of discretion to the Chief Executive, given the nature and complexity of such agreements, the humongous amounts of capital and financing required for large-scale mining operations, the complicated technology needed, and the intricacies of international trade, coupled with the State’s need to maintain flexibility in its dealings, in order to preserve and enhance our country’s competitiveness in world markets. On the basis of this control standard, the Court upholds the constitutionality of the Philippine Mining Law, its Implementing Rules and Regulations - insofar as they relate to financial and technical agreements - as well as the subject Financial and Technical Assistance Agreement (FTAA).

La Bugal-B’laan Tribal Association, Inc. Vs Ramos Natural Resources and Environmental Laws

G.R. No. 127882; January 27, 2004 FACTS: This petition for prohibition and mandamus challenges the constitutionality of Republic Act No. 7942 (The Philippine Mining Act of 1995), its implementing rules and regulations and the Financial and Technical Assistance Agreement (FTAA) dated March 30, 1995 by the government with Western Mining Corporation(Philippines) Inc. (WMCP). Accordingly, the FTAA violated the 1987 Constitution in that it is a service contract and is antithetical to the principle of sovereignty over our natural resources, because they allowed foreign control over the exploitation of our natural resources, to the prejudice of the Filipino nation. ISSUE: What is the proper interpretation of the phrase “Agreements involving Either Technical or Financial Assistance” contained in paragraph 4, Section 2, Article XII of the Constitution. HELD: The Supreme Court upheld the constitutionality of the Philippine Mining Law, its implementing rules and regulations – insofar as they relate to financial and technical agreements as well as the subject Financial and Technical Assistance Agreement. Full control is not anathematic to day-to-day management by the contractor, provided that the State retains the power to direct overall strategy; and to set aside, reverse or modify

plans and actions of the contractor. The idea of full control is similar to that which is exercised by the board of directors of a private corporation, the performance of managerial, operational, financial, marketing and other functions may be delegated to subordinate officers or given to contractual entities, but the board retains full residual control of the business.

LA BUGAL B’LAAN TRIBAL ASSOCIATION INC., et. al. v. V. O. RAMOS, Secretary Department of Environment and Natural Resources; H. RAMOS, Director, Mines and Geosciences Bureau (MGB-DENR); R. TORRES, Executive Secretary; and WMC (PHILIPPINES) INC. The constitutional provision allowing the President to enter into FTAA is a exception to the rule that participation in the nation’s natural resources is reserved exclusively to Filipinos. Provision must be construed strictly against their enjoyment by non-Filipinos. RA 7942 (The Philippine Mining Act) took effect on April 9, 1995. Before the effectivity of RA 7942, or on March 30, 1995, the President signed a Financial and Technical Assistance Agreement (FTAA) with WMCP, a corporation organized under Philippine laws, covering close to 100,000 hectares of land in South Cotabato, Sultan Kudarat, Davao del Sur and North Cotabato. On August 15, 1995, the Environment Secretary Victor Ramos issued DENR Administrative Order 95-23, which was later repealed by DENR Administrative Order 96-40, adopted on December 20, 1996. Petitioners prayed that RA 7942, its implementing rules, and the FTAA between the government and WMCP be declared unconstitutional on ground that they allow fully foreign owned corporations like WMCP to exploit, explore and develop Philippine mineral resources in contravention of Article XII Section 2 paragraphs 2 and 4 of the Charter. In January 2001, WMC – a publicly listed Australian mining and exploration company – sold its whole stake in WMCP to Sagittarius Mines, 60% of which is owned by Filipinos while 40% of which is owned by Indophil Resources, an Australian company. DENR approved the transfer and registration of the FTAA in Sagittarius‘ name but Lepanto Consolidated assailed the same. The latter case is still pending before the Court of Appeals. EO 279, issued by former President Aquino on July 25, 1987, authorizes the DENR to accept, consider and evaluate proposals from foreign owned corporations or foreign investors for contracts or agreements involving wither technical or financial assistance for large scale exploration, development and utilization of minerals which upon appropriate recommendation of the (DENR) Secretary, the President may execute with the foreign proponent. WMCP likewise contended that the annulment of the FTAA would violate a treaty between the Philippines and Australia which provides for the protection of Australian investments. ISSUES: 1. Whether or not the Philippine Mining Act is unconstitutional for allowing fully foreign-owned corporations to exploit the Philippine mineral resources. 2. Whether or

not the FTAA between the government and WMCP is a ―service contract that permits fully foreign owned companies to exploit the Philippine mineral resources. HELD: First Issue: RA 7942 is Unconstitutional RA 7942 or the Philippine Mining Act of 1995 is unconstitutional for permitting fully foreign owned corporations to exploit the Philippine natural resources. Article XII Section 2 of the 1987 Constitution retained the Regalian Doctrine which states that ―All lands of the public domain, waters, minerals, coal, petroleum, and other minerals, coal, petroleum, and other mineral oils, all forces of potential energy, fisheries, forests or timber, wildlife, flora and fauna, and other natural resources are owned by the State. The same section also states that, ―the exploration and development and utilization of natural resources shall be under the full control and supervision of the State. Conspicuously absent in Section 2 is the provision in the 1935 and 1973 Constitution authorizing the State to grant licenses, concessions, or leases for the exploration, exploitation, development, or utilization of natural resources. By such omission, the utilization of inalienable lands of the public domain through license, concession or lease is no longer allowed under the 1987 Constitution. Under the concession system, the concessionaire makes a direct equity investment for the purpose of exploiting a particular natural resource within a given area. The concession amounts to complete control by the concessionaireover the country‘s natural resource, for it is given exclusive and plenary rights to exploit a particular resource at the point of extraction. The 1987 Constitution, moreover, has deleted the phrase ―management or other forms of assistance in the 1973 Charter. The present Constitution now allows only ―technical and financial assistance. The management and the operation of the mining activities by foreign contractors, the primary feature of the service contracts was precisely the evil the drafters of the 1987 Constitution sought to avoid. The constitutional provision allowing the President to enter into FTAAs is an exception to the rule that participation in the nation‘s natural resources is reserved exclusively to Filipinos. Accordingly, such provision must be construed strictly against their enjoyment by non-Filipinos. Therefore, RA 7942 is invalid insofar as the said act authorizes service contracts. Although the statute employs the phrase ―financial and technical agreements in accordance with the 1987 Constitution, its pertinent provisions actually treat these agreements as service contracts that grant beneficial ownership to foreign contractors contrary to the fundamental law. The underlying assumption in the provisions of the law is that the foreign contractor manages the mineral resources just like the foreign contractor in a service contract. By allowing foreign contractors to manage or operate all the aspects of the mining operation, RA 7942 has, in effect, conveyed beneficial ownership over the nation‘s mineral resources to these contractors, leaving the State with nothing but bare title thereto. The same provisions, whether by design or inadvertence, permit a circumvention of the constitutionally ordained 60-40% capitalizationrequirement for corporations or associations engaged in the exploitation, development and utilization of Philippine

natural resources. When parts of a statute are so mutually dependent and connected as conditions, considerations, inducements or compensations for each other as to warrant a belief that the legislature intended them as a whole, then if some parts are unconstitutional, all provisions that are thus dependent, conditional or connected, must fail with them. Under Article XII Section 2 of the 1987 Charter, foreign owned corporations are limited only to merely technical or financial assistance to the State for large scale exploration, development and utilization of minerals, petroleum and other mineral oils. Second Issue: RP Government-WMCP FTAA is a Service Contract The FTAA between he WMCP and the Philippine government is likewise unconstitutional since the agreement itself is a service contract. Section 1.3 of the FTAA grants WMCP a fully foreign owned corporation, the exclusive right to explore, exploit, utilize and dispose of all minerals and by-products that may be produced from the contract area. Section 1.2 of the same agreement provides that EMCP shall provide all financing, technology, management, and personnel necessary for the Mining Operations. These contractual stipulations and related provisions in the FTAA taken together, grant WMCP beneficial ownership over natural resources that properly belong to the State and are intended for the benefit of its citizens. These stipulations are abhorrent to the 1987 Constitution. They are precisely the vices that the fundamental law seeks to avoid, the evils that it aims to suppress. Consequently, the contract from which they spring must be struck down.

JG Summit Holdings Inc. vs. CA on 7:00 AM in Case Digests, Commercial Law 0

G.R. No. 124293, November 20, 2000 FACTS: The National Investment and Development Corporation (NIDC), a government corporation, entered into a Joint Venture Agreement (JVA) with Kawasaki Heavy Industries, Ltd. for the construction, operation and management of the Subic National Shipyard, Inc., later became the Philippine Shipyard and Engineering Corporation (PHILSECO). Under the JVA, NIDC and Kawasaki would maintain a shareholding proportion of 60%-40% and that the parties have the right of first refusal in case of a sale. Through a series of transfers, NIDC’s rights, title and interest in PHILSECO eventually went to the National Government. In the interest of national economy, it was decided that PHILSECO should be privatized by selling 87.67% of its total outstanding capital stock to private entities. After negotiations, it was agreed that Kawasaki’s right of first refusal under the JVA be “exchanged” for the right to top by five percent the highest bid for said shares. Kawasaki that Philyards Holdings, Inc. (PHI), in which it was a stockholder, would exercise this right in its stead. During bidding, Kawasaki/PHI Consortium is the losing bidder. Even so, because of the right to top by 5% percent the highest bid, it was able to top JG Summit’s bid. JG Summit protested, contending that PHILSECO, as a shipyard is a public utility and, hence, must observe the 60%40% Filipino-foreign capitalization. By buying 87.67% of PHILSECO’s capital stock at bidding, Kawasaki/PHI in effect now owns more than 40% of the stock. ISSUE:

o o

Whether or not PHILSECO is a public utility Whether or not Kawasaki/PHI can purchase beyond 40% of PHILSECO’s stocks HELD: In arguing that PHILSECO, as a shipyard, was a public utility, JG Summit relied on sec. 13, CA No. 146. On the other hand, Kawasaki/PHI argued that PD No. 666 explicitly stated that a “shipyard” was not a “public utility.” But the SC stated that sec. 1 of PD No. 666 was expressly repealed by sec. 20, BP Blg. 391 and when BP Blg. 391 was subsequently repealed by EO 226, the latter law did not revive sec. 1 of PD No. 666. Therefore, the law that states that a shipyard

is a public utility still stands. A shipyard such as PHILSECO being a public utility as provided by law is therefore required to comply with the 60%-40% capitalization under the Constitution. Likewise, the JVA between NIDC and Kawasaki manifests an intention of the parties to abide by this constitutional mandate. Thus, under the JVA, should the NIDC opt to sell its shares of stock to a third party, Kawasaki could only exercise its right of first refusal to the extent that its total shares of stock would not exceed 40% of the entire shares of stock. The NIDC, on the other hand, may purchase even beyond 60% of the total shares. As a government corporation and necessarily a 100% Filipino-owned corporation, there is nothing to prevent its purchase of stocks even beyond 60% of the capitalization as the Constitution clearly limits only foreign capitalization. Kawasaki was bound by its contractual obligation under the JVA that limits its right of first refusal to 40% of the total capitalization of PHILSECO. Thus, Kawasaki cannot purchase beyond 40% of the capitalization of the joint venture on account of both constitutional and contractual proscriptions. JG SUMMIT HOLDINGS, INC., vs. COURT OF APPEALS, COMMITTEE ON PRIVATIZATION, ASSET PRIVATIZATION TRUST and PHILYARDS HOLDINGS G.R. No. 124293. November 20, 2000

FACTS: National Investment and Development Corporation (NIDC) and Kawasaki Heavy Industries entered into a Joint Venture Agreement in a shipyard business named PHILSECO, with a shareholding of 60-40 respectively. NIDC’s interest was later transferred to the National Government. Pursuant to President Aquino’s Proclamation No.5, which established the Committee on Privatization (COP) and Asset Privatization Trust (APT), and allowed for the disposition of the government’s non-performing assets, the latter allowed Kawasaki Heavy Industries to choose a company to which it has stockholdings, to top the winning bid of JG Summit Holdings over PHILSECO. JG Summit protested alleging that such act would effectively increase Kawasaki’s interest in PHILSECO—a shipyard is a public utility–and thus violative of the Constitution.

ISSUE: Whether or not respondents’ act is valid.

HELD: No. A shipyard such as PHILSECO being a public utility as provided by law, the following provision of the Article XII of the Constitution applies:

“Sec. 11. No franchise, certificate, or any other form of authorization for the operation of a public utility shall be granted except to citizens of the Philippines or to corporations or associations organized under the laws of the Philippines at least sixty per centum of whose capital is owned by such citizens, nor shall such franchise, certificate, or authorization be exclusive in character or for a longer period than fifty years. Neither shall any such franchise or right be granted except under the condition that it shall be subject to amendment, alteration, or repeal by the Congress when the common good so requires. The State shall encourage equity participation in public utilities by the general public. The participation of foreign investors in the governing body of any public utility enterprise shall be limited to their proportionate share in its capital, and all the executive and managing officers of such corporation or association shall be citizens of the Philippines.” Notably, paragraph 1.4 of the JVA accorded the parties the right of first refusal “under the same terms.” This phrase implies that when either party exercises the right of first refusal under paragraph 1.4, they can only do so to the extent allowed them by paragraphs 1.2 and 1.3 of the JVA or under the proportion of 60%-40% of the shares of stock. Thus, should the NIDC opt to sell its shares of stock to a third party, Kawasaki could only exercise its right of first refusal to the extent that its total shares of stock would not exceed 40% of the entire shares of stock of SNS or PHILSECO. The NIDC, on the other hand, may purchase even beyond 60% of the total shares. As a government corporation and necessarily a 100% Filipino-owned corporation, there is nothing to prevent its purchase of stocks even beyond 60% of the capitalization as the Constitution clearly limits only foreign capitalization.

JG Summit Holdings vs CA Case Digest JG Summit Holdings Inc. vs. Court of Appeals [GR 124293, 20 November 2000]

Facts: On 27 January 1977, the National Investment and Development Corporation (NIDC), a government corporation, entered into a Joint Venture Agreement (JVA) with Kawasaki Heavy Industries, Ltd. of Kobe, Japan (Kawasaki) for the construction, operation, and management of the Subic National Shipyard, Inc. (SNS), which subsequently became the Philippine Shipyard and Engineering Corporation (PHILSECO). Under the JVA, NIDC and Kawasaki would maintain a shareholding proportion of 60% - 40%, respectively. One of the provisions of the JVA accorded the parties the right of first refusal should either party sell, assign or transfer its interest in the joint venture. On 25 November 1986, NIDC transferred all its rights, title and interest in PHILSECO to the Philippine National Bank (PNB).

More than two months later or on 3 February 1987, by virtue of Administrative Order 14, PNB's interest in PHILSECO was transferred to the National Government. Meanwhile, on 8 December 1986, President Corazon C. Aquino issued Proclamation 50 establishing the Committee on Privatization (COP) and the Asset Privatization Trust (APT) to take title to and possession of, conserve, manage and dispose of non-performing assets of the National Government. On 27 February 1987, a trust agreement was entered into between the National Government and the APT by virtue of which the latter was named the trustee of the National Government's share in PHILSECO. In 1989, as a result of a quasi-reorganization of PHILSECO to settle its huge obligations to PNB, the National Government's shareholdings in PHILSECO increased to 97.41% thereby reducing Kawasaki's shareholdings to 2.59%. Exercising their discretion, the COP and the APT deemed it in the best interest of the national economy and the government to privatize PHILSECO by selling 87.67% of its total outstanding capital stock to private entities.

After a series of negotiations between the APT and Kasawaki, they agreed that the latter's right of first refusal under the JVA be "exchanged" for the right to top by 5% the highest bid for said shares. They further agreed that Kawasaki would be entitled to name a company in which it was a stockholder, which could exercise the right to top. On 7 September 1990, Kawasaki informed APT that Philyards Holdings, Inc. (PHI) would exercise its right to top by 5%. At the pre-bidding conference held on 28 September 1993, interested bidders were given copies of the JVA between NIDC and Kawasaki, and of the Asset Specific Bidding Rules (ASBR) drafted for the 87.67% equity (sic) in PHILSECO of the National Government. The provisions of the ASBR were explained to the interested bidders who were notified that bidding would be held on 2 December 1993. At the public bidding on said date, the consortium composed of JG Summit Holdings, Inc. (JGSMI), Sembawang Shipyard Ltd. of Singapore (Sembawang), and Jurong Shipyard Limited of Malaysia (Jurong), was declared the highest bidder at P2.03 billion. The following day, the COP approved the sale of 87.67% National Government shares of stock in PHILSECO to said consortium. It notified JGSMI of said approval "subject to the right of Kawasaki Heavy Industries, Inc./Philyards Holdings, Inc. to top JGSMI's bid by 5% as specified in the bidding rules."

On 29 December 1993, JGSMI informed the APT that it was protesting the offer of PHI to top its bid on the grounds that: (a) the Kawasaki/PHI consortium composed of Kawasaki, Philyards, Mitsui, Keppel, SM Group, ICTSI and Insular Life violated the ASBR because the last four (4) companies were the losing bidders (for P1.528 billion) thereby circumventing the law and prejudicing the weak winning bidder; (b) only Kawasaki could exercise the right to top; (c) giving the same option to top to PHI constituted unwarranted benefit to a third party; (d) no right of first refusal can be exercised in a public bidding or auction sale, and (e) the JG Summit Consortium was not estopped from questioning the proceedings. On 2 February 1994, JGSMI was notified that PHI had fully paid the balance of the purchase price of the subject bidding. On 7 February 1994, the APT notified JGSMI that PHI had exercised its option to top the highest bid and that the COP had approved the same on 6 January 1994. On 24 February 1994, the APT and PHI executed a Stock Purchase Agreement. Consequently, JGSMI filed with the Supreme Court a petition for mandamus under GR 114057. On 11 May 1994, said petition was referred to the Court of Appeals. On 18 July 1995, the Court of Appeals "denied" for lack of merit the petition for mandamus. JGSMI filed a motion for the reconsideration of said Decision which was denied on 15 March 1996. JGSMI filed the petition for review on certiorari.

Issue: Whether PHILSECO, as a shipyard, is a public utility and, hence, could be operated only by a corporation at least 60% of whose capital is owned by Filipino citizens, in accordance with Article XII, Section 10 of the Constitution.

Held: A shipyard such as PHILSECO being a public utility as provided by law, Section 11 of the Article XII of the Constitution applies. The provision states that "No franchise, certificate, or any other form of authorization for the operation of a public utility shall be granted except to citizens of the Philippines or to corporations or associations organized under the laws of the Philippines at least sixty per centum of whose capital is owned by such citizens, nor shall such franchise, certificate, or

authorization be exclusive in character or for a longer period than fifty years. Neither shall any such franchise or right be granted except under the condition that it shall be subject to amendment, alteration, or repeal by the Congress when the common good so requires. The State shall encourage equity participation in public utilities by the general public. The participation of foreign investors in the governing body of any public utility enterprise shall be limited to their proportionate share in its capital, and all the executive and managing officers of such corporation or association shall be citizens of the Philippines." The progenitor of this constitutional provision, Article XIV, Section 5 of the 1973 Constitution, required the same proportion of 60% - 40% capitalization. The JVA between NIDC and Kawasaki entered into on 27 January 1977 manifests the intention of the parties to abide by the constitutional mandate on capitalization of public utilities. The joint venture created between NIDC and Kawasaki falls within the purview of an "association" pursuant to Section 5 of Article XIV of the 1973 Constitution and Section 11 of Article XII of the 1987 Constitution. Consequently, a joint venture that would engage in the business of operating a public utility, such as a shipyard, must observe the proportion of 60%-40% Filipino-foreign capitalization. Further, paragraph 1.4 of the JVA accorded the parties the right of first refusal "under the same terms." This phrase implies that when either party exercises the right of first refusal under paragraph 1.4, they can only do so to the extent allowed them by paragraphs 1.2 and 1.3 of the JVA or under the proportion of 60%-40% of the shares of stock. Thus, should the NIDC opt to sell its shares of stock to a third party, Kawasaki could only exercise its right of first refusal to the extent that its total shares of stock would not exceed 40% of the entire shares of stock of SNS or PHILSECO. The NIDC, on the other hand, may purchase even beyond 60% of the total shares. As a government corporation and necessarily a 100% Filipino-owned corporation, there is nothing to prevent its purchase of stocks even beyond 60% of the capitalization as the Constitution clearly limits only foreign capitalization. Kawasaki was bound by its contractual obligation under the JVA that limits its right of first refusal to 40% of the total capitalization of PHILSECO. Thus, Kawasaki cannot purchase beyond 40% of the capitalization of the joint venture on account of both constitutional and contractual proscriptions. From the facts on record, it appears that at the outset, the APT and Kawasaki respected the 60%-40% capitalization proportion in PHILSECO. However, APT subsequently encouraged Kawasaki to participate in the public bidding of the National Government's shareholdings of 87.67% of the total PHILSECO shares, definitely over and above the 40% limit of its shareholdings. In so doing, the APT went beyond the ambit of its authority.

Francisco Tatad vs Jesus Garcia, Jr. November 27, 2012 Share this...

1

0

0

0

ADVERTISEMENTS

243 SCRA 436 – Business Organization – Corporation Law – Corporate Nationality – Public Utility – Nationality Requirement in Nationalized Areas of Activity In 1989, the government planned to build a railway transit line along EDSA. No bidding was made but certain corporations were invited to prequalify. The only corporation to qualify was the EDSA LRT Consortium which was obviously formed for this particular undertaking. An agreement was then made between the government, through the Department of Transportation and Communication (DOTC), and EDSA LRT Consortium. The agreement was based on the Build-Operate-Transfer scheme provided for by law (RA 6957, amended by RA 7718). Under the agreement, EDSA LRT Consortium shall build the facilities, i.e., railways, and shall supply the train cabs. Every phase that is completed shall be turned over to the DOTC and the latter shall pay rent for the same for 25 years. By the end of 25 years, it was projected that the government shall have fully paid EDSA LRT Consortium. Thereafter, EDSA LRT Consortium shall sell the facilities to the government for $1.00. However, Senators Francisco Tatad, John Osmeña, and Rodolfo Biazon opposed the implementation of said agreement as they averred that EDSA LRT Consortium is a foreign corporation as it was organized under Hongkong laws; that as such, it cannot own a public utility such as the EDSA railway transit because this falls under the nationalized areas of activities. The petition was filed against Jesus Garcia, Jr. in his capacity as DOTC Secretary. ISSUE: Whether or not the petition shall prosper. HELD: No. The Supreme Court made a clarification. The SC ruled that EDSA LRT Consortium, under the agreement, does not and will not become the owner of a public utility

hence, the question of its nationality is misplaced. It is true that a foreign corporation cannot own a public utility but in this case what EDSA LRT Consortium will be owning are the facilities that it will be building for the EDSA railway project. There is no prohibition against a foreign corporation to own facilities used for a public utility. Further, it cannot be said that EDSA LRT Consortium will be the one operating the public utility for it will be DOTC that will operate the railway transit. DOTC will be the one exacting fees from the people for the use of the railway and from the proceeds, it shall be paying the rent due to EDSA LRT Consortium. All that EDSA LRT Consortium has to do is to build the facilities and receive rent from the use thereof by the government for 25 years – it will not operate the railway transit. Although EDSA LRT Consortium is a corporation formed for the purpose of building a public utility it does not automatically mean that it is operating a public utility. The moment for determining the requisite Filipino nationality is when the entity applies for a franchise, certificate or any other form of authorization for that purpose.

TATAD V. GARCIA, JR. (G.R. NO. 114222) Facts: DOTC planned to construct a light railway transit line along EDSA (EDSA LRT III) to provide a mass transit system and alleviate the congestion and growing transportation problem in the metropolis. RA 6957 was enacted allowing for the financing, construction and operation of government projects through private initiative and investment. Accordingly, prequalification and bidding was made and EDSA LRT Corporation (organized under HK laws) was recommended to be awarded with the contract. The President approved the awarding of the contract. Petitioners are senators praying for the prohibition of respondents from further implementing and enforcing the contract. Issue: Whether or not the EDSA LRT III, a public utility, can be owned by a foreign corporation. Ruling: YES. The Constitution, in no uncertain terms, requires a franchise for the operation of a public utility. However, it does not require a franchise before one can own the facilities needed to operate a public utility so long as it does not operate them to serve the public. In law, there is a clear distinction between the “operation” of a public utility and the ownership of the facilities and equipment used to serve the public. Ownership is defined as a relation in law by virtue of which a thing pertaining to one person is completely subjected to his will in everything not prohibited by law or the concurrence with the rights of another. The exercise

of the rights encompassed in ownership is limited by law so that a property cannot be operated and used to serve the public as a public utility unless the operator has a franchise. The operation of a rail system as a public utility includes the transportation of passengers from one point to another point, their loading and unloading at designated places and the movement of the trains at pre-scheduled times. In sum, private respondent will not run the light rail vehicles and collect fees from the riding public. It will have no dealings with the public and the public will have no right to demand any services from it. Even the mere formation of a public utility corporation does not ipso facto characterize the corporation as one operating a public utility. The moment for determining the requisite Filipino nationality is when the entity applies for a franchise, certificate or any other form of authorization for that purpose.

JACOBUS BERNHARD HULST, Petitioner v. PR BUILDERS, INC., Respondent. RESOLUTION AUSTRIA-MARTINEZ, J.: This resolves petitioner's Motion for Partial Reconsideration. On September 3, 2007, the Court rendered a Decision1 in the present case, the dispositive portion of which reads: WHEREFORE, the instant petition is GRANTED. The Decision dated October 30, 2002 of the Court of Appeals in CA-G.R. SP No. 60981 is REVERSED and SET ASIDE. The Order dated August 28, 2000 of HLURB Arbiter Ma. Perpetua Y. Aquino and Director Belen G. Ceniza in HLRB Case No. IV6-071196-0618 is declared NULL and VOID. HLURB Arbiter Aquino and Director Ceniza are directed to issue the corresponding certificates of sale in favor of the winning bidder, Holly Properties Realty Corporation. Petitioner is ordered to return to respondent the amount of P2,125,540.00, without interest, in excess of the proceeds of the auction sale delivered to petitioner. After the finality of herein judgment, the amount of P2,125,540.00 shall earn 6% interest until fully paid. SO ORDERED.2 (Emphasis supplied)

cra lawlib rary

Petitioner filed the present Motion for Partial Reconsideration3 insofar as he was ordered to return to respondent the amount of P2,125,540.00 in excess of the proceeds of the auction sale delivered to petitioner. Petitioner contends that the Contract to Sell between petitioner and respondent involved a condominium unit and did not violate the Constitutional proscription against ownership of land by aliens. He argues that the contract to sell will not transfer to the buyer ownership of the land on which the unit is situated; thus, the buyer will not get a transfer certificate of title but merely a Condominium Certificate of Title as evidence of ownership; a perusal of the contract will show that what the buyer acquires is the seller's title and rights to and interests in the unit and the common areas. Despite receipt of this Court's Resolution dated February 6, 2008, respondent failed to file a comment on the subject motion. The Motion for Partial Reconsideration is impressed with merit. The Contract to Sell between petitioner and respondent provides as follows: Section 3. TITLE AND OWNERSHIP OF UNIT A. Upon full payment by the BUYER of the purchase price stipulated in Section 2 hereof, x x x, the SELLER shall deliver to the BUYER the Deed of Absolute Sale conveying its rights, interests and title to the UNIT and to the common areas appurtenant to such UNIT, and the corresponding Condominium Certificate of Title in the SELLER's name; x x x b. The Seller shall register with the proper Registry of Deeds, the Master Deed with the Declaration of Restrictions and other documents and shall immediately comply with all requirements of Republic Act No. 4726 (The Condominium Act) and Presidential Decree No. 957 (Regulating the Sale of Subdivision Lots and Condominiums, Providing Penalties for Violations Thereof). It is hereby understood that all title, rights and interest so conveyed shall be subject to the provisions of the Condominium Act, the Master Deed with Declaration of Restrictions, the Articles of Incorporation and By-Laws and the Rules and Regulations of the Condominium Corporation, zoning regulations and such other restrictions on the use of the property as annotated on the title or may be imposed by any government agency or instrumentality having jurisdiction thereon.4(Emphasis supplied) c ralawl ibra ry

Under Republic Act (R.A.) No. 4726, otherwise known as the Condominium Act, foreign nationals can own Philippine real estate through the purchase of condominium units or townhouses constituted under the Condominium principle with Condominium Certificates of Title. Section 5 of R.A. No. 4726 states: SECTION 5. Any transfer or conveyance of a unit or an apartment, office or store or other space therein, shall include the transfer or conveyance of the undivided interest in the common areas or, in a proper case, the membership or shareholdings in the condominium corporation; Provided, however, That where the common areas in the condominium project are held by the owners of separate units as co-owners thereof, no condominium unit therein shall be conveyed or transferred to persons other than Filipino citizens or corporations at least 60% of the capital stock of which belong to Filipino citizens, except in cases of hereditary succession. Where the common areas in a condominium project are held by a corporation, no transfer or conveyance of a unit shall be valid if the concomitant transfer of the appurtenant membership or stockholding in the corporation will cause the alien interest in such corporation to exceed the limits imposed by existing laws. (Emphasis supplied) cral awlib rary

The law provides that no condominium unit can be sold without at the same time selling the corresponding amount of rights, shares or other interests in the condominium management body, the Condominium Corporation; and no one can buy shares in a Condominium Corporation without at the same time buying a condominium unit. It expressly allows foreigners to acquire condominium units and shares in condominium corporations up to not more than 40% of the total and outstanding capital stock of a Filipino-owned or controlled corporation. Under this set up, the ownership of the land is legally separated from the unit itself. The land is owned by a Condominium Corporation and the unit owner is simply a member in this Condominium Corporation.5 As long as 60% of the members of this Condominium Corporation are Filipino, the remaining members can be foreigners. Considering that the rights and liabilities of the parties under the Contract to Sell is covered by the Condominium Act wherein petitioner as unit owner was simply a member of the Condominium Corporation and the land remained owned by respondent, then the constitutional proscription against aliens owning real property does not apply to the present case. There being no circumvention of the constitutional prohibition, the Court's pronouncements on the invalidity of the Contract of Sale should be set aside. WHEREFORE, the Motion for Partial Reconsideration is GRANTED. Accordingly, the Decision dated September 3, 2007 of the Court is MODIFIED by deleting the order to petitioner to return to respondent the amount of P2,125,540.00 in excess of the proceeds of the auction sale delivered to petitioner. SO ORDERED.

Agan v PIATCO G.R. No. 155001. May 5, 2003 7/6/2010 0 Comments

Facts: In August 1989, the DOTC engaged the services of Aeroport de Paris (ADP) to conduct a comprehensive study of the Ninoy Aquino International Airport (NAIA) and determine whether the present airport can cope with the traffic development up to the year 2010. On March 27, 1995, then DOTC Secretary Jose Garcia endorsed the proposal of Asia's Emerging Dragon Corp. (unsolicited proposal dated Oct. 5, 1994) to the National Economic and Development Authority (NEDA). A revised proposal, however, was forwarded by the DOTC to NEDA on December 13, 1995. On January 5, 1996, the NEDA Investment Coordinating Council (NEDA ICC) — Technical Board favorably endorsed the project to the ICC — Cabinet Committee which approved the same, subject to certain conditions, on January 19, 1996. On February 13, 1996, the NEDA passed Board Resolution No. 2 which approved the NAIA IPT III Project. On August 29, 1996, the Second Pre-Bid Conference was held where certain clarifications were made. Upon the request of prospective bidder People's Air Cargo & Warehousing Co., Inc (Paircargo), the PBAC warranted that based on Sec. 11.6, Rule 11 of the Implementing Rules and Regulations of the BOT Law, only the proposed Annual Guaranteed Payment submitted by the challengers would be revealed to AEDC, and that the challengers' technical and financial proposals would remain confidential. The PBAC also clarified that the list of revenue sources contained in Annex 4.2a of the Bid Documents was merely indicative and that other revenue sources may be included by the proponent, subject to approval by DOTC/MIAA. Furthermore, the PBAC clarified that only those fees and charges denominated as Public Utility Fees would be subject to regulation, and those charges which would be actually deemed Public Utility Fees could still be revised, depending on the outcome of PBAC's query on the matter with the Department of Justice. On September 26, 1996, AEDC informed the PBAC in writing of its reservations as regards the Paircargo Consortium, which include: a. The lack of corporate approvals and financial capability of PAIRCARGO; b. The lack of corporate approvals and financial capability of PAGS; c. The prohibition imposed by RA 337, as amended (the General Banking Act) on the amount that Security Bank could legally invest in the project; d. The inclusion of Siemens as a contractor of the PAIRCARGO Joint Venture, for prequalification purposes; and e. The appointment of Lufthansa as the facility operator, in view of the Philippine requirement in the operation of a public utility. The PBAC gave its reply on October 2, 1996, informing AEDC that it had considered the issues raised by the latter, and that based on the documents submitted by Paircargo and the established prequalification criteria, the PBAC had found that the challenger, Paircargo, had prequalified to undertake the project. The Secretary of the DOTC approved the finding of the PBAC. On October 16, 1996, the PBAC opened the third envelope submitted by AEDC and the Paircargo Consortium containing their respective financial proposals. Both proponents offered to build the NAIA Passenger Terminal III for at least $350 million at no cost to the government and to pay the government: 5% share in gross revenues for the first five years of operation, 7.5% share in gross revenues for the next ten years of operation, and 10% share in gross revenues for the last ten years of operation, in accordance with the Bid Documents. As AEDC failed to match the proposal within the 30-day period, then DOTC Secretary Amado Lagdameo, on December 11, 1996, issued a notice to Paircargo Consortium regarding AEDC's failure to match the proposal. AEDC subsequently protested the alleged undue preference given to PIATCO and reiterated its objections as regards the

prequalification of PIATCO. On July 12, 1997, the Government, through then DOTC Secretary Arturo T. Enrile, and PIATCO, through its President, Henry T. Go, signed the "Concession Agreement for the Build-Operate-and-Transfer Arrangement of the Ninoy Aquino International Airport Passenger Terminal III" (1997 Concession Agreement). The Government granted PIATCO the franchise to operate and maintain the said terminal during the concession period and to collect the fees, rentals and other charges in accordance with the rates or schedules stipulated in the 1997 Concession Agreement. The Agreement provided that the concession period shall be for twenty-five (25) years commencing from the inservice date, and may be renewed at the option of the Government for a period not exceeding twenty-five (25) years. At the end of the concession period, PIATCO shall transfer the development facility to MIAA. During the pendency of the case before this Court, President Gloria Macapagal Arroyo, on November 29, 2002, in her speech at the 2002 Golden Shell Export Awards at Malacañang Palace, stated that she will not "honor (PIATCO) contracts which the Executive Branch's legal offices have concluded (as) null and void."

Issue: Whether the petitioners and the petitioners-in-intervention have standing; Whether this Court has jurisdiction; and Whether the BOT and contracts therein are unconstitutional.

Held: YES. Ratio: Messrs. Lopez et al. are employees of the MIAA. These petitioners (Messrs. Agan et al. and Messrs. Lopez et al.) are confronted with the prospect of being laid off from their jobs and losing their means of livelihood when their employer-companies are forced to shut down or otherwise retrench and cut back on manpower. Such development would result from the imminent implementation of certain provisions in the contracts that tend toward the creation of a monopoly in favor of Piatco, its subsidiaries and related companies. Petitioners-in-intervention are service providers in the business of furnishing airport-related services to international airlines and passengers in the NAIA and are therefore competitors of Piatco as far as that line of business is concerned. On account of provisions in the Piatco contracts, petitioners-in-intervention have to enter into a written contract with Piatco so as not to be shut out of NAIA Terminal III and barred from doing business there. Since there is no provision to ensure or safeguard free and fair competition, they are literally at its mercy. They claim injury on account of their deprivation of property (business) and of the liberty to contract, without due process of law. By way of background, two monopolies were actually created by the Piatco contracts. The first and more obvious one refers to the business of operating an international passenger terminal in Luzon, the business end of which involves providing international airlines with parking space for their aircraft, and airline passengers with the use of departure and arrival areas, check-in counters, information systems, conveyor systems, security equipment and paraphernalia, immigrations and customs processing areas; and amenities such as comfort rooms, restaurants and shops. In furtherance of the first monopoly, the Piatco Contracts stipulate that the NAIA Terminal III will be the only facility to be operated as an international passenger terminal; that NAIA Terminals I and II will no longer be operated as such; and that no one (including the government) will be allowed to compete with Piatco in the operation of an international passenger terminal in the NAIA Complex. Given that, at this time, the government and Piatco are the only ones engaged in the business of operating an international passenger terminal, I am not acutely concerned with this particular monopolistic situation. There was however another monopoly within the NAIA created by the subject contracts for Piatco — in the business of providing international airlines with the following: groundhandling, in-flight catering, cargo handling, and aircraft repair and maintenance services. These are lines of business activity in which are engaged many service providers (including the petitioners-in-intervention), who will be adversely affected upon full implementation of the Piatco Contracts, particularly Sections 3.01(d) and (e) of both the ARCA and the CA.

Should government pay at all for reasonable expenses incurred in the construction of the Terminal? Indeed it should, otherwise it will be unjustly enriching itself at the expense of Piatco and, in particular, its funders, contractors and investors — both local and foreign. After all, there is no question that the State needs and will make use of Terminal III, it being part and parcel of the critical infrastructure and transportation-related programs of government. The rule on hierarchy of courts will not also prevent this Court from assuming jurisdiction over the cases at bar. The said rule may be relaxed when the redress desired cannot be obtained in the appropriate courts or where exceptional and compelling circumstances justify availment of a remedy within and calling for the exercise of this Court's primary jurisdiction. Thus, considering the nature of the controversy before the Court, procedural bars may be lowered to give way for the speedy disposition of the instant cases. In sum, this Court rules that in view of the absence of the requisite financial capacity of the Paircargo Consortium, predecessor of respondent PIATCO, the award by the PBAC of the contract for the construction, operation and maintenance of the NAIA IPT III is null and void.

Is PIATCO a qualified bidder? Public respondents argue that the Paircargo Consortium, PIATCO's predecessor, was not a duly pre-qualified bidder on the unsolicited proposal submitted by AEDC as the Paircargo Consortium failed to meet the financial capability required under the BOT Law and the Bid Documents. They allege that in computing the ability of the Paircargo Consortium to meet the minimum equity requirements for the project, the entire net worth of Security Bank, a member of the consortium, should not be considered. R.A. No. 337, as amended or the General Banking Act that a commercial bank cannot invest in any single enterprise in an amount more than 15% of its net worth. We agree with public respondents that with respect to Security Bank, the entire amount of its net worth could not be invested in a single undertaking or enterprise, whether allied or non-allied in accordance with the provisions of R.A. No. 337 The PBAC should not be allowed to speculate on the future financial ability of the bidder to undertake the project on the basis of documents submitted. This would open doors to abuse and defeat the very purpose of a public bidding.

Is the 1997 Concession Agreement valid? Petitioners and public respondents contend that the 1997 Concession Agreement is invalid as it contains provisions that substantially depart from the draft Concession Agreement included in the Bid Documents. They maintain that a substantial departure from the draft Concession Agreement is a violation of public policy and renders the 1997 Concession Agreement null and void. If the winning bidder is allowed to later include or modify certain provisions in the contract awarded such that the contract is altered in any material respect, then the essence of fair competition in the public bidding is destroyed. A public bidding would indeed be a farce if after the contract is awarded; the winning bidder may modify the contract and include provisions which are favorable to it that were not previously made available to the other bidders. With respect to terminal fees that may be charged by PIATCO, as shown earlier, this was included within the category of "Public Utility Revenues" under the 1997 Concession Agreement. This classification is significant because under the 1997 Concession Agreement, "Public Utility Revenues" are subject to an "Interim Adjustment" of fees upon the occurrence of certain extraordinary events specified in the agreement. However, under the draft Concession Agreement, terminal fees are not included in the types of fees that may be subject to "Interim Adjustment." Finally, under the 1997 Concession Agreement, "Public Utility Revenues," except terminal fees, are denominated in US Dollars while payments to the Government are in Philippine Pesos. In the draft Concession Agreement, no such stipulation was included. By stipulating that "Public Utility Revenues" will be paid to PIATCO in US Dollars while payments by PIATCO to the Government are in Philippine currency under the 1997 Concession Agreement, PIATCO

is able to enjoy the benefits of depreciations of the Philippine Peso, while being effectively insulated from the detrimental effects of exchange rate fluctuations. Under the draft Concession Agreement, default by PIATCO of any of its obligations to creditors who have provided, loaned or advanced funds for the NAIA IPT III project does not result in the assumption by the Government of these liabilities. In fact, nowhere in the said contract does default of PIATCO's loans figure in the agreement. Such default does not directly result in any concomitant right or obligation in favor of the Government. It is clear from the above-quoted provisions that Government, in the event that PIATCO defaults in its loan obligations, is obligated to pay "all amounts recorded and from time to time outstanding from the books" of PIATCO which the latter owes to its creditors. These amounts include "all interests, penalties, associated fees, charges, surcharges, indemnities, reimbursements and other related expenses." This obligation of the Government to pay PIATCO's creditors upon PIATCO's default would arise if the Government opts to take over NAIA IPT III. It should be noted, however, that even if the Government chooses the second option, which is to allow PIATCO's unpaid creditors operate NAIA IPT III, the Government is still at a risk of being liable to PIATCO's creditors should the latter be unable to designate a qualified operator within the prescribed period. In effect, whatever option the Government chooses to take in the event of PIATCO's failure to fulfill its loan obligations, the Government is still at a risk of assuming PIATCO's outstanding loans. As such the Government is virtually at the mercy of PIATCO (that it would not default on its loan obligations to its Senior Lenders), the Senior Lenders (that they would appoint a qualified nominee or transferee or agree to some other arrangement with the Government) and the existence of a qualified nominee or transferee who is able and willing to take the place of PIATCO in NAIA IPT III.

In view of regulation of monopolies The operation of an international passenger airport terminal is no doubt an undertaking imbued with public interest. In entering into a Build-Operate-and-Transfer contract for the construction, operation and maintenance of NAIA IPT III, the government has determined that public interest would be served better if private sector resources were used in its construction and an exclusive right to operate be granted to the private entity undertaking the said project, in this case PIATCO. Nonetheless, the privilege given to PIATCO is subject to reasonable regulation and supervision by the Government through the MIAA, which is the government agency authorized to operate the NAIA complex, as well as DOTC, the department to which MIAA is attached. While it is the declared policy of the BOT Law to encourage private sector participation by "providing a climate of minimum government regulations," the same does not mean that Government must completely surrender its sovereign power to protect public interest in the operation of a public utility as a monopoly. The operation of said public utility cannot be done in an arbitrary manner to the detriment of the public which it seeks to serve. In contrast to the arrastre and stevedoring service providers in the case of Anglo-Fil Trading Corporation v. Lazaro whose contracts consist of temporary hold-over permits, the affected service providers in the cases at bar, have a valid and binding contract with the Government, through MIAA, whose period of effectivity, as well as the other terms and conditions thereof cannot be violated.

Should the dispute be referred to arbitration prior to judicial recourse? Respondent Piatco claims that Section 10.02 of the Amended and Restated Concession Agreement (ARCA) provides for arbitration under the auspices of the International Chamber of Commerce to settle any dispute or controversy or claim arising in connection with the Concession Agreement, its amendments and supplements. The government disagrees; however, insisting that there can be no arbitration based on Section 10.02 of the ARCA, since all the Piatco contracts are void ab initio.

G.R. No. L-27170 November 22, 1977 IN RE APPLICATION FOR THE REGISTRATION OF LAND. EUGENE MOSS, through his Administrator, DR. TEODORICO H. JACELDO, applicant-appellant, vs. DIRECTOR OF LANDS, oppositor-appellee. Estanislao L. Granados for appellant. Solicitor General Estelito P. Mendoza, Assistant Solicitor General Eulogio Raquel-Santos and Solicitor Salvador C. Jacob for appellee.

AQUINO, J.: Eugene Moss appealed from the decision of the Court of First Instance of Leyte, Carigara Branch VI, denying his application for the registration of a ten-hectare island on the ground that, being an American citizen or an alien, he is disqualified to acquire lands under section 5, Article XIII of the 1935 Constitution, as held in Krivenko vs. Register of Deeds, 79 Phil. 461 (Land Registration Case No. N-68, LRC Record No. N-27971). The Solicitor General, disagreeing with the trial court, recommends in his brief for the Director of Lands that the decision be reversed and that the application of Moss be granted. On Carigara Bay there is an islet known as Calumpihan Island within the jurisdiction of Barrio Calumpihan, Capoocan, Leyte. It is planted to coconuts more than sixty to eighty years old in 1966. Fruit trees, corn and tobacco are also grown on the land. That land was already declared for tax p in 1930. It was then owned by Vicente Pragas who had possessed and cultivated it since 1894. On November 14, 1932 Pragas sold the island to Eduardo Cecilio. On March 23, 1937 Cecilio sold it to Catalina Pabilion. Then, on December 23, 1944 the spouses Catalina Pabilion and Guillermo Dadizon sold the land to Rufino M. Pascual who, in turn sold it on January 20, 1945 for P 1,200 to Eugene Moss and Albert Boyd Cassidy, American nationals. In an action to quiet title filed by Moss against Cassidy, an absentee, Moss was ajudged the sole owner of the and in a decision dated March 27, 1962 rendered by the Court of First Instance of Leyte in Civil Case No. 645. Moss declared the land for tax purposes on June 27, 1950. He paid the realty taxes on the said land for the years 1945 to 1966. He had it surveyed on November 20, 1962. On April 3, 1965 Moss, through Dr. Teodorico H. Jaceldo, his administrator and attorney-in-fact, filed an application for the registration of the said land. Moss is a retired army colonel, an American citizen, married to Eileen Moss and a former resident of 504 Glen Park Drive, Nashville, Tennessee and now Los Fresnos Texas. As already stated, the trial court denied the application for registration because Moss is an alien. That holding is erroneous. It is erroneous because while aliens are disqualified to Acquire lands

under the 1935 Constitution (Levy Hermanos vs. Ledesma, 69 Phil. 49; Krivenko vs. Register of Deeds, supra), citizens of the United States can acquire lands like Filipino citizens. The trial court overlooked the Ordinance which was appended to the 1935 Constitution by Resolution No. 39 of the National Assembly dated September 15, 1939. That resolution was approved by the President of the United States on November 10, 1939. It provides as follows: ORDINANCE APPENDED TO THE CONSTITUTION SECTION 1. Notwithstanding the provisions of the foregoing Constitution, pending the final and complete withdrawal of the sovereignty of the United States over the Philippines — xxx xxx xxx (17) Citizens and corporations of the United States shall enjoy in the Commonwealth of the Philippines all the civil rights of the citizens and corporations, respectively, thereof. (1 Philippine Annotated Laws, pp. 31-34). That Ordinance was made a part of the 1935 Constitution as directed in section 2 of the TydingsMcDuffie Law or the Independence law. Subsection (17) quoted above is a reproduction of subsection (16) of section 2(a) of the Independence Law. Moss validly acquired the island in question under the provisions of subsection (17), section I of the aforementioned Ordinance (See sec. 127 of Public Land Law; Republic vs. Quasha, L-30299, August 17, 1972, 46 SCRA 160, 172-173. Note that the instant case is not covered by section 11, Article XVII of the 1973 Constitution and by Presidential Decree No. 713 dated May 27, 1975, 71 O.G. 4115 re residential lands of American citizens). The proclamation of Philippine independence on July 4, 1946, did not impair Moss' proprietary fights over the said land because the 1935 Constitution provides that upon the proclamation of Philippine independence 'all existing property rights of citizens or corporations of the United States shall be acknowledged, respected, and safeguarded to the same extent as property right of citizens of the Philippines" (Sec. 1[1], Article XVII). That constitutional provision is implemented in Article VI of the Treaty of General Relations entered into between the Republic of the Philippines and the United States on July 4, 1946 which provides that all existing property rights of citizens and corporations of the United States in the Republic of the Philippines shall be acknowledged, respected and safeguarded to the same extent as property rights of citizens and corporation of the Republic of the Philippines (42 O.G. 1651. See Republic Act No. 76). When Moss purchased the land, Leyte was already liberated as shown in the proclamation of General Douglas MacArthur dated October 23, 1944. (Moss and Cassidy were allegedly members of the American liberation forces that landed in Leyte). See 41 O.G. 147. Since Moss and his predecessors in interest have been in possession en concepts de dueño of Calumpihan Island for more than thirty years immediately preceding the firing of his application for confirmation of his title, he is entitled to the registration of his title to the island (Sec. 48[b], Public Land Law, as amended by Republic Act No. 1942).

WHEREFORE, the trial court's decision is reversed and set aside and the application of Eugene Moss, of age, citizen of the United States, married to Eileen Moss, P. O. Box 184, Los Fresnos Texas, U. S. A., for the confirmation and registration of his title to the land shown in the plan Psu198022, is hereby granted. SO ORDERED.

BOARD OF MEDICINE, DR. RAUL FLORES (now DR. JOSE S. RAMIREZ), in his capacity as Chairman of the Board, PROFESSIONAL REGULATION COMMISSION, through its Chairman, HERMOGENES POBRE (now DR. ALCESTIS M. GUIANG), Petitioners,

G.R. No. 166097 Present: QUISUMBING,* J., YNARES-SANTIAGO, Chairperson, AUSTRIA-MARTINEZ, NACHURA, and REYES, JJ.

-versusPromulgated: July 14, 2008

YASUYUKI OTA, Respondent. x -- -- -- -- -- -- -- -- -- -- -- -- -- -- -- -- -- -- -- -- -- -- -- -- -- -- -- -- -- -- -- -- x

DECISION AUSTRIA-MARTINEZ, J.: Before the Court is a Petition for Review on Certiorari assailing the Decision[1] of the Court of Appeals (CA) in CA-G.R. SP No. 84945[2] dated November 16, 2004 which affirmed the Decision[3] of the Regional Trial Court (RTC), Branch 22, Manila, dated October 19, 2003.[4] The facts are as follows: Yasuyuki Ota (respondent) is a Japanese national, married to a Filipina, who has continuously resided in the Philippines for more than 10 years. He graduated from BicolChristian College of Medicine on April 21, 1991 with a degree of Doctor of Medicine.[5] After successfully completing a one-year post graduate internship training at the JoseReyes Memorial Medical Center, he filed an application to take the medical board examinations in order to obtain a medical license. He was required by the Professional Regulation Commission (PRC) to submit an affidavit of undertaking, stating among others that should he

successfully pass the same, he would not practice medicine until he submits proof that reciprocity exists between Japan and the Philippines in admitting foreigners into the practice of medicine.[6] Respondent submitted a duly notarized English translation of the Medical Practitioners Law of Japan duly authenticated by the Consul General of the Philippine Embassy to Japan, Jesus I. Yabes;[7] thus, he was allowed to take the Medical Board Examinations in August 1992, which he subsequently passed.[8] In spite of all these, the Board of Medicine (Board) of the PRC, in a letter dated March 8, 1993, denied respondent's request for a license to practice medicine in the Philippines on the ground that the Board believes that no genuine reciprocity can be found in the law of Japan as there is no Filipino or foreigner who can possibly practice there.[9] Respondent then filed a Petition for Certiorari and Mandamus against the Board before the RTC of Manila on June 24, 1993, which petition was amended on February 14, 1994to implead the PRC through its Chairman.[10] In his petition before the RTC, respondent alleged that the Board and the PRC, in refusing to issue in his favor a Certificate of Registration and/or license to practice medicine, had acted arbitrarily, in clear contravention of the provision of Section 20 of Republic Act (R.A.) No. 2382 (The Medical Act of 1959), depriving him of his legitimate right to practice his profession in the Philippines to his great damage and prejudice.[11] On October 19, 2003, the RTC rendered its Decision finding that respondent had adequately proved that the medical laws of Japan allow foreigners like Filipinos to be granted license and be admitted into the practice of medicine under the principle of reciprocity; and that the Board had a ministerial duty of issuing the Certificate of Registration and license to respondent, as it was shown that he had substantially complied with the requirements under the law.[12] The RTC then ordered the Board to issue in favor of respondent the corresponding Certificate of Registration and/or license to practice medicine in the Philippines.[13] The Board and the PRC (petitioners) appealed the case to the CA, stating that while respondent submitted documents showing that foreigners are allowed to practice medicine in Japan, it was not shown that the conditions for the practice of

medicine there are practical and attainable by a foreign applicant, hence, reciprocity was not established; also, the power of the PRC and the Board to regulate and control the practice of medicine is discretionary and not ministerial, hence, not compellable by a writ of mandamus.[14] The CA denied the appeal and affirmed the ruling of the RTC.[15] Hence, herein petition raising the following issue: WHETHER THE COURT OF APPEALS COMMITTED A REVERSIBLE ERROR IN FINDING THAT RESPONDENT HAD ESTABLISHED THE EXISTENCE OF RECIPROCITY IN THE PRACTICE OF MEDICINE BETWEEN THE PHILIPPINES AND JAPAN.[16]

Petitioners claim that: respondent has not established by competent and conclusive evidence that reciprocity in the practice of medicine exists between the Philippines and Japan. While documents state that foreigners are allowed to practice medicine in Japan, they do not similarly show that the conditions for the practice of medicine in said country are practical and attainable by a foreign applicant. There is no reciprocity in this case, as the requirements to practice medicine in Japan are practically impossible for a Filipino to comply with. There are also ambiguities in the Medical Practitioners Law of Japan, which were not clarified by respondent, i.e., what are the provisions of the School Educations Laws, what are the criteria of the Minister of Health and Welfare of Japan in determining whether the academic and technical capability of foreign medical graduates are the same or better than graduates of medical schools in Japan, and who can actually qualify to take the preparatory test for the National Medical Examination. Consul General Yabes also stated that there had not been a single Filipino who was issued a license to practice medicine by the Japanese Government. The publication showing that there were foreigners practicing medicine in Japan, which respondent presented before the Court, also did not specifically show that Filipinos were among those listed as practicing said profession.[17]Furthermore, under Professional Regulation Commission v. De Guzman,[18] the power of the PRC and the Board to regulate and control the practice of medicine includes the power to regulate admission to the ranks of those authorized to practice medicine, which power is discretionary and not ministerial, hence, not compellable by a writ of mandamus.[19] Petitioners pray that the CA Decision dated November 16, 2004 be reversed and set aside, that a new one be rendered reinstating the Board Order dated March 8,

1993 which disallows respondent to practice medicine in the Philippines, and that respondent's petition before the trial court be dismissed for lack of merit. [20] In his Comment, respondent argues that: Articles 2 and 11 of the Medical Practitioners Law of Japan and Section 9 of the Philippine Medical Act of 1959 show that reciprocity exists between the Philippines and Japan concerning the practice of medicine. Said laws clearly state that both countries allow foreigners to practice medicine in their respective jurisdictions as long as the applicant meets the educational requirements, training or residency in hospitals and pass the licensure examination given by either country. Consul General Yabes in his letter dated January 28, 1992 stated that the Japanese Government allows a foreigner to practice medicine in Japan after complying with the local requirements. The fact that there is no reported Filipino who has successfully penetrated the medical practice in Japan does not mean that there is no reciprocity between the two countries, since it does not follow that no Filipino will ever be granted a medical license by the Japanese Government. It is not the essence of reciprocity that before a citizen of one of the contracting countries can demand its application, it is necessary that the interested citizens country has previously granted the same privilege to the citizens of the other contracting country.[21] Respondent further argues that Section 20 of the Medical Act of 1959[22] indicates the mandatory character of the statute and an imperative obligation on the part of the Board inconsistent with the idea of discretion. Thus, a foreigner, just like a Filipino citizen, who successfully passes the examination and has all the qualifications and none of the disqualifications, is entitled as a matter of right to the issuance of a certificate of registration or a physicians license, which right is enforceable by mandamus.[23] Petitioners filed a Reply[24] and both memoranda[25] reiterating their arguments.

parties

filed

their

respective

The Court denies the petition for lack of merit. There is no question that a license to practice medicine is a privilege or franchise granted by the government.[26] It is a right that is earned through years of education and training, and which requires that one must first secure a license from the state through professional board examinations.[27] Indeed,

[T]he regulation of the practice of medicine in all its branches has long been recognized as a reasonable method of protecting the health and safety of the public. That the power to regulate and control the practice of medicine includes the power to regulate admission to the ranks of those authorized to practice medicine, is also well recognized. Thus, legislation and administrative regulations requiring those who wish to practice medicine first to take and pass medical board examinations have long ago been recognized as valid exercises of governmental power. Similarly, the establishment of minimum medical educational requirements i.e., the completion of prescribed courses in a recognized medical school for admission to the medical profession, has also been sustained as a legitimate exercise of the regulatory authority of the state.[28]

It must be stressed however that the power to regulate the exercise of a profession or pursuit of an occupation cannot be exercised by the State or its agents in an arbitrary, despotic, or oppressive manner. A political body which regulates the exercise of a particular privilege has the authority to both forbid and grant such privilege in accordance with certain conditions. As the legislature cannot validly bestow an arbitrary power to grant or refuse a license on a public agency or officer, courts will generally strike down license legislation that vests in public officials discretion to grant or refuse a license to carry on some ordinarily lawful business, profession, or activity without prescribing definite rules and conditions for the guidance of said officials in the exercise of their power.[29] R.A. No. 2382 otherwise known as the Medical Act of 1959 states in Section 9 thereof that: Section 9. Candidates for Board Examinations.- Candidates examinations shall have the following qualifications:

for

Board

1. He shall be a citizen of the Philippines or a citizen of any foreign country who has submitted competent and conclusive documentary evidence, confirmed by the Department of Foreign Affairs, showing that his countrys existing laws permit citizens of the Philippines to practice medicine under the same rules and regulations governing citizens thereof; xxxx

Presidential Decree (P.D.) No. 223[30] also provides in Section (j) thereof that:

j) The [Professional Regulation] Commission may, upon the recommendation of the Board concerned, approve the registration of and authorize the issuance of a certificate of registration with or without examination to a foreigner who is registered under the laws of his country: Provided, That the requirement for the registration or licensing in said foreign state or country are substantially the same as those required and contemplated by the laws of the Philippines and that the laws of such foreign state or country allow the citizens of the Philippines to practice the profession on the same basis and grant the same privileges as the subject or citizens of such foreign state or country: Provided, finally, That the applicant shall submit competent and conclusive documentary evidence, confirmed by the Department of Foreign Affairs, showing that his country's existing laws permit citizens of the Philippines to practice the profession under the rules and regulations governing citizens thereof. The Commission is also hereby authorized to prescribe additional requirements or grant certain privileges to foreigners seeking registration in the Philippines if the same privileges are granted to or some additional requirements are required of citizens of the Philippines in acquiring the same certificates in his country; xxxx

As required by the said laws, respondent submitted a copy of the Medical Practitioners Law of Japan, duly authenticated by the Consul General of the Embassy of the Philippines in Japan, which provides in Articles 2 and 11, thus: Article 2. Anyone who wants to be medical practitioner must pass the national examination for medical practitioner and get license from the Minister of Health and Welfare. xxxx Article 11. No one can take the National Medical Examination except persons who conform to one of the following items: 1. Persons who finished regular medical courses at a university based on the School Education Laws (December 26, 1947) and graduated from said university. 2. Persons who passed the preparatory test for the National Medical Examination and practiced clinics and public sanitation more than one year after passing the said test. 3. Persons who graduated from a foreign medical school or acquired medical practitioner license in a foreign country, and also are recognized to have the same or more academic ability and techniques as persons stated in item 1 and item 2 of this article.[31]

Petitioners argue that while the Medical Practitioners Law of Japan allows foreigners to practice medicine therein, said document does not show that conditions for the practice of medicine in said country are practical and attainable by a foreign applicant; and since the requirements are practically impossible for a Filipino to comply with, there is no reciprocity between the two countries, hence, respondent may not be granted license to practice medicine in the Philippines. The Court does not agree. R.A. No. 2382, which provides who may be candidates for the medical board examinations, merely requires a foreign citizen to submit competent and conclusive documentary evidence, confirmed by the Department of Foreign Affairs (DFA), showing that his countrys existing laws permit citizens of the Philippines to practice medicine under the same rules and regulations governing citizens thereof. Section (j) of P.D. No. 223 also defines the extent of PRC's power to grant licenses, i.e., it may, upon recommendation of the board, approve the registration and authorize the issuance of a certificate of registration with or without examination to a foreigner who is registered under the laws of his country, provided the following conditions are met: (1) that the requirement for the registration or licensing in said foreign state or country are substantially the same as those required and contemplated by the laws of the Philippines; (2) that the laws of such foreign state or country allow the citizens of the Philippines to practice the profession on the same basis and grant the same privileges as the subject or citizens of such foreign state or country; and (3) that the applicant shall submit competent and conclusive documentary evidence, confirmed by the DFA, showing that his country's existing laws permit citizens of the Philippines to practice the profession under the rules and regulations governing citizens thereof. The said provision further states that the PRC is authorized to prescribe additional requirements or grant certain privileges to foreigners seeking registration in the Philippines if the same privileges are granted to or some additional requirements are required of citizens of the Philippines in acquiring the same certificates in his country.

Nowhere in said statutes is it stated that the foreign applicant must show that the conditions for the practice of medicine in said country are practical and attainable by Filipinos. Neither is it stated that it must first be proven that a Filipino has been granted license and allowed to practice his profession in said country before a foreign applicant may be given license to practice in the Philippines. Indeed, the phrase used in both R.A. No. 2382 and P.D. No. 223 is that: [T]he applicant shall submit] competent and conclusive documentary evidence, confirmed by the Department of Foreign Affairs, showing that his country's existing laws permit citizens of the Philippines to practice the profession [of medicine] under the [same] rules and regulations governing citizens thereof. x x x (Emphasis supplied)

It is enough that the laws in the foreign country permit a Filipino to get license and practice therein. Requiring respondent to prove first that a Filipino has already been granted license and is actually practicing therein unduly expands the requirements provided for under R.A. No. 2382 and P.D. No. 223. While it is true that respondent failed to give details as to the conditions stated in the Medical Practitioners Law of Japan -- i.e., the provisions of the School Educations Laws, the criteria of the Minister of Health and Welfare of Japan in determining whether the academic and technical capability of foreign medical graduates are the same as or better than that of graduates of medical schools in Japan, and who can actually qualify to take the preparatory test for the National Medical Examination respondent, however, presented proof that foreigners are actually practicing in Japan and that Filipinos are not precluded from getting a license to practice there. Respondent presented before the trial court a Japanese Government publication, Physician-Dentist-Pharmaceutist Survey, showing that there are a number of foreign physicians practicing medicine in Japan.[32] He also presented a letter dated January 28, 1992 from Consul General Yabes,[33] which states: Sir: With reference to your letter dated 12 January 1993, concerning your request for a Certificate of Confirmation for the purpose of establishing a reciprocity with Japan in the practice of medical profession relative to the case of Mr. Yasuyuki Ota, a Japanese national, the Embassy wishes to inform you that

inquiries from the Japanese Ministry of Foreign Affairs, Ministry of Health and Welfare as well as Bureau of Immigration yielded the following information: 1. They are not aware of a Filipino physician who was granted a license by the Japanese Government to practice medicine in Japan; 2. However, the Japanese Government allows a foreigner to practice medicine in Japan after complying with the local requirements such as holding a valid visa for the purpose of taking the medical board exam, checking the applicant's qualifications to take the examination, taking the national board examination in Japanese and filing an application for the issuance of the medical license. Accordingly, the Embassy is not aware of a single Filipino physician who was issued by the Japanese Government a license to practice medicine, because it is extremely difficult to pass the medical board examination in the Japanese language. Filipino doctors here are only allowed to work in Japanese hospitals as trainees under the supervision of a Japanese doctor.On certain occasions, they are allowed to show their medical skills during seminars for demonstration purposes only. (Emphasis supplied) Very truly yours, Jesus I. Yabes Minister Counsellor & Consul General

From said letter, one can see that the Japanese Government allows foreigners to practice medicine therein provided that the local requirements are complied with, and that it is not the impossibility or the prohibition against Filipinos that would account for the absence of Filipino physicians holding licenses and practicing medicine in Japan, but the difficulty of passing the board examination in the Japanese language. Granting that there is still no Filipino who has been given license to practice medicine in Japan, it does not mean that no Filipino will ever be able to be given one. Petitioners next argue that as held in De Guzman, its power to issue licenses is discretionary, hence, not compellable by mandamus. The Court finds that the factual circumstances of De Guzman are different from those of the case at bar; hence, the principle applied therein should be viewed differently in this case. In De Guzman, there were doubts about the integrity and validity of the test results of the examinees from a particular school which

garnered unusually high scores in the two most difficult subjects. Said doubts called for serious inquiry concerning the applicants satisfactory compliance with the Board requirements.[34] And as there was no definite showing that the requirements and conditions to be granted license to practice medicine had been satisfactorily met, the Court held that the writ of mandamus may not be granted to secure said privilege without thwarting the legislative will.[35] Indeed, to be granted the privilege to practice medicine, the applicant must show that he possesses all the qualifications and none of the disqualifications. It must also appear that he has fully complied with all the conditions and requirements imposed by the law and the licensing authority.[36] In De Guzman itself, the Court explained that: A careful reading of Section 20[37] of the Medical Act of 1959 discloses that the law uses the word shall with respect to the issuance of certificates of registration. Thus, the petitioners [PRC] shall sign and issue certificates of registration to those who have satisfactorily complied with the requirements of the Board. In statutory construction the term shall is a word of command. It is given imperative meaning. Thus, when an examinee satisfies the requirements for the grant of his physician's license, the Board is obliged to administer to him his oath and register him as a physician, pursuant to Section 20 and par. (1) of Section 22 of the Medical Act of 1959.[38]

In this case, there is no doubt as to the competence and qualifications of respondent. He finished his medical degree from Bicol Christian College of Medicine. He completed a one-year post graduate internship training at the Jose Reyes Memorial Medical Center, a government hospital. Then he passed the Medical Board Examinations which was given on August 8, 1992 with a general average of 81.83, with scores higher than 80 in 9 of the 12 subjects. In fine, the only matter being questioned by petitioners is the alleged failure of respondent to prove that there is reciprocity between the laws of Japan and the Philippines in admitting foreigners into the practice of medicine. Respondent has satisfactorily complied with the said requirement and the CA has not committed any reversible error in rendering its Decision dated November 16, 2004 and Resolution dated October 19, 2003. WHEREFORE, the petition is hereby DENIED for lack of merit.

SO ORDERED.

G.R. No. L-32049 June 25, 1984 MATAAS NA LUPA TENANTS ASSOCIATION, INC., NICOLAS AGLABAY, and Those Mentioned in Annex "A" of Complaint, petitioners, vs. CARLOS DIMAYUGA and JULIANA DIEZ Vda. de GABRIEL, respondents. Ramon Gonzales for petitioners. The Solicitor General and Magno T. Bueses for respondents.

MAKASIAR, J.: This petition for review on certiorari presents for review the order dated October 30, 1969 of the defunct Court of First Instance of Manila, Branch IV, which granted the motion to dismiss the complaint of petitioners in Civil Case No. 75391 on the ground that the same failed to state a cause of action (p. 16, rec.; pp. 1, 100, CFI rec.). The undisputed facts are as follows: On January 17, 1969, petitioners filed a complaint for the exercise of preferential rights with the then Court of First Instance of Manila, Branch IV, docketed as Civil Case No. 75391 (p. 32, rec.; p. 1, CFI rec.). The said complaint alleged that petitioner association has for its members Nicolas Aglabay, et al., named and listed in Annex "A" of said complaint, which members are heads of 110 tenant families, and who have been, for more than ten years prior to 1959, occupants of a parcel of land (with their 110 houses built thereon), formerly owned by the respondent, Juliana Diez Vda. de Gabriel, to whom petitioners have been paying rents for the lease thereof, but who, on May 14, 1968, without notice to petitioners, sold the same to respondent Carlos Dimayuga, who, in turn, mortgaged the same to her for the balance of the purchase price; that according to Republic Act 1162, as amended by Republic Act 2342, a parcel of land in Manila and suburbs, with at least fifty (50) houses of tenants erected thereon and actually leased to said tenants for at least ten (10) years prior to June 20, 1959, may not be sold by the landowner to any person other than such tenants, unless the latter renounced their rights in a public instrument; that without said tenants-appellants having renounced their preferential rights in public instrument, respondent Vda. de Gabriel sold the land to respondent Dimayuga; that petitioners-tenants are willing to purchase said land at the same price and on the same terms and conditions observed in the contract of sale with respondent Dimayuga; and that since aforesaid contract of sale is expressly prohibited by law, the same is null and void, while it is mandatory for respondent Vda. de Gabriel to execute such sale to petitioners, Petitioners therefore prayed that said contract of sale be declared void, and that respondent Vda. de Gabriel be ordered to execute a deed of sale in favor of petitioners at the same price and conditions followed in the contract with respondent Dimayuga, plus attorney's fees and damages (p. 32, rec.; p. 1, CFI rec.). On January 31, 1969, respondent Vda. de Gabriel filed a motion to dismiss on the ground that the complaint stated no cause of action because the land subject of the complaint is not a landed estate, and not being such, the same cannot be expropriated, and not being expropriable, no preferential rights could be availed of by the tenants (p. 41, rec.; p. 22, CFI rec.).

Respondent Dimayuga filed his answer to aforesaid complaint on February 6, 1969 admitting therein certain factual allegations, denied some averments, interposed the affirmative defenses that plaintiffs had no personality to initiate the action since the Land Tenure Administration possessed the power to institute the proper expropriation proceedings before the competent court and that the subject complaint stated no cause of action against respondent, alleged a counterclaim to eject plaintiffs from the property, and prayed for the dismissal of the complaint and other remedies (p. 44, rec.; p. 155, CFI rec.). lwphl @itç

On February 6, 1969, plaintiffs-petitioners filed their opposition to the motion to dismiss, maintaining, among others, that Republic Act 1162, as amended by Republic Act 2342 (law which respondent Vda. de Gabriel invoked), does not necessarily refer to landed estates, but to any piece of land occupied by more than 50 families leasing the same for more than ten (10) years prior to June 20, 1959; that their preferential rights are independent of the expropriability of the land; that therefore, said rights may be exercised even if the land is not expropriable; and that these rights were granted pursuant to the police power of the State for the general welfare, with prayer that aforesaid motion to dismiss be denied (p. 47, rec.; p. 26, CFI rec.). On February 13, 1969, respondent Vda. de Gabriel replied to the aforesaid opposition to motion to dismiss, reiterating therein her prayer to dismiss the complaint (p. 57, rec.; p. 38, CFI rec.). Plaintiffs-petitioners filed their rejoinder to above reply to their opposition on February 19, 1969, laying emphasis on the alleged distinction between the two ways of acquiring occupied land under Republic Act 1162, which are expropriation and voluntary disposal of the land by the owner thereof, and which are exercisable independently of each other (p. 56, rec.; p. 42, CFI rec.). On October 30, 1969, Branch IV of the Court of First Instance of Manila issued the subject order which found respondent's motion to dismiss well-taken and thereby dismissed the complaint (p. 69, rec.; p. 100, CFI rec.). Petitioners moved for reconsideration of the aforecited order on January 7, 1970, which motion was denied in the lower court's order of January 27, 1970 (p. 111, 190, CFI rec.). On February 9, 1970, petitioners filed a notice of appeal with the lower court to which respondent Vda. de Gabriel moved for dismissal of the same on February 11, 1970 on the alleged ground that pursuant to Republic Act 5440, petitioners should have appealed from the questioned order by way of a petition for certiorari to this Court since the matter involved only errors or questions of law (p. 143, CFI rec.). After a series of motions, reply, rejoinder, sur-rejoinder, and answer between both parties, the lower court issued its order of May 11, 1970 dismissing petitioners' appeal (p. 225, CFI rec.). Petitioners thus resorted to this petition. Petitioners contend that the lower court committed an error in dismissing their complaint on the ground that since the land is not expropriable, it follows that the tenants therein have no preferential rights to buy said land, if the same is sold voluntarily. Petitioners' contention is anchored on the amendment introduced by Republic Act 3516 into Section 1 of Republic Act 1162, which latter law had been invoked in the decision of the lower court. According to petitioners, the phrase "any landed estates or haciendas herein authorized to be expropriated" had been amended to read "any landed estates or haciendas or lands herein

authorized to be expropriated"; hence, Republic Act 1162 does not refer exclusively to landed estates or haciendas, but even to smaller lands. The particular section as amended reads thus: The expropriation of landed estates or haciendas, or lands which formerly formed part thereof, or any piece of land in the City of Manila, Quezon City and suburbs, which have been and are actually being leased to tenants for at least ten years, is hereby authorized: Provided, That such lands shall have at least forty families of tenants thereon. (Sec. 1 of R. A. 3516). Petitioners likewise invoke the amended title of Republic Act 1162 which had been introduced by Republic Act 2342 which title now reads as follows: An Act Providing for the Expropriation of Landed Estates or Haciendas or Lands Which Formerly Formed Part Thereof or Any Piece of Land in the City of Manila, Quezon City and Suburbs, Their Subdivision into Small Lots, and the Sale of Such Lots at Cost or Their Lease on Reasonable Terms, and for Other Purposes (emphasis supplied). Petitioners further allege that Republic Act 1162 is both an exercise of the power of eminent domain and the police power of the State. The exercise of the police power of the State refers to the grant of preferential rights to the tenants of such land, if the same is disposed of voluntarily. Simply stated, petitioners theorize that Republic Act 1162 covers both compulsory and voluntary sale; hence, while expropriability is pertinent to compulsory sale, the same does not relate to voluntary sale. Even if the land is not expropriable, if the same is however actually leased to the occupants for more than ten years prior to May 22, 1963 (when R.A. 3516 took effect) with at least 40 families, said land, if sold voluntarily, is subject to the preferential rights of the tenants. Respondent Vda. de Gabriel maintains, on the other hand, that there is no more issue regarding the non-expropriability of subject land, which condition or status was expressly admitted by petitioners in the lower court; that the title of Republic Act 1162, as amended by Republic Act Nos. 2342 and 3516 clearly embraces expropriation; that the prohibitive acts enumerated in Section 5 of R.A. 1162, as amended, are entirely dependent on the expropriability of the land in controversy; that there is nothing in the aforecited law which validly supports the alleged preferential right of petitioners to purchase the property at the same price and under the same conditions; that the only reasonable interpretation of the opening lines of Section 5 of Republic Act 1162, as amended, is that pending expropriation, the landowner shall not sell the land to any other person than the tenant or occupant unless the latter renounces his rights in a public instrument; but if the land is not expropriable, as petitioners have admitted, the prohibition does not apply; and that clearly, from the provision of Section 6 of the amended law, Section 5 thereof may be violated only if the land is "herein authorized to be expropriated" and since petitioners have admitted the non-expropriability of subject land, it necessarily follows that said Section 5 cannot apply. Respondent Dimayuga avers that Section 9, in relation to the title of R.A. 1162, clearly provides that the preferential right could be exercised only when the land under question is subject to expropriation, or better still, if the tenanted property which formerly formed part of an hacienda or is a landed estate, had been expropriated; and, that R.A. 1162, as amended, embraces only landed estates or haciendas with an extensive area. The sole issue raised by petitioners is whether or not they have the pre-emptive or preferential rights to buy the land in question. WE find for petitioners.

I The third proviso in Section 5 of Republic Act 3516, which law further amended R.A. 1162, reads: Provided, furthermore, That no lot or portion thereof actually occupied by a tenant or occupant shall be sold by the landowner to any other person than such tenant or occupant, unless the latter renounce in a public instrument his rights under this Act: Provided, finally, That if there shall be tenants who have constructed bona fide improvements on the lots leased by them, the rights of these tenants should be recognized in the sale or in the lease of the lots, the limitation as to area in Section three notwithstanding. The provision clearly defines the preferential right of herein petitioners to buy the parcel of land. It should be noted that respondent Vda. de Gabriel voluntarily sold the land to respondent Dimayuga without informing the petitioners of the transaction. Respondent Vda. de Gabriel did not give the first offer to petitioners who were then tenants-lessees and who would have either accepted or refused to buy the land in a public 7 document. The fact is that on discovery of the sale to respondent dent Dimayuga, petitioners filed their original claim for preferential rights eight months after the clandestine sale. Thus, the condition set forth in the aforesaid proviso — that of offering first the sale of the land to petitioners and the latter's renunciation in a public instrument — were not met when the land was sold to respondent Dimayuga. Evidently, said sale was made illegally and, therefore, void. Petitioners have still the first option to buy the land as provided for in the above provision. II A brief run down of this Court's decisions easily reveal the adherence to the principle that the test for a valid expropriation of private land for resale to its occupants, is the number of families to be benefited thereby, and not the area. In his book on Constitutional Law, Dean Isagani A. Cruz recapitulates thus: In the earlier case of Rural Progress Administration v. Reyes, the Supreme Court held that the criterion for determining the validity of expropriation under this provision was not the area of the land sought to be taken but the number of people intended to be benefited thereby. The land, in other words, could be small provided it was tenanted by a sizable number of people. This ruling was abandoned in the case of Guido v. Rural Progress Administration where the Supreme Court declared, also by a split decision as in the Reyes case, that the test to be applied was the area of the land and not the number of people who stood to be benefitted by the expropriation. The land should be a landed estate or one comprising a very fast area. It was stressed that one of the purposes of the framers was precisely to break up these estates in the hands of only a few individuals or families and thus more equitably distribute them along the landless. xxx xxx xxx It has also been held that where a landed estate is broken up into reasonable portions which are thereafter sold to separate purchasers, the resultant portions cannot be deemed as still subject to expropriation under this provision simply because they used to form part of a landed estate.

In the case of Tuason v. PHHC, which was a petition for prohibition to nullify a law directing the expropriation of Tatalon Estate in Quezon City, Justice Fernando suggested a ruling to the Reyes ruling arguing that the propriety of expropriation "could not be determined on a purely quantitative or area basis," quoting from Justice J.B.L. Reyes in his dissenting opinion in the Baylosis Case. ... (p 71,1983 Ed.; emphasis supplied). From the Reyes case where the number of beneficiaries test was applied in determining public use down to the Guido and Baylosis cases where the land or area size test was invoked, then to the Tuason case where a return to the Reyes decision was made and then up to the recent case of Pulido vs. Court of Appeals (L-57625, May 3, 1983; 122 SCRA 63) where this Court found it "unfortunate that petitioner would be deprived of his land holdings, but his interest and that of his family should not stand in the way of progress and the benefit of the greater majority of the inhabitants of the country," there has evolved a clear pattern of adherence to the "number of people to be benefited test. This is made more manifest by the new constitutional provisions on the equitable diffusion of property ownership and profits (Sec. 6, Art. 11) and the implementation of an agrarian reform program aimed at emancipating the tenant from the bondage of the soil (Sec. 12, Art. XIV). It has been noted with concern that while respondents raised the issue of expropriability of the parcel of land, petitioners limited themselves to the issue of preferential or pre-emptive rights. What petitioners might have failed to realize is that had they invoked the expropriability of subject land, they would have had a foolproof case. Right from the start, they would have had the upper hand. Ironically, however, instead of anchoring their case on the expropriability of such land, they concentrated on asserting their preferential right to buy the land. For, Section 1 of R.A. 1162, as amended by R.A. 3516, specifically authorizes the expropriation of any piece of land in the City of Manila, Quezon City and suburbs which have been and are actually being leased to tenants for at least ten (10) years, provided said lands have at least forty families of tenants thereon. The case at bar comes within the coverage of the aforesaid legal provision since the parcel of land is located in Manila which was then actually leased to 110 tenant families 20 years prior to the commencement of this action in the lower court. Clearly, therefore, the land in question is capable of expropriation. The above situation now brings Us back to the case of J.M. Tuason & Co. vs. Land Tenure Administration (L-21064, Feb. 18, 1970, 31 SCRA 413-417) where this Court laid down certain basic doctrines on the power of eminent domain. Thus, this Court, speaking thru then Justice Fernando, declared: It does not admit of doubt that the congressional power conferred by the Constitution is far from limited. It is left to the legislative will to determine what lands may be expropriated so that they could be subdivided for resale to those in need of them. Nor can it be doubted either that as to when such authority may be exercised is purely for Congress to decide. Its discretion on the matter is not to be interfered with. This is shown by reference to the historical basis of the provision as reflected in the proceedings of the Constitutional Convention. Historical discussion while valuable is not necessarily decisive. It is easy to understand why. The social and economic conditions are not static. They change with the times. To Identify the text of a written constitution with the circumstances that inspired its inclusion may render it incapable of being responsive to future needs. Precisely, it is assumed to be one of the virtues of a written constitution that it

suffices to govern the life of the people not only at the time of its framing but far into the indefinite future. It is not to be considered as so lacking in flexibility and suppleness that it may be a bar to measures, novel and unorthodox, as they may appear to some, but nonetheless imperatively called for. xxx xxx xxx The framers of the Constitution were seriously concerned with the grave problems of inequality of wealth, with its highly divisive tendency, resulting in the generous scope accorded the police power and eminent domain prerogatives of the state, even if the exercise thereof would cover terrain of property right previously thought of as beyond state control, to promote social justice and the general welfare. As in the case of the more general provision on eminent domain, the power to expropriate lands under Sec. 4 of Art. XIII of the Constitution requires the payment of just compensation, the taking to be for the public use, and to meet the exacting standard of due process and equal protection guaranty of the Constitution. xxx xxx xxx The power granted to Congress by the Constitution to "authorize, upon payment of just compensation, the expropriation of lands to be subdivided into small lots and conveyed at cost to individuals" is unlimited by any other provision of the Constitution. Just compensation is in reality a part of the power granted rather than a limitation thereto, just as just compensation is of the essence in any exercise of the power of eminent domain as, otherwise it would be plain commandeering. While the taking must be for public use as a matter of principle, in the judicial proceeding, the Government need not present evidence of such public use as a fact. The constitutional provision itself declares the public objective, purpose or use of the expropriation contemplated, hence, it should follow that as long as a congressional legislation declares that the condemnation of a particular land is for the specific purpose stated in the Constitution, it is not for the judiciary to inquire as to whether or not the taking of such land is for public use. The Constitution itself which is supposed to be the supreme law on private property rights declares it to be so, and leaves it to Congress, not to the judiciary, to make the choice of the lands to be taken to attain the objective the constituent assembly aimed to achieve. The scope and the limit of the power of the judiciary in this regard is only to determine the existence of enabling legislation, to see to it that the facts are as contemplated in such enabling act and to provide the vehicle for compliance with procedural due process in the implementation of the congressional act. On the matter of taking for public use, Chief Justice Fernando summarily observed: The taking to be valid must be for public use. There was a time when it was felt that a literal meaning should be attached to such a requirement. Whatever project is undertaken must be for the public to enjoy, as in the case of streets or parks. Otherwise, expropriation is not allowable. It is not so any more. As long as the purpose of the taking is public, then the power of eminent domain comes into play. As just noted, the Constitution in at least two cases, to remove any doubt, determines what is public use One is the expropriation of lands to be subdivided into small lots for resale at cost to individuals. The other is the transfer, through the

exercise of this power, of utilities and other private enterprise to the government. It is accurate to state then that at present whatever may be beneficially employed for the general welfare satisfies the requirement of public use (The Constitution of the Philippines, 2nd Ed., 1977, pp. 523-24). III This preferential right of petitioners and the power of eminent domain have been further mandated, strengthened and expanded by recent developments in law and jurisprudence. It must be recalled that the 1973 Constitution embodies certain original and innovative provisions on eminent domain. The new Constitution provides thus: Private property shall not be taken for public use without just compensation" (Sec. 2, Art. IV). The Batasang Pambansa may authorize, upon payment of just compensation, the expropriation of private lands to be subdivided into small lots and conveyed at cost to deserving citizens (See. 13, Art. XIV). The State shall promote social justice to ensure the dignity, welfare and security of affirmatively the people. Toward this end, the State shall regulate the acquisition Ownership, use, enjoyment and disposition of private property, and equitably diffuse property ownership and profits (Sec. 6, Art. 11; emphasis supplied). The State shall establish, maintain, and ensure adequate social services in the field of education, health, housing, employment, welfare, and social security to guarantee the enjoyment by the people of a decent standard of living (Sec. 7, Art. 11). The State shall formulate and implement an agrarian reform program aimed at emancipating the tenant from the bondage of the soil and achieving the goals enunciated in this Constitution (Sec. 12, Art. XIV). The aforequoted Section 6 of Article 11, which is a modified version of the original provision of the 1935 Constitution, "emphasizes the stewardship concept, under which private property is supposed to be held by the individual only as a trustee for the people in general, who are its real owners. As a mere steward, the individual must exercise his rights to the proper- 4 ty not for his own exclusive and selfish benefit but for the good of the entire community or nation" (p. 70, Phil. Political Law, Cruz, 1983 ed.). lwphl @itç

In the case of Almeda vs. Court of Appeals, et al. (L-43800, 78 SCRA 194 [July 29, 1977]), this Court thus declared: It is to be noted that under the new Constitution, property ownership is impressed with social function. Property use must not only be for the benefit of the owner but of society as well. The State, in the promotion of social justice, may "regulate the acquisition, ownership, use, enjoyment and disposition of private property, and equitably diffuse property — ownership and profits." One governmental policy of recent date projects the emancipation of tenants from the bondage of the soil and the transfer to them of the ownership of the land they till.

"The Legislature may regulate 'the acquisition, ownership, use, enjoyment and disposition of private property,' to the end that maximum advantage can be derived from it by the people as a whole. Thus, it may limit the size of private landholdings, impose higher taxes on agricultural lands that are not being tilled, or provide for a wider distribution of land among the landless. ... (p. 70, Phil. Political Law, Cruz, 1983 ed.). It is obviously in the spirit of Sections 6 and 7 of Article 11 that P.D. No. 1517 on urban land reform was enacted and the subsequent implementing Proclamation No. 1967 was issued. Significantly also, the latest amendment to the Constitution on urban land reform and social housing program which has been proclaimed by the President as having been approved in the recent plebiscite on January 17, 1984 all the more emphasizes and strengthens the constitutional base for urban land reform consistent with the provisions on social justice. Even as we have consistently and explicitly pronounced that the power of eminent domain is a basic and inherent power of government which does not have to be spelled out by the Constitution, still our legislators felt such urgent demands for redistribution of land in this country that they had to incorporate into the 1935 and 1973 Constitutions a specific provision on expropriation of land for resale. Section 13, Article IV of the 1973 Constitution specially authorizes the expropriation of private lands for resale. Thus, as earlier mentioned, P.D. No. 1517 entitled "Proclaiming Urban Land Reform in the Philippines and Providing for the Implementing Machinery Thereof" was enacted and beer effective on June 1 1, 1978 and Proclamation No. 1967 was issued on May 14, 1980 as an implementing law. This decree, which is firmly based on Section 6, Article 11 of the new Constitution, undoubtedly adopts and crystallizes the greater number of people criterion when it speaks of tenants and residents in declared urban land reform zones or areas without any mention of the land area covered by such zones. The focus, therefore, is on people who would stand to benefit and not on the size of the land involved. It should now be clarified that Section 22 of the aforecited decree declares thus: Sec. 22. Repealing Clause. — All laws, decrees, executive orders, rules and regulations inconsistent herewith are hereby repealed, amended or modified accordingly. The decree has, therefore, superseded R.A. Nos. 1162, 2342 and 3516. The issue of pre-emptive or preferential rights still remains for Our resolution within the purview of the said decree. The pertinent provisions of P.D. No. 1517 are as follows: Sec. 4. Proclamation of Urban Land Reform Zones. — The President shall proclaim specific parcels of urban and urbanizable lands as Urban Land Reform Zones, otherwise known as Urban Zones for purposes of this Decree, which may include Bagong Lipunan Sites, as defined in P.D. 1396 (par. 1 of the section). xxx xxx xxx Sec. 6. Land Tenancy in Urban Land Reform A Teas. Within the Urban Zones legitimate tenants who have resided on the land for ten years or more who have built

their homes on the land and residents who have legally occupied the lands by contract, continuously for the last ten years shall not be dispossessed of the land and shall be allowed the right of first refusal to purchase the same within a reasonable time and at reasonable prices, under terms and conditions to be determined by the Urban Zone Expropriation and Land Management Committee created by Section 8 of the Decree. xxx xxx xxx Sec. 9. Compulsory Declaration of Sale and Preemptive Rights. Upon the proclamation by the President of an area as an Urban Land Reform Zone, all landowners, tenants and residents thereupon are required to declare to the Ministry any proposal to sell, lease or encumber lands and improvements thereon, including the proposed price, rent or value of encumbrances and secure approval of said proposed transactions. The Ministry shag have the pre-emptive right to acquire the above-mentioned lands and improvements thereon which shall include, but shag not be limited to lands occupied by tenants as provided for in Section 6 of this Decree (emphasis supplied). Pursuant to the above decree and for purposes of making specific the applicability of the same and other subsequent laws on the matter, the President issued Proclamation No. 1967 dated May 14, 1980 declaring Metropolitan Manila Area as Urban Land Reform Zone. Thus, on page 2, No. 14 of said proclamation, Mataas na Lupa, the land in controversy, (an area bounded on the northwest by Quirino Avenue, South Superhighway on the east, San Andres Street on the south, and on the west, by Anak Bayan Street) was declared as an area for priority development and urban land reform zone. The aforequoted provisions of P.D. 1517 and the declaration in the aforesaid proclamation are clear and leave no room for any interpretation. Evidently, petitioners' case falls squarely within the law. Under Section 6 of the decree, the 110 tenant-families have been vested with the right of first refusal to purchase the land in question within a reasonable time and reasonable prices, subject to Ministry of Human Settlements rules and regulations. WHEREFORE, THE ORDER DATED OCTOBER 30, 1969 OF THE THEN MANILA COURT OF FIRST INSTANCE, BRANCH IV, IS HEREBY SET ASIDE AND THE MINISTRY OF HUMAN SETTLEMENTS IS HEREBY DIRECTED TO FACILITATE AND ADMINISTER THE IMPLEMENTATION OF THE RIGHTS OF HEREIN PETITIONERS. COSTS AGAINST RESPONDENTS.

.R. No. 155001. May 5, 2003 En Banc

Delegation

of

[Non-legislative power of Congress; Police Power; emergency powers]

FACTS: On October 5, 1994, AEDC submitted an unsolicited proposal to the Government through the DOTC/MIAA for the development of NAIA International Passenger Terminal III (NAIA IPT III). DOTC constituted the Prequalification Bids and Awards Committee (PBAC) for the implementation of the project and submitted with its endorsement proposal to the NEDA, which approved the project. On June 7, 14, and 21, 1996, DOTC/MIAA caused the publication in two daily newspapers of an invitation for competitive or comparative proposals on AEDC’s unsolicited proposal, in accordance with Sec. 4-A of RA 6957, as amended. On September 20, 1996, the consortium composed of People’s Air Cargo and Warehousing Co., Inc. (Paircargo), Phil. Air and Grounds Services, Inc. (PAGS) and Security Bank Corp. (Security Bank) (collectively, Paircargo Consortium) submitted their competitive proposal to the PBAC. PBAC awarded the project to Paircargo Consortium. Because of that, it was incorporated into Philippine International Airport Terminals Co., Inc. AEDC subsequently protested the alleged undue preference given to PIATCO and reiterated its objections as regards the prequalification of PIATCO. On July 12, 1997, the Government and PIATCO signed the “Concession Agreement for the Build-Operate-and-Transfer Arrangement of the NAIA Passenger Terminal III” (1997 Concession Agreement). The Government granted PIATCO the franchise to operate and maintain the said terminal during the concession period and to collect the fees, rentals and other charges in accordance with the rates or schedules stipulated in the 1997 Concession Agreement. The Agreement provided that the concession period shall be for twenty-five (25) years commencing from the in-service date, and may be renewed at the option of the Government for a period not exceeding twenty-five (25) years. At the end of the concession period, PIATCO shall transfer the development facility to MIAA. Meanwhile, the MIAA which is charged with the maintenance and operation of the NAIA Terminals I and II, had existing concession contracts with various service providers to offer international airline airport services, such as in-flight catering, passenger handling, ramp and ground support, aircraft maintenance and provisions, cargo handling and warehousing, and other services, to several international airlines at the NAIA. On September 17, 2002, the workers of the international airline service providers, claiming that they would lose their job upon the implementation of the questioned agreements, filed a petition for prohibition. Several employees of MIAA likewise filed a petition assailing the legality of the various agreements.

During the pendency of the cases, PGMA, on her speech, stated that she will not “honor (PIATCO) contracts which the Executive Branch’s legal offices have concluded (as) null and void.” ISSUE: Whether or not the State can temporarily take over a business affected with public interest. RULING: Yes. PIATCO cannot, by mere contractual stipulation, contravene the Constitutional provision on temporary government takeover and obligate the government to pay “reasonable cost for the use of the Terminal and/or Terminal Complex.” Article XII, Section 17 of the 1987 Constitution provides: Section 17. In times of national emergency, when the public interest so requires, the State may, during the emergency and under reasonable terms prescribed by it, temporarily take over or direct the operation of any privately owned public utility or business affected with public interest. The above provision pertains to the right of the State in times of national emergency, and in the exercise of its police power, to temporarily take over the operation of any business affected with public interest. The duration of the emergency itself is the determining factor as to how long the temporary takeover by the government would last. The temporary takeover by the government extends only to the operation of the business and not to the ownership thereof. As such the government is not required to compensate the private entity-owner of the said business as there is no transfer of ownership, whether permanent or temporary. The private entityowner affected by the temporary takeover cannot, likewise, claim just compensation for the use of the said business and its properties as the temporary takeover by the government is in exercise of its police power and not of its power of eminent domain. Article XII, section 17 of the 1987 Constitution envisions a situation wherein the exigencies of the times necessitate the government to “temporarily take over or direct the operation of any privately owned public utility or business affected with public interest.” It is the welfare and interest of the public which is the paramount consideration in determining whether or not to temporarily take over a particular business. Clearly, the State in effecting the temporary takeover is exercising its police power. Police power is the “most essential, insistent, and illimitable of powers.” Its exercise therefore must not be unreasonably hampered nor its exercise be a source of obligation by the government in the absence of damage due to arbitrariness of its exercise. Thus, requiring the government to pay reasonable compensation for the reasonable use of the property pursuant to the operation of the business contravenes the Constitution.

A summary of the Agan vs. Piatco case. Warning, though: this is a first year law student’s summary of the case. It may not be correct. Portions were also directly lifted from the decision. Title: Agan Jr. vs. Philippine International Air Terminals Co. Inc Date: – May 5, 2003 Ponente: Puno Nature of the case: Special Civil Action in the Supreme Court. Prohibition. Facts: In August 1989 the DOTC had a study conducted to determine whether or not the present Ninoy Aquino International Airport (NAIA) can cope with traffic development up to the year 2010. A draft final report was submitted to the DOTC in December 1989. Four years later, in 1993, six Filipino-Chinese business leaders met with then President Fidel Ramos to explore the possibility of investing in the construction and operation of a new international airport terminal. The six later formed the Asia’s Emerging Dragon Corp (AEDC) which submitted an unsolicited proposal for the development of NAIA International Passenger Terminal III more than a year after the first meeting with Ramos, in October 1994. In March 1995, the DOTC endorsed the AEDC proposal to the National Economic and Development Authority (NEDA). In January 1996, NEDA passed Board Resolution No. 2 which approved the NAIA IPT III project. In June 1996, the DOTC published an invitation for competitive bidding in two daily newspapers, as required by law (sec 4-A of RA6957). The alternative bidders had to submit three envelopes. The first contains the prequalification documents, the second the technical proposal, and the third the financial proposal of the proponent. The bidding was scheduled on September 20, 1996. The bid documents issued by the Prequalification Bids and Awards Committee said the proponent must have adequate capability to sustain the financing requirement for the engineering, design, construction, operation, and maintenance phases of the project. The proponent must have an equity that is at least 30% of the project cost, and be able to secure external financing for the project. Government was also guaranteed a five percent share in the gross revenue of the project for the first five years; 7.5% in the next 10 years, and 10% in the next 10 years. This would be in addition to a fixed annual guaranteed payment to the government. The basis for the prequalification shall be the proponent’s compliance with the minimum technical and financial requirements provided in the bid documents and the IRR of the BOT (build operate and transfer) Law. The bid documents allowed amendments to the draft concession agreement, but said that these should cover only items that would not materially affect the preparation of the proponent’s proposal. In August 1996, during the second pre-bid conference, the PBAC made several clarifications, upon the request of People’s Air Cargo & Warehousing Co. Inc (PAIRCARGO), which wanted to challenge the AEDC bid. The PBAC said the list of revenue sources mentioned in the bid documents were “merely indicative.” The project proponent may add other revenue sources, subject to approval by the DOTC/MIAA. Also, only fees and charges denominated as “public utility fees” would be subject to regulation, and these could still be revised, because the PBAC has a pending query with the justice department. In September 1996, PBAC issued a bid bulletin in which it said that since PAIRCARGO could not meet the required minimum equity prescribed in the bid documents, it would accept instead an audited financial statement of the financial capability of all member companies of the consortium. In September 1996, PAIRCARGO submitted their competitive proposal to the PBAC. The first envelope, containing the prequalification documents, was opened on September 23, 1996, and PBAC prequalified the PAIRCARGO consortium the following day. On September 26, AEDC filed with PBAC its reservations regarding PAIRCARGo, noting the “lack of corporate approvals and financial capability of PAIRCARGO.” For one, PAIRCARGO included in the computation of its financial capability the total net worth of Security Bank, when the Banking Law limits to 15% the total investment that a bank may make on one project. It also questioned the appointment of Lufthansa as facility operator, because Philippine laws limit to Filipinos the operation of a public utility. The PBAC, however, said on October 2, 1996 that based on the documents submitted and the prequalification criteria, PAIRCARGO was prequalified. The DOTC secretary approved PBAC’s findings. The AEDC reiterated its objections two more times. On October 16, the third envelope containing the financial proposals were opened, and PAIRCARGO had offered to pay the government higher. Both PAIRCARGO and AEDC offered to build the NAIA Passenger Terminal III for at least $350 million at no cost to the government and to pay the government a 5% share in gross revenues for the first five years of operation; a 7.5% share in gross revenues for the next 10 years of operation; and a 10% share in gross revenues for the last 10 years of operation. In addition to this, AEDC offered to pay the government P135 million as guaranteed payment for 27 years. Paircargo Consortium offered a total of P17.75 billion for the same period. PBAC informed AEDC it had accepted Paircago’s price proposal, and given AEDC 30 working days to match the bid. When AEDC failed to do so, the DOTC issued a notice on December 11 1996 regarding AEDC’s failure to match the proposal. In February 1997, Paircargo Consortium incorporated into Philippine International Airport Terminals Co Inc (PIATCO).

AEDC protested the alleged undue preference given to PIATCO and reiterated its objections regarding the prequalification of PIATCO. In April 1997, it filed before the Pasig RTC a petition for declaration of nullity of the proceedings, mandamus and injunction against the DOTC secretary, the PBAC chair and its voting members, and the chair of the PBAC technical committee. On April 17, the NEDA ICC conducted an ad referendum to facilitate the approval of the BOT agreement between the DOTC and PIATCO. Because there were only four instead of the required six signatures, the NEDA merely noted the agreement. On July 9, 1997, the DOTC issued the notice of award for the project to PIATCO. The Concession Agreement was signed on July 12, 1997, granting PIATCO the franchise to operate and maintain the NAIA Passenger Terminal III for 25 years, with an option to renew for a period not exceeding 25 years. PIATCO was allowed to collect fees, rentals and other charges in accordance with the rates or schedules in the 1997 Concession Agreement. At the end of the concession period, PIATCO will transfer the airport to MIAA. In November 1998, the government and PIATCO signed an Amended and Restated Concession Agreement (ARCA). The ARCA amended provisions on the special obligations of the government, the exclusivity of the franchise given to PIATCO, the proceeds of the insurance, the taxes, duties and other charges that may be levied PIATCO, and the provisions on the termination of the contract. Three more supplements to the ARCA were signed afterwards: in August 1999, in September 2000, and in June 2001. The first redefined revenues, required government to construct an access road connecting NAIA II and III, and added to the special obligations of government. The second supplement required government to clear structures at the construction site and to pay PIATCO for these. The third provided for PIATCO’s obligations regarding the construction of the surface road connecting Terminals II and III. In September 2002, workers of the international airline service providers filed before the Supreme Court a petition for prohibition enjoining the enforcement of the agreements. They said the transfer to NAIA III could cost them their jobs, since under the agreements, PIATCO is not required to honor MIAA’s existing concession contracts with various service providers for international airline airport services. In October 2002, the service providers filed a motion for intervention and a petition in intervention, joining the cause of the petitioning workers. Three congressmen – Salacnib Baterina, Clavel Martinez and Constantino Jaraula – filed a similar petition shortly after. In November 2002, several MIAA employees also filed a petition questioning the legality of the agreements. In December 2002, another group of congressmen – Jacinto Paras, Rafael Nantes, Eduardo Zialcita, Willie Villarama, Prospero Nograles, Prospero Pichay Jr., Harlin Cast Abayon and Benasing Macarambon – filed their comment in intervention defending the validity of the agreements and praying for the dismissal of the petitions. On December 10, 2002, the court heard the case on oral argument, the required the parties to file their respective memoranda, and to explore the possibility of arbitration as provided in the challenged contracts. In their consolidated memorandum, the Office of the Solicitor General and the Office of the Government Corporate Counsel prayed that the petitions be given due course and that the 1997 Concession Agreement, the ARCA and the supplements be declared void for being contrary to the Constitution, the BOT Law and its implementing rules and regulations. In March 2003, PIATCO commenced arbitration proceedings before the International Chamber of Commerce, International Court of Arbitration. Issue: Did the PIATCO agreements – the 1997 Concession Agreement, the ARCA, and the three supplemental agreements – violate the Constitution and the BOT Law? Held/Decision: YES. Ratio: In the first place, PIATCO was not a qualified bidder. The minimum project cost was estimated to be P9.183 billion, which meant that Paircargo Consortium had to prove it could provide at least P2.755 billion. Paircargo’s audited financial statement for 1994 showed it had a net worth of P3.123 billion, but that was because it included in the computation the total net worth of Security Bank, which as of 1995 was at P3.523 billion. Since banks are allowed to invest only 15% of it entire net worth, Security Bank could only invest P528 million, which brings down Paircargo’s equity to P558.384 million, or only 6% of the project cost. “Disregarding the investment ceilings provided by applicable law would not result in a proper evaluation of whether or not a bidder is pre-qualified to undertake the project as for all intents and purposes, such ceiling or legal restriction determines the true maximum amount which a bidder may invest in the project…If the maximum amount of equity that a bidder may invest in the project at the time the bids are submitted falls short of the minimum amounts required to be put up by the bidder, said bidder should be properly disqualified…we hold that Paircargo Consortium was not a qualified bidder. Thus the award of the contract by the PBAC to the Paircargo consortium is null and void.” Other issues: 1. The signed agreement was different from the draft that was bidded on. Under the law, substantial changes require another bidding. The three principles in public bidding are: an offer to the public; opportunity for competition; and

basis for the exact comparison of bids. Changing the parameters would change the agreement, which might have changed the technical and financial parameters of other bidders had they known that such terms were available. The 1997 Concession Agreement signed between PIATCO and the government was substantially different from the draft concession agreement that was bidded on. While it was a draft and was expected to amended from time to time, the PBAC bid bulletin also said that the amendments should only cover items that would not materially affect the preparation of the proposal. The changes should not be substantial or material enough to alter the basic parameters of the contract, and constitute a denial to the other bidders of the opportunity to bid on the same terms. There were two main differences between the draft agreement and the one that was signed. These concerned the fees that may be imposed and collected by PIATCO, and the extent of control and regulation that MIAA has over the fees that PIATCO will charge. The draft agreement classified aircraft parking and tacking fees, groundhandling fees, rentals and airlines offices, check-in counter rentals and porterage fees under those that are regulated – subject to periodic adjustment of once every two years and in accordance to a certain formula. The signed agreement said fees subject to MIAA approval are “public utility fees” and took out groundhandling and rentals and airlines offices from the list. There was also “an obvious relaxation of the extent of control and regulation by MIAA with respect to the particular fees that may be charged by PIATCO.” The signed agreement also allowed PIATCO to charge in US dollars, while paying the government in pesos. “When taken as a whole, the changes under the 1997 Concession Agreement with respect to reduction in the types of fees that are subject to MIAA regulation and the relaxation of such regulation with respect to other fees are significant amendments that substantially distinguish the draft Concession Agreement from the 1997 Concession Agreement. The 1997 Concession Agreement, in this respect, clearly gives PIATCO more favorable terms than what was available to other bidders at the time the contract was bidded out. It is not very difficult to see that the changes in the 1997 Concession Agreement translate to direct and concrete financial advantages for PIATCO which were not available at the time the contract was offered for bidding. It cannot be denied that under the 1997 Concession Agreement only “Public Utility Revenues” are subject to MIAA regulation. Adjustments of all other fees imposed and collected by PIATCO are entirely within its control. Moreover, with respect to terminal fees, under the 1997 Concession Agreement, the same is further subject to “Interim Adjustments” not previously stipulated in the draft Concession Agreement. Finally, the change in the currency stipulated for “Public Utility Revenues” under the 1997 Concession Agreement, except terminal fees, gives PIATCO an added benefit which was not available at the time of bidding. 2. Under the draft Concession Agreement, default by PIATCO of its obligations does not result in the assumption by government of these liabilities. Under the signed agreement, default by PIATCO of its loans used to finance the project eventually leads to government assumption of the liability for the loans. This is in violation of the BOT Law, which prohibits direct government guarantees. “If a proposal can be denied by reason of the existence of direct government guarantee, then its inclusion in the contract executed after the said proposal has been accepted is likewise sufficient to invalidate the contract itself…To declare the PIATCO contracts valid despite the clear statutory prohibition against a direct government guarantee would not only make a mockery of what the BOT Law seeks to prevent – which is to expose the government to the risk of incurring a monetary obligation resulting from a contract of loan between the project proponent and its lenders and to which the Government is not a party to – but would also render the BOT Law useless for what it seeks to achieve – to make use of the resources of the private sector in the financing, operation and maintenance of infrastructure and development projects which are necessary for national growth and development but which the government, unfortunately, could illafford to finance at this point in time.” 3. Sec. 5.10 of the 1997 Concession Agreement violates Article XII, Sec. 12 of the 1987 Constitution. The Constitutional provision allows for temporary takeover of public facilities in times of national emergency. Since the takeover is temporary and extends only to the operation of the business and not the ownership, government is not required to compensate the owner. Neither can the owner claim just compensation for the use of the business and its properties because the takeover is in exercise of the State’s police power and not of its power of eminent domain. The 1997 Concession Agreement, on the other hand, says that in the event of a takeover, “Concessionaire shall be entitled to reasonable compensation for the duration of the temporary takeover…” “PIATCO cannot, by mere contractual stipulation, contravene the Constitutional provision on temporary government and obligate the government pay reasonable cost for the use of Terminal and/or Terminal Complex. Police power is the most essential, insistent, and illimitable of powers. Its exercise must not be unreasonably hampered nor its exercise be a source of obligation by the government in the absence of damage due to arbitrariness of its exercise. 4. The 1987 Constitution strictly regulates monopolies. Art XII, Sec. 19 says: The State shall regulate or prohibit monopolies when the public interest so requires. The 1997 Concession Agreement gave PIATCO the exclusive right to operate a commercial international passenger terminal within the island of Luzon, with the exception of already existing terminals such as those in the Subic Bay Freeport, Clark Special Economic Zone, and in Laoag City. This privilege, however, is subject to reasonable regulation and supervision and should not violate the rights of third parties. There are service providers at the NAIA I with existing contracts with the MIAA valid until 2010; since the 1997

Concession Agreement says PIATCO is not bound to honor existing contracts with MIAA, transferring operations from NAIA I to NAIA III would unduly prejudice them. “PIATCO cannot, by law and certainly not by contract, render a valid and binding contract nugatory. PIATCO, by the mere expedient claiming an exclusive right to operate, cannot require the Government to break its contractual obligations to the service providers.” IN SUM, THIS COURT RULES THAT IN VIEW OF THE ABSENCE OF THE REQUISITE FINANCIAL CAPACITY OF THE PAIRCARGO CONSORTIUM, PREDECESSOR OF RESPONDENT PIATCO, THE AWARD BY THE PBAC OF THE CONTRACT FOR THE CONSTRUCTION, OPERATION AND MAINTENANCE OF THE NAIA IPT III IS NULL AND VOID. FURTHER, CONSIDERING THAT THE 1997 CONCESSION AGREEMENT CONTAINS MATERIAL AND SUBSTANTIAL AMENDMENTS, WHICH AMENDMENTS HAD THE EFFECT OF CONVERTING THE 1997 CONCESSION AGREEMENT INTO AN ENTIRELY DIFFERENT AGREEMENT FROM THE CONTRACT BIDDED UPON, THE 1997 CONCESSION AGREEMENT IS SIMILARLY NULL AND VOID FOR BEING CONTRARY TO PUBLIC POLICY. THE PROVISIONS UNDER SECTIONS 4.04(B) AND (C) IN RELATION TO SECTION 1.06 OF THE 1997 CONCESSION AGREEMENT AND SECTION 4.04 (C) IN RELATION TO SECTION 1.06 OF THE ARCA, WHICH CONSTITUTE A DIRECT GOVERNMENT GUARANTEE EXPRESSLY PROHIBITED BY, AMONG OTHERS, THE BOT LAW AND ITS IMPLEMENTING RULES AND REGULATIONS ARE ALSO NULL AND VOID. THE SUPPLEMENTS, BEING ACCESSORY CONTRACTS TO THE ARCA, ARE LIKEWISE NULL AND VOID. JUDGMENT: The 1997 Concession Agreement, the Amended and Restated Concession Agreement and the Supplements thereto are set aside for being null and void. ***VITUG: separate opinion – court has no jurisdiction. Petition prays for nullification of contract and does not involve judicial, quasi-judicial or ministerial functions. The parties allege contentious evidentiary facts and “it would be difficult to decide the jurisdictional issue on the basis of the contradictory factual submissions made by the parties.” ***issues on standing: piatco tried to raise the issue of jurisdiction saying they had filed a case with the international chamber of commerce, international court of arbitration for arbitration. The court however pointed out that it includes multiple contracts and not all of the petitioners are parties to the contract.

La Bugal-B’laan Tribal Association, Inc. vs. Ramos

Ponente: Panganiban Civil Action for Prohibition and Mandamus

Date: 1 December 2004

Nature: Special

Parties Involved:

La Bugal-B’laan Tribal Association, Inc., represented by its Chairman F’long Miguel M. Lumayong; et. al. (petitioner

Victor Ramos, DENR Sec.; Horacio Ramos, MGB-DENR Dir.; Ruben Torres, Exec. Sec. WMCP, Inc. (respondent)

Facts and Background of the Case -

-

24 January 2007: the Court en banc declared unconstitutional: (a) certain provisions of RA No. 7942 (Mining Law), (b) its Implementing Rules and Regulations DAO No. 96-40, and (c) the Financial and Technical Assistance Agreement (FTAA) dated March 30, 1995, executed by the government with Western Mining Corporation (Philippines), Inc. (WMCP), mainly on the finding that FTAAs are service contracts prohibited by the 1987 Constitution Respondents filed separate Motions for Reconsideration

Legal Issues A.

W/N case has been rendered moot by the sale of WMC shares in WMCP to Sagittarius Mines, Inc. (a Filipino corporation) and by the subsequent transfer and registration of the FTAA from WMCP to Sagittarius. B. W/N it is still proper to resolve constitutionality of the assailed provisions, assuming that the case has been rendered moot. C. the proper interpretation of the phrase: Agreements Involving Either Technical or Financial Assistance, contained in paragraph 4 of Section 2 of Article XII of the Constitution. D. thus, W/N provisions are unconstitutional. Contentions and Resolutions A. The case has not become moot: 1- the alleged invalidity of the transfer of the WMCP shares to Sagittarius violates the fourth paragraph of Section 2 of Article XII of the Constitution. Petitioners claim that a Filipino corporation is not allowed by the Constitution to enter into an FTAA with the government. 2-it is contrary to the provisions of the WMCP FTA itself, in that the FTAA was intended to apply solely to a foreign corporation, as can allegedly be seen from the provisions therein. [Section 12, which provides for international commercial arbitration under the auspices of the International Chamber of Commerce, after local remedies are exhausted—clearly intended to apply only to a foreign contractor] 3-the sale of the shares is suspect and should therefore be the subject of a case in

Court A. 1- Nowhere in the provision is there any express limitation or restriction insofar as arrangements other than the three mentioned contractual schemes are concerned. There is no reason to believe that the framers of the Constitution, a majority of whom where obviously concerned with furthering the development and utilization of the country’s natural resources, could have wanted to restrict Filipino participation in that area. 2- This provision does not necessarily imply that the WMCP FTAA cannot be transferred to and assumed by a Filipino corporation like Sagittarius, in which event the said provision should simply be



which it validity may be properly litigated. Section 40 of RA 7942 allegedly requires the President’s prior approval of a transfer.

Petitioners also insist that FTAA is void, and hence, cannot be transferred. Its transfer does not operate to cure the constitutional infirmity that is inherent in it; neither will a change in the circumstances of one of the parties serve to ratify the void contract.

B. 1- What is at issue is not only the validity of the WMCP-FTAA, but also the constitutionality of RA 7942 and its Implementing Rules and Regulations 2- the acts of private respondent cannot operate to cure the law of its alleged unconstitutionality or to divest this Court of its jurisdiction to decide. 3- the Constitution imposes upon the Supreme Court the duty to declare invalid any law that offends the Constitution.

C. Petitioners claim that the phrase ”agreements x x x involving either technical or financial assistance” simply means technical assistance or financial assistance agreements, nothing more and nothing else, and such a limitation, they argue, excludes foreign management and operation of a mining enterprise. Court’s previous decision was correct, in

disregarded as a superfluity. 3- Section 40 expressly applies to the assignment or transfer of the FTAA, not to the sale and transfer of shares of stock in WMCP. When the transfer of the FTAA happens to be a Filipino corporation, the need for such safeguard is not critical. Petitioners have assumed as fact that which has yet to be established. The Decision of the Court declaring the FTAA void has not yet become final. The FTAA is not per se defective or unconstitutional. It was questioned only because it had been issued to an allegedly non-qualified, foreign-owned corporation. B. The court concedes that there exists the distinct possibility that one or more of the future FTAAs will be the subject of yet another suit grounded on constitutional issues. [as of June 2002, some 43 FTAAs had been filed] The Court must recognize the exceptional character of the situation and the paramount public interest involved, as well as the necessity for a ruling to put an end to the uncertainties plaguing the mining industry and the affected communities as a result of doubts cast upon the constitutionality and validity of the Mining Act, the subject FTAA and future FTAAs, and the need to avert a multiplicity of suits. The real issue in this case is whether paragraph 4 of Section 2 of Article XII of the Constitution is contravened by RA 7942 and DAO 96-40. By the mere enactment of the questioned law or the approval of the challenged action, the dispute is said to have ripened into a judicial controversy even without any other overt act. C. The use of the word “involving” signifies the possibility of the inclusion of other forms of assistance or activities having to do with, otherwise related to or compatible with financial or technical assistance. These agreements with foreign corporations are not limited to mere financial or technical assistance. Otherwise, the language of the drafters would have certainly been so

B. The courts will decide a question— otherwise moot and academic—if it is “capable of repetition yet evading review.”

C. The FTAA allowed in the Constitution is a comprehensive agreement for the foreign-owned corporation’s integrated exploration, development and utilization of mineral, petroleum or other mineral oils on a large-scale basis.

that the FTAA allowed the foreign contractor to have direct and exclusive management of a mining enterprise, and vests in the foreign company beneficial ownership of our mineral resources.

The deletion or omission from the 1987 Constitution of the term “service contracts” found in the 1973 Constitution sufficiently proves the drafters’ intent to exclude foreigners from the management of the affected enterprises.

D. RA 7942, as well as its Implementing Rules and Regulations, makes it possible for FTAA contracts to cede full control and management of mining enterprises over to fully foreign-owned corporations, with the result that the State is allegedly reduced to a passive regulator dependent on submitted plans and reports, with weak review and audit powers. The law, the implementing regulations, and the WMCP FTAA cede “beneficial ownership” of the mineral resources to the foreign contractor. Section 3 of RA 7942—which allows a foreign contractor to apply for and hold an exploration permit—is unconstitutional.

unmistakably restrictive and stringent as to leave no doubt in anyone’s mind about their true intent. There was a conscious and deliberate decision to avoid the use of restrictive wording that bespeaks an intent not to use the expression “agreements x x x involving either technical or financial assistance” in an exclusionary and limiting manner. Such intent cannot be definitively and conclusively established from the mere failure to carry the same expression or term over to the new Constitution, absent amore specific, explicit and unequivocal statement to that effect. Pertinent portions of the deliberations of the members of the ConCom conclusively show that they discussed agreements involving either technical or financial assistance in the same breadth as service contracts and used the terms interchangeably. D. The gamut of requirements, regulations, restrictions and limitations imposed upon the FTAA contractor by the statute and regulations easily overturns petitioners’ contention. The FTAA contractor is not free to do whatever it pleases and get away with it; on the contrary, it will have to follow the government line if it wants to stay in the enterprise. Ineluctably then, RA 7942 and DAO 96-40 vest in the government more than a sufficient degree of control and supervision over the conduct of mining operations. The exploration permit serves a practical and legitimate purpose in that it protects the interests and preserves the rights of the exploration permit grantee (the would-be contractor)—foreign or local— during the period of time that it is spending heavily on exploration works, without yet being able to earn revenues to recoup any of its investments and expenditures. The evaluation and analysis of the FTAA provisions sufficiently overturn petitioners’ litany of objections to and criticisms of the State’s alleged lack of control. The provisions vest the State

RA 7942 allegedly limits the State’s share in FTAAs with foreign contractors to just taxes, fees and duties, and depriving the State of a share in the after-tax income of the enterprise.

with control and supervision over practically all aspects of the operations of the FTAA contractor, including the charging of pre-operating and operating expenses, and the disposition of mineral products. The Court does not share the view that in FTAAs with foreign contractors under RA 7942, the government’s share is limited to taxes, fees and duties. The inclusion of the phrase “among other things” in the second paragraph of Section 81 clearly and unmistakable reveals the legislative intent to have the State collect more than just the usual taxes, duties and fees. BUT, the WMCP FTAA has invalid provisions: - Section 7.9 of the WMCP FTAA clearly renders illusory the State’s 60 percent share of WMCP’s revenues in providing that: should WMCP’s foreign stockholders sell 60 percent or more of their equity to a Filipino citizen or corporation, the State loses its right to receive its share in net mining revenues under Section 7.7, with any offsetting compensation to the State. - Section 7.8 is likewise invalid, since by allowing the sums spent by government for the benefit of the contractor to be deductible from the State’s share in net mining revenues, it results in benefiting the contractor twice over. This constitute unjust enrichment on the part of the contractor, at the expense of the government

Judgment A.

Because of the validity of sale and the transfer, and that the FTAA is not void per se, the case has become moot. However, B. there is a need to resolve the unconstitutionality of the assailed provisions. C. The phrase “agreements involving either technical and financial assistance,” as used interchangeably with the term “service contracts” by the drafters of the Constitution and as interpreted by the court, is not exclusionary and limiting. D. The assailed provisions are not unconstitutional.

Important Notes

-

The FTAA is for the exploration, development and commercial exploitation of mineral deposits in South Cotabato, Sultan Kudarat, Davao del Sur and North Cotabato, covering an area of 99,837 ha. WMC is wholly owned subsidiary of Western Mining Corporation Holdings Limited, a publicly listed major Australian mining and exploration company. A service contract has been defined as a contractual agreement for engaging in the exploitation and development of petroleum, mineral, energy, land and other natural resources, whereby a government or an agency thereof, or a private person granted a right or privilege by said government, authorizes the other party—the service contractor—to engage or participate in the exercise of such right or the enjoyment of the privilege, by providing financial or technical resources, undertaking the exploitation or production of a given resource, or directly managing the productive enterprise, operations of the exploration and exploitation of the resources, or the disposition or marketing of said resources.

Personal Notes As to the case being a political question, the EPILOGUE provides: “The crux of the controversy is the amount of discretion to be accorded the Executive Department, particularly the President of the Republic, in respect of negotiations over the terms of FTAAs, particularly when it comes to the government share of financial benefits from FTAAs.” “Verily, under the doctrine of separation of powers and due respect for co-equal and coordinate branches of government, this Court must restrain itself from intruding into policy matters and must allow the President and Congress maximum discretion in using the resources of our country and in securing the assistance of foreign groups to eradicate the grinding poverty of our people and answer their cry or viable employment opportunities in this country.” “Let the development of the mining industry be the responsibility of the political branches of the government. And let not this Court interfere inordinately and unnecessarily.”

- sir pointed out: although the issue is very technical, we must all go back to the basic issue of whether or not Congress, DENR and the President acted within jurisdiction. The issue is always about jurisdiction. > and in this case, the court found that they did act within their respective jurisdictions.

Related Documents


More Documents from ""